НайдитС ΠΎΠ±Π»Π°ΡΡ‚ΡŒ опрСдСлСния Ρ„ΡƒΠ½ΠΊΡ†ΠΈΠΈ y 4 x: Mathway | ΠŸΠΎΠΏΡƒΠ»ΡΡ€Π½Ρ‹Π΅ Π·Π°Π΄Π°Ρ‡ΠΈ

36Risolvere per ?cos(x)=1/27Risolvere per xsin(x)=-1/28ΠŸΡ€Π΅ΠΎΠ±Ρ€Π°Π·ΠΎΠ²Π°Ρ‚ΡŒ ΠΈΠ· градусов Π² Ρ€Π°Π΄ΠΈΠ°Π½Ρ‹2259Risolvere per ?cos(x)=( ΠΊΠ²Π°Π΄Ρ€Π°Ρ‚Π½Ρ‹ΠΉ ΠΊΠΎΡ€Π΅Π½ΡŒ ΠΈΠ· 2)/210Risolvere per xcos(x)=( ΠΊΠ²Π°Π΄Ρ€Π°Ρ‚Π½Ρ‹ΠΉ ΠΊΠΎΡ€Π΅Π½ΡŒ ΠΈΠ· 3)/211Risolvere per xsin(x)=( ΠΊΠ²Π°Π΄Ρ€Π°Ρ‚Π½Ρ‹ΠΉ ΠΊΠΎΡ€Π΅Π½ΡŒ ΠΈΠ· 3)/212Π“Ρ€Π°Ρ„ΠΈΠΊg(x)=3/4* ΠΊΠΎΡ€Π΅Π½ΡŒ пятой стСпСни ΠΈΠ· x
13Найти Ρ†Π΅Π½Ρ‚Ρ€ ΠΈ радиусx^2+y^2=914ΠŸΡ€Π΅ΠΎΠ±Ρ€Π°Π·ΠΎΠ²Π°Ρ‚ΡŒ ΠΈΠ· градусов Π² Ρ€Π°Π΄ΠΈΠ°Π½Ρ‹120 Π³Ρ€Π°Π΄. 2-4x/2-x

Π‘ΠΎΠ΄Π΅Ρ€ΠΆΠ°Π½ΠΈΠ΅

Π›ΡƒΡ‡ΡˆΠΈΠΉ ΠΎΡ‚Π²Π΅Ρ‚ ΠΏΠΎ мнСнию Π°Π²Ρ‚ΠΎΡ€Π°

09. 05.18
Π›ΡƒΡ‡ΡˆΠΈΠΉ ΠΎΡ‚Π²Π΅Ρ‚ ΠΏΠΎ мнСнию Π°Π²Ρ‚ΠΎΡ€Π°

Π”Ρ€ΡƒΠ³ΠΈΠ΅ ΠΎΡ‚Π²Π΅Ρ‚Ρ‹

ΠΊΠ°ΠΊ Π΄ΠΎΠ»ΠΆΠ½ΠΎ выглядит сама функция?

09. 2 большС Ρ€Π°Π²Π½ΠΎ 3x. Если ΠΊΠΎΡ€Π΅Π½ΡŒ стоит Π½Π° всСй Ρ„ΡƒΠ½ΠΊΡ†ΠΈΠΈ

09.05.18

ΠœΠΈΡ…Π°ΠΈΠ» АлСксандров

Π§ΠΈΡ‚Π°Ρ‚ΡŒ ΠΎΡ‚Π²Π΅Ρ‚Ρ‹

АндрСй АндрССвич

Π§ΠΈΡ‚Π°Ρ‚ΡŒ ΠΎΡ‚Π²Π΅Ρ‚Ρ‹

Eleonora Gabrielyan

Π§ΠΈΡ‚Π°Ρ‚ΡŒ ΠΎΡ‚Π²Π΅Ρ‚Ρ‹

ΠŸΠΎΡΠΌΠΎΡ‚Ρ€Π΅Ρ‚ΡŒ всСх экспСртов ΠΈΠ· Ρ€Π°Π·Π΄Π΅Π»Π° Π£Ρ‡Π΅Π±Π° ΠΈ Π½Π°ΡƒΠΊΠ°

ΠŸΠΎΡ…ΠΎΠΆΠΈΠ΅ вопросы

РСшСно

К окруТности с Ρ†Π΅Π½Ρ‚Ρ€ΠΎΠΌ О ΠΏΡ€ΠΎΠ²Π΅Π΄Π΅Π½Π° ΠΊΠ°ΡΠ°Ρ‚Π΅Π»ΡŒΠ½Π°Ρ AB (А — Ρ‚ΠΎΡ‡ΠΊΠ° касания).

НайдитС радиус окруТности, Ссли OB = 10 см ΠΈ ΡƒΠ³ΠΎΠ» ABO = 30 градусам.

Π‘ΠΈΠ»Π΅Ρ‚Ρ‹ ΠΏΠΎ Π³Π΅ΠΎΠΌΠ΅Ρ‚Ρ€ΠΈΠΈ 7 класс Π‘ΠΈΠ»Π΅Ρ‚ β„–1. 1. Π’ΠΎΡ‡ΠΊΠΈ. ΠŸΡ€ΡΠΌΡ‹Π΅. ΠžΡ‚Ρ€Π΅Π·ΠΊΠΈ. 2. Π‘Ρ„ΠΎΡ€ΠΌΡƒΠ»ΠΈΡ€ΠΎΠ²Π°Ρ‚ΡŒ ΠΈ Π΄ΠΎΠΊΠ°Π·Π°Ρ‚ΡŒ Ρ‚Π΅ΠΎΡ€Π΅ΠΌΡƒ, Π²Ρ‹Ρ€Π°ΠΆΠ°ΡŽΡ‰ΡƒΡŽ Ρ‚Ρ€Π΅Ρ‚ΠΈΠΉ ΠΏΡ€ΠΈΠ·Π½Π°ΠΊ равСнства

ΠΠ°ΠΏΠΈΡˆΠΈΡ‚Π΅ сочинСниС Π½Π° Ρ‚Π΅ΠΌΡƒΒ«ΠšΠΎΠ³Π΄Π° моя ΠΌΠ°ΠΌΠ°(сСстра, со сСд, Π΄Π΅Π΄ΡƒΡˆΠΊΠ° ΠΈ Ρ‚. Π΄.) ΡƒΡ‡ΠΈΠ»Π°ΡΡŒ Π² школС(Π½Π°Ρ‡ΠΈΠ½Π°Π»Π° Ρ€Π°Π±ΠΎΡ‚Π°Ρ‚ΡŒ, Π²ΠΎΠ΅Π²Π°Π»Π°, ΠΏΡƒΡ‚Π΅ΡˆΠ΅ΡΡ‚Π²ΠΎΠ²Π°Π»Π°, ΠΎΡ‚Π΄Ρ‹Ρ…Π°Π»Π° Π² Π³ΠΎΡ€Π°Ρ… ΠΈ Ρ‚.ΠΏ.)Β». Π£Π·Π½Π°ΠΉΡ‚Π΅ Ρƒ ΡΡ‚Π°Ρ€ΡˆΠΈΡ… ΠΎ Ρ‚ΠΎΠΌ,

Π‘ΠΎΠΊΠΎΠ²ΠΎΠ΅ Ρ€Π΅Π±Ρ€ΠΎ ΠΏΡ€Π°Π²ΠΈΠ»ΡŒΠ½ΠΎΠΉ Ρ‚Ρ€Π΅ΡƒΠ³ΠΎΠ»ΡŒΠ½ΠΎΠΉ ΠΏΡ€ΠΈΠ·ΠΌΡ‹ Ρ€Π°Π²Π½ΠΎ 9 см,Π° диагональ Π±ΠΎΠΊΠΎΠ²ΠΎΠΉ Π³Ρ€Π°Π½ΠΈ Ρ€Π°Π²Π½Π° 15 см. Найти ΠΏΠ»ΠΎΡ‰Π°Π΄ΡŒ Π±ΠΎΠΊΠΎΠ²ΠΎΠΉ ΠΈ ΠΏΠΎΠ»Π½ΠΎΠΉ повСрхности ΠΏΡ€ΠΈΠ·ΠΌΡ‹

РСшСно

Радиус вписанной Π² ΠΊΠ²Π°Π΄Ρ€Π°Ρ‚ окруТности Ρ€Π°Π²Π΅Π½ 4√2 Π½Π°ΠΉΡ‚ΠΈ радиус окруТности описанной ΠΎΠΊΠΎΠ»ΠΎ этого ΠΊΠ²Π°Π΄Ρ€Π°Ρ‚Π°

ΠŸΠΎΠ»ΡŒΠ·ΡƒΠΉΡ‚Π΅ΡΡŒ нашим ΠΏΡ€ΠΈΠ»ΠΎΠΆΠ΅Π½ΠΈΠ΅ΠΌ

ΠžΠ±Π»Π°ΡΡ‚ΡŒ опрСдСлСния Ρ„ΡƒΠ½ΠΊΡ†ΠΈΠΈ — прСзСнтация ΠΎΠ½Π»Π°ΠΉΠ½

ΠŸΠΎΡ…ΠΎΠΆΠΈΠ΅ ΠΏΡ€Π΅Π·Π΅Π½Ρ‚Π°Ρ†ΠΈΠΈ:

Π­Π»Π΅ΠΌΠ΅Π½Ρ‚Ρ‹ ΠΊΠΎΠΌΠ±ΠΈΠ½Π°Ρ‚ΠΎΡ€ΠΈΠΊΠΈ ( 9-11 классы)

ΠŸΡ€ΠΈΠΌΠ΅Π½Π΅Π½ΠΈΠ΅ ΠΏΡ€ΠΎΠΈΠ·Π²ΠΎΠ΄Π½ΠΎΠΉ Π² Π½Π°ΡƒΠΊΠ΅ ΠΈ Π² ΠΆΠΈΠ·Π½ΠΈ

ΠŸΡ€ΠΎΠ΅ΠΊΡ‚ ΠΏΠΎ ΠΌΠ°Ρ‚Π΅ΠΌΠ°Ρ‚ΠΈΠΊΠ΅ Β«ΠœΠ°Ρ‚Π΅ΠΌΠ°Ρ‚ΠΈΠΊΠ° Π²ΠΎΠΊΡ€ΡƒΠ³ нас. Π£Π·ΠΎΡ€Ρ‹ ΠΈ ΠΎΡ€Π½Π°ΠΌΠ΅Π½Ρ‚Ρ‹ Π½Π° посудС»

Знакомство Π΄Π΅Ρ‚Π΅ΠΉ с матСматичСскими Π·Π½Π°ΠΊΠ°ΠΌΠΈ ΠΈ ΠΌΠΎΠ½Π΅Ρ‚Π°ΠΌΠΈ

Π’Ρ€Π΅Π½Π°ΠΆΡ‘Ρ€ ΠΏΠΎ ΠΌΠ°Ρ‚Π΅ΠΌΠ°Ρ‚ΠΈΠΊΠ΅ Β«Π‘ΠΎΠ±ΠΈΡ€Π°Π΅ΠΌ ΡƒΡ€ΠΎΠΆΠ°ΠΉΒ». Π‘Ρ‡Π΅Ρ‚ Π² ΠΏΡ€Π΅Π΄Π΅Π»Π°Ρ… 10

ΠœΠ΅Ρ‚ΠΎΠ΄Ρ‹ ΠΎΠ±Ρ€Π°Π±ΠΎΡ‚ΠΊΠΈ ΡΠΊΡΠΏΠ΅Ρ€ΠΈΠΌΠ΅Π½Ρ‚Π°Π»ΡŒΠ½Ρ‹Ρ… Π΄Π°Π½Π½Ρ‹Ρ…

ЛСкция 6. ΠšΠΎΡ€Ρ€Π΅Π»ΡΡ†ΠΈΠΎΠ½Π½Ρ‹ΠΉ ΠΈ рСгрСссионный Π°Π½Π°Π»ΠΈΠ·

РСшСниС Π·Π°Π΄Π°Ρ‡ ΠΎΠ±ΡΠ·Π°Ρ‚Π΅Π»ΡŒΠ½ΠΎΠΉ части ΠžΠ“Π­ ΠΏΠΎ Π³Π΅ΠΎΠΌΠ΅Ρ‚Ρ€ΠΈΠΈ

Π”ΠΈΡ„Ρ„Π΅Ρ€Π΅Π½Ρ†ΠΈΠ°Π»ΡŒΠ½Ρ‹Π΅ уравнСния

ΠŸΠΎΠ΄Π³ΠΎΡ‚ΠΎΠ²ΠΊΠ° ΠΊ Π•Π“Π­ ΠΏΠΎ ΠΌΠ°Ρ‚Π΅ΠΌΠ°Ρ‚ΠΈΠΊΠ΅. Π‘Π°Π·ΠΎΠ²Ρ‹ΠΉ ΡƒΡ€ΠΎΠ²Π΅Π½ΡŒ Π‘Π»ΠΎΠΆΠ½Ρ‹Π΅ Π·Π°Π΄Π°Ρ‡ΠΈ

1. Π£Ρ€ΠΎΠΊ ΠΌΠ°Ρ‚Π΅ΠΌΠ°Ρ‚ΠΈΠΊΠΈ.

Β«Π‘ Ρ‚Π΅Ρ… ΠΏΠΎΡ€ ΠΊΠ°ΠΊ сущСствуСт
ΠΌΠΈΡ€ΠΎΠ·Π΄Π°Π½ΡŒΠ΅,
Π’Π°ΠΊΠΎΠ³ΠΎ Π½Π΅Ρ‚, ΠΊΡ‚ΠΎ Π± Π½Π΅ нуТдался
в знаньС.
Какой ΠΌΡ‹ Π½ΠΈ возьмСм язык ΠΈ
Π²Π΅ΠΊ,
ВсСгда стрСмится ΠΊ знанью
Ρ‡Π΅Π»ΠΎΠ²Π΅ΠΊ Β»

3. ΠžΡΠ½ΠΎΠ²Π½Ρ‹Π΅ этапы ΡƒΡ€ΠΎΠΊΠ°:

ΠœΠ°Ρ‚Π΅ΠΌΠ°Ρ‚ΠΈΡ‡Π΅ΡΠΊΠ°Ρ
Ρ€Π°Π·ΠΌΠΈΠ½ΠΊΠ°.
Устная коллСктивная Ρ€Π°Π±ΠΎΡ‚Π°.
Π‘Π°ΠΌΠΎΡΡ‚ΠΎΡΡ‚Π΅Π»ΡŒΠ½Π°Ρ Ρ€Π°Π±ΠΎΡ‚Π° Π² Π³Ρ€ΡƒΠΏΠΏΠ°Ρ….
РСфлСксия.

4. Π Π°Π·ΠΌΠΈΠ½ΠΊΠ° 1 ΠΊΠΎΠΌΠ°Π½Π΄Π°.

Как называСтся пСрСмСнная
ΠΎΠ±Ρ€Π°Π·ΡƒΡŽΡ‰Π°Ρ ΠΎΠ±Π»Π°ΡΡ‚ΡŒ опрСдСлСния
Ρ„ΡƒΠ½ΠΊΡ†ΠΈΠΈ?
Какая ось опрСдСляСт значСния
Π°Ρ€Π³ΡƒΠΌΠ΅Π½Ρ‚Π°?
Как ΠΊΡ€Π°Ρ‚ΠΊΠΎ Π·Π°ΠΏΠΈΡΠ°Ρ‚ΡŒ Β«ΠΎΠ±Π»Π°ΡΡ‚ΡŒ значСния
Ρ„ΡƒΠ½ΠΊΡ†ΠΈΠΈΒ»
Как называСтся функция, Π³Ρ€Π°Ρ„ΠΈΠΊ
ΠΊΠΎΡ‚ΠΎΡ€ΠΎΠΉ – прямая?

5.

ΠŸΡ€ΠΎΠ΄ΠΎΠ»ΠΆΠ΅Π½ΠΈΠ΅ Ρ€Π°Π·ΠΌΠΈΠ½ΠΊΠΈΠšΠ°ΠΊΠΈΠΌ числом Π½Π΅ ΠΌΠΎΠΆΠ΅Ρ‚ Π±Ρ‹Ρ‚ΡŒ
ΠΏΠΎΠΊΠ°Π·Π°Ρ‚Π΅Π»ΡŒΠ½Π°Ρ Ρ„ΡƒΠ½ΠΊΡ†ΠΈΠΈ?
Π§Ρ‚ΠΎ Π΅ΡΡ‚ΡŒ Π² Π·Π΅ΠΌΠ»Π΅, Π² словС ΠΈ Π² ΡƒΡ€Π°Π²Π½Π΅Π½ΠΈΠ΅?
Бколько ΠΊΠΎΡ€Π½Π΅ΠΉ ΠΌΠΎΠΆΠ΅Ρ‚ ΠΈΠΌΠ΅Ρ‚ΡŒ ΠΊΠ²Π°Π΄Ρ€Π°Ρ‚Π½ΠΎΠ΅
ΡƒΡ€Π°Π²Π½Π΅Π½ΠΈΠ΅?
Как называСтся Ρ€Π°Π·Π΄Π΅Π» ΠΌΠ°Ρ‚Π΅ΠΌΠ°Ρ‚ΠΈΠΊΠΈ,
ΠΊΠΎΡ‚ΠΎΡ€Ρ‹ΠΉ ΠΈΠ·ΡƒΡ‡Π°Π΅Ρ‚ свойства Ρ„ΠΈΠ³ΡƒΡ€?
Π§Ρ‚ΠΎ Ρ‚Π°ΠΊΠΎΠ΅ функция?

6. Π Π°Π·ΠΌΠΈΠ½ΠΊΠ° 2 ΠΊΠΎΠΌΠ°Π½Π΄Π°

Как называСтся пСрСмСнная которая
ΠΎΠ±Ρ€Π°Π·ΡƒΠ΅Ρ‚ ΠΎΠ±Π»Π°ΡΡ‚ΡŒ значСния Ρ„ΡƒΠ½ΠΊΡ†ΠΈΠΈ?
Какая ось опрСдСляСт значСния
Ρ„ΡƒΠ½ΠΊΡ†ΠΈΠΈ?
Как ΠΊΡ€Π°Ρ‚ΠΊΠΎ Π·Π°ΠΏΠΈΡΠ°Ρ‚ΡŒ Β«ΠΎΠ±Π»Π°ΡΡ‚ΡŒ
опрСдСлСния Ρ„ΡƒΠ½ΠΊΡ†ΠΈΠΈΒ»?
Как называСтся Ρ„ΡƒΠ½ΠΊΡ†ΠΈΠΈ, Π³Ρ€Π°Ρ„ΠΈΠΊΠΎΠΌ
ΠΊΠΎΡ‚ΠΎΡ€ΠΎΠΉ являСтся ΠΏΠ°Ρ€Π°Π±ΠΎΠ»Π°?
Π§Ρ‚ΠΎ Ρ‚Π°ΠΊΠΎΠ΅ ΡƒΡ€Π°Π²Π½Π΅Π½ΠΈΠ΅?

7. ΠŸΡ€ΠΎΠ΄ΠΎΠ»ΠΆΠ΅Π½ΠΈΠ΅ Ρ€Π°Π·ΠΌΠΈΠ½ΠΊΠΈ

Бколько
ΠΊΠΎΡ€Π½Π΅ΠΉ ΠΌΠΎΠΆΠ΅Ρ‚ ΠΈΠΌΠ΅Ρ‚ΡŒ
Π»ΠΈΠ½Π΅ΠΉΠ½ΠΎΠ΅ ΡƒΡ€Π°Π²Π½Π΅Π½ΠΈΠ΅?
Π§Ρ‚ΠΎ Π»Π΅Π³Ρ‡Π΅ 1 ΠΊΠ³. Π–Π΅Π»Π΅Π·Π° ΠΈΠ»ΠΈ 1 ΠΊΠ³.
ΠŸΡƒΡ…Π°?
Как называСтся Ρ€Π°Π·Π΄Π΅Π» ΠΌΠ°Ρ‚Π΅ΠΌΠ°Ρ‚ΠΈΠΊΠΈ,
ΠΊΠΎΡ‚ΠΎΡ€Ρ‹ΠΉ ΠΈΠ·ΡƒΡ‡Π°Π΅Ρ‚ ΠΏΡ€ΠΎΠΈΠ·Π²ΠΎΠ΄Π½ΡƒΡŽ ΠΈ
ΠΏΠ΅Ρ€Π²ΠΎΠΎΠ±Ρ€Π°Π·Π½ΡƒΡŽ Ρ„ΡƒΠ½ΠΊΡ†ΠΈΠΈ?
Π§Ρ‚ΠΎ ΠΎΠ±Ρ‰Π΅Π³ΠΎ ΠΌΠ΅ΠΆΠ΄Ρƒ качСлями,
ΠΌΡƒΠ·Ρ‹ΠΊΠΎΠΉ ΠΈ свСтом?

8. ΠžΡΠ½ΠΎΠ²Π½Ρ‹Π΅ вопросы ΡƒΡ€ΠΎΠΊΠ°.

Π§Ρ‚ΠΎ
Ρ‚Π°ΠΊΠΎΠ΅ функция?
Π§Ρ‚ΠΎ Ρ‚Π°ΠΊΠΎΠ΅ ΠΎΠ±Π»Π°ΡΡ‚ΡŒ опрСдСлСния
Ρ„ΡƒΠ½ΠΊΡ†ΠΈΠΈ?
Π§Ρ‚ΠΎ Ρ‚Π°ΠΊΠΎΠ΅ Π³Ρ€Π°Ρ„ΠΈΠΊ Ρ„ΡƒΠ½ΠΊΡ†ΠΈΠΈ?
ΠžΡΠ½ΠΎΠ²Π½Ρ‹Π΅ условия сущСствования
Ρ„ΡƒΠ½ΠΊΡ†ΠΈΠΈ?

9.

ΠžΠΏΡ€Π΅Π΄Π΅Π»Π΅Π½ΠΈΠ΅ Ρ„ΡƒΠ½ΠΊΡ†ΠΈΠΈΠ€ΡƒΠ½ΠΊΡ†ΠΈΠ΅ΠΉ
называСтся Π·Π°Π²ΠΈΡΠΈΠΌΠΎΡΡ‚ΡŒ
ΠΎΠ΄Π½ΠΎΠΉ ΠΏΠ΅Ρ€Π΅ΠΌΠ΅Π½Π½ΠΎΠΉ ΠΎΡ‚ Π΄Ρ€ΡƒΠ³ΠΎΠΉ, ΠΏΡ€ΠΈ
ΠΊΠΎΡ‚ΠΎΡ€ΠΎΠΌ ΠΊΠ°ΠΆΠ΄ΠΎΠΌΡƒ Π·Π½Π°Ρ‡Π΅Π½ΠΈΡŽ
нСзависимой ΠΏΠ΅Ρ€Π΅ΠΌΠ΅Π½Π½ΠΎΠΉ
соотвСтствуСт СдинствСнноС Π·Π½Π°Ρ‡Π΅Π½ΠΈΠ΅
зависимой ΠΏΠ΅Ρ€Π΅ΠΌΠ΅Π½Π½ΠΎΠΉ.
ΠžΠ±ΠΎΠ·Π½Π°Ρ‡Π°ΡŽΡ‚: Ρƒ= f(x)

10. Π—Π°Π΄Π°Π½ΠΈΠ΅: Π―Π²Π»ΡΡŽΡ‚ΡΡ Π»ΠΈ ΠΈΠ·ΠΎΠ±Ρ€Π°ΠΆΠ΅Π½Π½Ρ‹Π΅ Π½Π° рисунках Π»ΠΈΠ½ΠΈΠΈ Π³Ρ€Π°Ρ„ΠΈΠΊΠ°ΠΌΠΈ Π½Π΅ΠΊΠΎΡ‚ΠΎΡ€Ρ‹Ρ… Ρ„ΡƒΠ½ΠΊΡ†ΠΈΠΉ?

Рис.1
Рис.2
Ρƒ
Ρƒ
Ρ…
Ρ…

11. Π—Π°Π΄Π°Π½ΠΈΠ΅: Π―Π²Π»ΡΡŽΡ‚ΡΡ Π»ΠΈ ΠΈΠ·ΠΎΠ±Ρ€Π°ΠΆΠ΅Π½Π½Ρ‹Π΅ Π½Π° рисунках Π»ΠΈΠ½ΠΈΠΈ Π³Ρ€Π°Ρ„ΠΈΠΊΠ°ΠΌΠΈ Π½Π΅ΠΊΠΎΡ‚ΠΎΡ€Ρ‹Ρ… Ρ„ΡƒΠ½ΠΊΡ†ΠΈΠΉ?

Рис.3
Ρƒ
Рис.4
Ρ…
Ρƒ
Ρ…

12. ΠžΠ±Π»Π°ΡΡ‚ΡŒ опрСдСлСния Ρ„ΡƒΠ½ΠΊΡ†ΠΈΠΈ

ΠžΠ±Π»Π°ΡΡ‚ΡŒΡŽ
опрСдСлСния
Ρ„ΡƒΠ½ΠΊΡ†ΠΈΠΈ называСтся
мноТСство всСх
Π·Π½Π°Ρ‡Π΅Π½ΠΈΠΉ нСзависимой
ΠΏΠ΅Ρ€Π΅ΠΌΠ΅Π½Π½ΠΎΠΉ Ρ…, Ρ‚.Π΅.
Π°Ρ€Π³ΡƒΠΌΠ΅Π½Ρ‚Π°.

13. Π—Π°Π΄Π°Π½ΠΈΠ΅: Π½Π°ΠΉΡ‚ΠΈ ΠΎΠ±Π»Π°ΡΡ‚ΡŒ опрСдСлСния Ρ„ΡƒΠ½ΠΊΡ†ΠΈΠΈ ΠΏΠΎ Π΅Π΅ Π³Ρ€Π°Ρ„ΠΈΠΊΡƒ

1) D(Ρƒ) 4; 1 1;
2)
D( Ρƒ) ;0 0;2
3)
D(Ρƒ) 4; 1 1;
4)
D( Ρƒ) 4;
Ρƒ
-1
0
Ρ…

14. Π—Π°Π΄Π°Π½ΠΈΠ΅: Π½Π°ΠΉΡ‚ΠΈ ΠΎΠ±Π»Π°ΡΡ‚ΡŒ опрСдСлСния Ρ„ΡƒΠ½ΠΊΡ†ΠΈΠΈ ΠΏΠΎ Π΅Π΅ Π³Ρ€Π°Ρ„ΠΈΠΊΡƒ

1)
D( Ρƒ) ;
2)
D(Ρƒ) ;0 0;
3)
D(Ρƒ) ;2 2;
4)
D( Ρƒ) ;2
Ρƒ
2
0
Ρ…

15.

Π—Π°Π΄Π°Π½ΠΈΠ΅: Π½Π°ΠΉΡ‚ΠΈ ΠΎΠ±Π»Π°ΡΡ‚ΡŒ опрСдСлСния Ρ„ΡƒΠ½ΠΊΡ†ΠΈΠΈ ΠΏΠΎ Π΅Π΅ Π³Ρ€Π°Ρ„ΠΈΠΊΡƒ1)
D(Ρƒ) ;0 2;
Ρƒ
2) D(Ρƒ) ; 1 0;
3)
D( Ρƒ) 0;2 2;
4)
D( Ρƒ) 0;
0
-1
2
Ρ…

16. Условия сущСствования Ρ„ΡƒΠ½ΠΊΡ†ΠΈΠΈ

β„–
1
2
3
4
5
Π€ΠΎΡ€ΠΌΡƒΠ»Π°
g ( x)
f ( x)
f ( x) 0
y 2n f ( x)
1
y
2n f ( x)
y log a f ( x)
f ( x) 0
y log g ( x) b
y
условия
f ( x) 0
f ( x) 0
6
y log g ( x) f ( x)
g ( x) 0;
g ( x ) 1.
g ( x) 0;
g ( x) 1;
f ( x ) 0.
7
8
9
y f ( x) p , Π³Π΄Π΅ p Π½Π΅Ρ†Π΅Π»ΠΎΠ΅, p 0
f ( x) 0 ;
y f ( x) p , Π³Π΄Π΅ p Π½Π΅Ρ†Π΅Π»ΠΎΠ΅, p 0
y tgt
f ( x) 0
10
y ctgt
n, Π³Π΄Π΅n Z
2
t n, Π³Π΄Π΅n Z
t

17. Найти ΠΎΠ±Π»Π°ΡΡ‚ΡŒ опрСдСлСния Ρ„ΡƒΠ½ΠΊΡ†ΠΈΠΈ.

1.
y log 5 2 x 8
2.
y x 3x 4
6
2
y 6 3x
0 , 3
3.
4.
5
y tg 2 x
2

18. ΠžΠ±Ρ€Π°Π·Π΅Ρ† Ρ‚Π°Π±Π»ΠΈΡ†Ρ‹ заполнСния ΠΎΡ‚Π²Π΅Ρ‚ΠΎΠ²

Ѐамилия,
имя
β„–
ΠΎΡ‚Π²Π΅Ρ‚Π°
1
2
3
4
Π—Π°Π΄
β„–1
X
Π—Π°Π΄.
β„–2
Π—Π°Π΄.
β„–3
Π—Π°Π΄
β„–4
Π—Π°Π΄.β„– Π—Π°Π΄.
5
β„–6

19. Π Π΅ΡˆΠΈΡ‚Π΅ ΡΠ°ΠΌΠΎΡΡ‚ΠΎΡΡ‚Π΅Π»ΡŒΠ½ΠΎ

Π’Π°Ρ€ΠΈΠ°Π½Ρ‚ I.
x
.
2x 6
2) ;3 3;
4) ;
1. Найти ΠΎΠ±Π»Π°ΡΡ‚ΡŒ опрСдСлСния Ρ„ΡƒΠ½ΠΊΡ†ΠΈΠΈ: y
ΠžΡ‚Π²Π΅Ρ‚Ρ‹: 1) ;0 0;3 3;
3) ;0 0;
2. Найти ΠΎΠ±Π»Π°ΡΡ‚ΡŒ опрСдСлСния Ρ„ΡƒΠ½ΠΊΡ†ΠΈΠΈ: y x 2 9 .
ΠžΡ‚Π²Π΅Ρ‚Ρ‹: 1) ; 3 3;
2) ;
3) ; 3 3;
4) 3;
3. Найти ΠΎΠ±Π»Π°ΡΡ‚ΡŒ опрСдСлСния Ρ„ΡƒΠ½ΠΊΡ†ΠΈΠΈ: y log 2 2 x 7
ΠžΡ‚Π²Π΅Ρ‚Ρ‹: 1) 0;
2) ;3,5
3) 3,5;
4) 3,5;
4. Найти ΠΎΠ±Π»Π°ΡΡ‚ΡŒ опрСдСлСния Ρ„ΡƒΠ½ΠΊΡ†ΠΈΠΈ: y 4 x 2 2
1
ΠžΡ‚Π²Π΅Ρ‚Ρ‹: 1) 0; 2) 0,5; 3) ; 4) ;
2
2
5. Найти ΠΎΠ±Π»Π°ΡΡ‚ΡŒ опрСдСлСния Ρ„ΡƒΠ½ΠΊΡ†ΠΈΠΈ: y x x 6
ΠžΡ‚Π²Π΅Ρ‚Ρ‹: 1) ; 2 3; 2) 2;3
3) ; 2 3;
4) ; 3 2;
1
x
6. Найти ΠΎΠ±Π»Π°ΡΡ‚ΡŒ опрСдСлСния Ρ„ΡƒΠ½ΠΊΡ†ΠΈΠΈ: y 2 tg sin x
ΠžΡ‚Π²Π΅Ρ‚Ρ‹: 1) x
n , Π³Π΄Π΅ n Z
2
3) x 4 n , Π³Π΄Π΅ n Z
4
2) x n , Π³Π΄Π΅ n Z
4) x 2 4 n , Π³Π΄Π΅ n Z

20. Π Π΅ΡˆΠΈΡ‚Π΅ ΡΠ°ΠΌΠΎΡΡ‚ΠΎΡΡ‚Π΅Π»ΡŒΠ½ΠΎ

Π’Π°Ρ€ΠΈΠ°Π½Ρ‚ II
x
.
4x 8
ΠžΡ‚Π²Π΅Ρ‚Ρ‹: 1) ; 2) ; 2 2;0 0;
3) ;0 0;
4) ; 2 2;
1. Найти ΠΎΠ±Π»Π°ΡΡ‚ΡŒ опрСдСлСния Ρ„ΡƒΠ½ΠΊΡ†ΠΈΠΈ: y
2. Найти ΠΎΠ±Π»Π°ΡΡ‚ΡŒ опрСдСлСния Ρ„ΡƒΠ½ΠΊΡ†ΠΈΠΈ: y 4 x 2 .
ΠžΡ‚Π²Π΅Ρ‚Ρ‹: 1) ;
2) 0;
3) 2;
4) ; 2 2;
3. Найти ΠΎΠ±Π»Π°ΡΡ‚ΡŒ опрСдСлСния Ρ„ΡƒΠ½ΠΊΡ†ΠΈΠΈ: y log 1 8 5 x
ΠžΡ‚Π²Π΅Ρ‚Ρ‹: 1) 0;
4) 1,6;
2) ;1,6
2
3) ;1,6
4. Найти ΠΎΠ±Π»Π°ΡΡ‚ΡŒ опрСдСлСния Ρ„ΡƒΠ½ΠΊΡ†ΠΈΠΈ: y 0,2 x 4 5
ΠžΡ‚Π²Π΅Ρ‚Ρ‹: 1) 20; 2) 0; 3) 20; 4) ;20
5. Найти ΠΎΠ±Π»Π°ΡΡ‚ΡŒ опрСдСлСния Ρ„ΡƒΠ½ΠΊΡ†ΠΈΠΈ: y x 2 4 x 5
ΠžΡ‚Π²Π΅Ρ‚Ρ‹: 1) ; 1 5;
2) 1;5
3) ; 1 5;
4) ; 5 1;
3
1
2
6. Найти ΠΎΠ±Π»Π°ΡΡ‚ΡŒ опрСдСлСния Ρ„ΡƒΠ½ΠΊΡ†ΠΈΠΈ: y ctg 2 x cos x
ΠžΡ‚Π²Π΅Ρ‚Ρ‹: 1) x
n
, Π³Π΄Π΅ n Z
2
n
3) x
, Π³Π΄Π΅ n Z
4
2
2) x n , Π³Π΄Π΅ n Z
4) x 2 n , Π³Π΄Π΅ n Z

English Β  Β  Русский ΠŸΡ€Π°Π²ΠΈΠ»Π°

ΠΊΠ°ΠΊ ΠΎΡ‚ΡΠ»Π΅ΠΆΠΈΠ²Π°Ρ‚ΡŒ Ρ†Π΅Π»ΡŒ Π² ΠΈΠ·ΠΌΠ΅Π½Ρ‡ΠΈΠ²Ρ‹Ρ… условиях сцСны / Π₯Π°Π±Ρ€

БпСциалисты ΠΏΠΎ ΠΊΠΎΠΌΠΏΡŒΡŽΡ‚Π΅Ρ€Π½ΠΎΠΌΡƒ Π·Ρ€Π΅Π½ΠΈΡŽ Π½Π΅ ΠΎΠ΄ΠΈΠ½ дСсяток Π»Π΅Ρ‚ Π±ΡŒΡŽΡ‚ΡΡ Π½Π°Π΄ Ρ‚Ρ€Π΅ΠΊΠΈΠ½Π³ΠΎΠΌ ΠΎΠ±ΡŠΠ΅ΠΊΡ‚ΠΎΠ². Они ΠΏΠ΅Ρ€Π΅ΠΏΡ€ΠΎΠ±ΠΎΠ²Π°Π»ΠΈ ΠΌΠ½ΠΎΠ³ΠΎΠ΅: ΠΎΡ‚ старой-Π΄ΠΎΠ±Ρ€ΠΎΠΉ ΠΎΡ†Π΅Π½ΠΊΠΈ двиТСния оптичСским ΠΏΠΎΡ‚ΠΎΠΊΠΎΠΌ Π΄ΠΎ сСтСй-трансформСров.

Π•ΡΡ‚ΡŒ ΠΎΠ΄ΠΈΠ½ ΠΏΠΎΠ΄Ρ…ΠΎΠ΄ ΠΊ Ρ‚Ρ€Π΅ΠΊΠΈΠ½Π³Ρƒ, ΡˆΠΈΡ€ΠΎΠΊΠΎ извСстный Π½Π° Π·Π°ΠΏΠ°Π΄Π΅, Π½ΠΎ ΠΎ ΠΊΠΎΡ‚ΠΎΡ€ΠΎΠΌ ΠΌΠ°Π»ΠΎ ΠΏΠΈΡˆΡƒΡ‚ ΠΏΠΎ-русски: Incremental Visual Tracker (IVT). Π­Ρ‚ΠΎ Ρ‚Ρ€Π΅ΠΊΠ΅Ρ€ ΠΎΠ±ΡŠΠ΅ΠΊΡ‚ΠΎΠ² Π½Π° основС ΠΌΠΎΠ΄ΠΈΡ„ΠΈΡ†ΠΈΡ€ΠΎΠ²Π°Π½Π½ΠΎΠ³ΠΎ ΠΌΠ΅Ρ‚ΠΎΠ΄Π° Π³Π»Π°Π²Π½Ρ‹Ρ… ΠΊΠΎΠΌΠΏΠΎΠ½Π΅Π½Ρ‚: ΠΎΠ½ самообучаСтся Π½Π° Ρ…ΠΎΠ΄Ρƒ ΠΈ адаптируСтся ΠΊ ΠΈΠ·ΠΌΠ΅Π½Ρ‡ΠΈΠ²Ρ‹ΠΌ условиям.

Π”Π°Π²Π°ΠΉΡ‚Π΅ исслСдуСм Ρ„ΠΈΠ·ΠΈΠΎΠ»ΠΎΠ³ΠΈΡŽ этого Ρ‚Ρ€Π΅ΠΊΠ΅Ρ€Π°, Ρ‡Π΅ΠΌ ΠΎΠ½ интСрСсСн ΠΈ Π³Π΄Π΅ Π΅Π³ΠΎ ΠΌΠΎΠΆΠ½ΠΎ ΠΏΡ€ΠΈΠΌΠ΅Π½ΠΈΡ‚ΡŒ — Π° Π·Π°Ρ‚Π΅ΠΌ ΠΈΠ·ΡƒΡ‡ΠΈΠΌ ΠΏΡ€ΠΎΠ±Π»Π΅ΠΌΡ‹ Π΅Π³ΠΎ Ρ€Π΅Π°Π»ΠΈΠ·Π°Ρ†ΠΈΠΈ ΠΈ Π½ΡŽΠ°Π½ΡΡ‹ использования. Под ΠΊΠ°Ρ‚ΠΎΠΌ ссылка Π½Π° Ρ€Π΅ΠΏΠΎΠ·ΠΈΡ‚ΠΎΡ€ΠΈΠΉ ΠΈ ΠΌΠ½ΠΎΠ³ΠΎ ΠΌΠ°Ρ‚Π΅ΠΌΠ°Ρ‚ΠΈΠΊΠΈ.

ВсСм, ΠΊΡ‚ΠΎ интСрСсуСтся ΠΈΡΠΊΠ»ΡŽΡ‡ΠΈΡ‚Π΅Π»ΡŒΠ½ΠΎ Ρ€Π΅Π°Π»ΠΈΠ·Π°Ρ†ΠΈΠ΅ΠΉ, ΠΏΡ€Π΅Π΄ΡŠΡΠ²Π»ΡΡŽ C++-ΠΊΠΎΠ΄. Π•ΡΡ‚ΡŒ Ρ‚Π°ΠΊΠΆΠ΅ ΠΏΡ€ΠΎΡ‚ΠΎΡ‚ΠΈΠΏ Π½Π° Python. Π›ΠΈΡ†Π΅Π½Π·ΠΈΠΈ Π½Π΅Ρ‚Ρƒ, Π΄Π΅Π»Π°Ρ‚ΡŒ ΠΌΠΎΠΆΠ½ΠΎ Ρ‡Ρ‚ΠΎ ΡƒΠ³ΠΎΠ΄Π½ΠΎ.

О Ρ‚Ρ€Π΅ΠΊΠΈΠ½Π³Π΅ Π²ΠΊΡ€Π°Ρ‚Ρ†Π΅

Π’Ρ€Π΅ΠΊΠΈΠ½Π³ — это Π·Π°Π΄Π°Ρ‡Π° отслСТивания ΠΎΠ±ΡŠΠ΅ΠΊΡ‚Π° Π² сцСнС. Π Π΅ΡˆΠ°Π΅Ρ‚ΡΡ ΠΎΠ½Π° ΠΏΡƒΡ‚Π΅ΠΌ прСдсказания мСстополоТСния ΠΎΠ±ΡŠΠ΅ΠΊΡ‚Π° Π½Π° ΠΏΠΎΡΠ»Π΅Π΄ΡƒΡŽΡ‰Π΅ΠΌ ΠΊΠ°Π΄Ρ€Π΅ с ΡƒΡ‡Π΅Ρ‚ΠΎΠΌ Π΄ΠΈΠ½Π°ΠΌΠΈΠΊΠΈ Π΅Π³ΠΎ двиТСния. Π’Ρ€Π΅ΠΊΠΈΠ½Π³ примСняСтся Π²ΠΎ ΠΌΠ½ΠΎΠ³ΠΈΡ… Π·Π°Π΄Π°Ρ‡Π°Ρ… Π²ΠΈΠ΄Π΅ΠΎΠ°Π½Π°Π»ΠΈΡ‚ΠΈΠΊΠΈ срСди ΠΊΠΎΡ‚ΠΎΡ€Ρ‹Ρ… Π½Π°ΠΈΠ±ΠΎΠ»Π΅Π΅ Π²Ρ‹Π΄Π΅Π»ΡΡŽΡ‚ΡΡ подсчСт посСтитСлСй, Π°Π½Π°Π»ΠΈΠ· повСдСния (Π½Π°ΠΏΡ€ΠΈΠΌΠ΅Ρ€, ΠΆΠΈΠ²ΠΎΡ‚Π½Ρ‹Ρ… Π½Π° Ρ„Π΅Ρ€ΠΌΠ΅ ΠΈΠ»ΠΈ ΠΌΡ‹ΡˆΠ΅ΠΉ Π² Π»Π°Π±ΠΈΡ€ΠΈΠ½Ρ‚Π΅), Π°Π½Π°Π»ΠΈΠ· Ρ‚Ρ€Π°Π΅ΠΊΡ‚ΠΎΡ€ΠΈΠΈ Π°Π²Ρ‚ΠΎ для вычислСния срСднСй скорости, отслСТиваниС Π»ΠΈΡ†Π° для опрСдСлСния Π»ΡƒΡ‡ΡˆΠ΅Π³ΠΎ ΠΊΠ°Π΄Ρ€Π°, подходящСго для ΠΈΠ΄Π΅Π½Ρ‚ΠΈΡ„ΠΈΠΊΠ°Ρ†ΠΈΠΈ. ВсСго сцСнариСв использования, СстСствСнно, Π³ΠΎΡ€Π°Π·Π΄ΠΎ большС.

Для ΠΈΠ½Ρ‚Π΅Ρ€Π΅ΡΡƒΡŽΡ‰ΠΈΡ…ΡΡ Π΄Π°Π½Π½ΠΎΠΉ Ρ‚Π΅ΠΌΠΎΠΉ Ρ€Π΅ΠΊΠΎΠΌΠ΅Π½Π΄ΡƒΡŽ ΠΏΠΎΠ΄Ρ€ΠΎΠ±Π½ΡƒΡŽ ΡΡ‚Π°Ρ‚ΡŒΡŽ с ΠΏΡ€ΠΈΠΌΠ΅Ρ€Π°ΠΌΠΈ.

IVT Ρ‚Ρ€Π΅ΠΊΠ΅Ρ€ относится ΠΊ классу Ρ‚Π°ΠΊ Π½Π°Π·Ρ‹Π²Π°Π΅ΠΌΡ‹Ρ… appearance-based Ρ‚Ρ€Π΅ΠΊΠ΅Ρ€ΠΎΠ². ИдСя appearance-based ΠΏΠΎΠ΄Ρ…ΠΎΠ΄Π° состоит Π² Ρ‚ΠΎΠΌ, Ρ‡Ρ‚ΠΎΠ±Ρ‹ ΡΠΎΠ·Π΄Π°Ρ‚ΡŒ ΠΈΠ»ΠΈ ΠΎΠ±ΡƒΡ‡ΠΈΡ‚ΡŒ ΠΏΡ€ΠΈΠ·Π½Π°ΠΊΠΎΠ²ΠΎΠ΅ описаниС Ρ†Π΅Π»Π΅Π²ΠΎΠ³ΠΎ ΠΎΠ±ΡŠΠ΅ΠΊΡ‚Π° Π½Π° Π½Π°Ρ‡Π°Π»ΡŒΠ½ΠΎΠΌ ΠΊΠ°Π΄Ρ€Π΅ ΠΈ ΠΎΡ‚ΡΠ»Π΅ΠΆΠΈΠ²Π°Ρ‚ΡŒ Π΅Π³ΠΎ ΠΏΠ΅Ρ€Π΅ΠΌΠ΅Ρ‰Π΅Π½ΠΈΠ΅ с ΠΏΠΎΠΌΠΎΡ‰ΡŒΡŽ этого описания Π½Π° ΠΏΠΎΡΠ»Π΅Π΄ΡƒΡŽΡ‰ΠΈΡ… ΠΊΠ°Π΄Ρ€Π°Ρ…. НапримСр, ΠΌΡ‹ ΠΌΠΎΠΆΠ΅ΠΌ ΠΏΡ€Π΅Π΄ΡΡ‚Π°Π²ΠΈΡ‚ΡŒ ΠΎΠ±ΡŠΠ΅ΠΊΡ‚ Ρ†Π²Π΅Ρ‚ΠΎΠ²ΠΎΠΉ гистограммой ΠΈ Π² дальнСйшСм ΠΈΡΠΊΠ°Ρ‚ΡŒ Ρ€Π΅Π³ΠΈΠΎΠ½ с Π½Π°ΠΈΠ±ΠΎΠ»Π΅Π΅ ΠΏΠΎΡ…ΠΎΠΆΠΈΠΌ Ρ†Π²Π΅Ρ‚ΠΎΠ²Ρ‹ΠΌ распрСдСлСниСм. Или ΠΎΠ±ΡƒΡ‡ΠΈΡ‚ΡŒ Π½Π΅ΠΉΡ€ΠΎΡΠ΅Ρ‚ΡŒ ΠΏΡ€Π΅Π΄ΡΡ‚Π°Π²Π»ΡΡ‚ΡŒ ΠΎΠ±ΡŠΠ΅ΠΊΡ‚ Π²Π΅ΠΊΡ‚ΠΎΡ€ΠΎΠΌ эмбСддингов Π² Π΅Π²ΠΊΠ»ΠΈΠ΄ΠΎΠ²ΠΎΠΌ пространствС с Ρ‚Π΅ΠΌ свойством, Ρ‡Ρ‚ΠΎ ΠΏΠΎΡ…ΠΎΠΆΠΈΠ΅ ΠΎΠ±ΡŠΠ΅ΠΊΡ‚Ρ‹ Π² этом пространствС Π±ΡƒΠ΄ΡƒΡ‚ Π½Π°Ρ…ΠΎΠ΄ΠΈΡ‚ΡŒΡΡ рядом, Π² Ρ‚ΠΎ врСмя ΠΊΠ°ΠΊ Π½Π΅ΠΏΠΎΡ…ΠΎΠΆΠΈΠ΅ ΠΎΠ±ΡŠΠ΅ΠΊΡ‚Ρ‹ Π΄Π°Π»Π΅ΠΊΠΎ. Π­Ρ‚ΠΎ Π±Ρ‹Π» Π½Π°ΠΌΠ΅ΠΊ Π½Π° DeepSORT. ΠΠ»ΡŒΡ‚Π΅Ρ€Π½Π°Ρ‚ΠΈΠ²Π½Ρ‹ΠΉ ΠΏΠΎΠ΄Ρ…ΠΎΠ΄ ΠΊ Ρ‚Ρ€Π΅ΠΊΠΈΠ½Π³Ρƒ, Π½Π΅Ρ„ΠΎΡ€ΠΌΠ°Π»ΡŒΠ½ΠΎ — detection-based, Π½Π΅ ΠΏΡ€Π΅Π΄ΠΏΠΎΠ»Π°Π³Π°Π΅Ρ‚ использования β€œΠ²Π½Π΅ΡˆΠ½Π΅Π³ΠΎ вида” ΠΎΠ±ΡŠΠ΅ΠΊΡ‚Π° ΠΈ основываСтся ΠΈΡΠΊΠ»ΡŽΡ‡ΠΈΡ‚Π΅Π»ΡŒΠ½ΠΎ Π½Π° отслСТивании Π΅Π³ΠΎ ΠΊΠΎΠΎΡ€Π΄ΠΈΠ½Π°Ρ‚. ΠŸΡ€Π΅ΠΈΠΌΡƒΡ‰Π΅ΡΡ‚Π²ΠΎ ΠΆΠ΅ кодирования β€œΠ²Π½Π΅ΡˆΠ½Π΅Π³ΠΎ вида” ΠΎΠ±ΡŠΠ΅ΠΊΡ‚Π° Π·Π°ΠΊΠ»ΡŽΡ‡Π°Π΅Ρ‚ΡΡ Π² Ρ‚ΠΎΠΌ, Ρ‡Ρ‚ΠΎ эта информация Π²ΠΊΡƒΠΏΠ΅ с ΠΊΠΎΠΎΡ€Π΄ΠΈΠ½Π°Ρ‚Π°ΠΌΠΈ ΠΎΠ±ΡŠΠ΅ΠΊΡ‚Π° ΡƒΠ»ΡƒΡ‡ΡˆΠ°Π΅Ρ‚ качСство Ρ‚Ρ€Π΅ΠΊΠΈΠ½Π³Π°. Π’ΠΈΠ·ΡƒΠ°Π»ΡŒΠ½ΠΎ Ρ€Π°Π·Π½ΠΈΡ†Ρƒ ΠΌΠ΅ΠΆΠ΄Ρƒ detection-модСлью ΠΈ appearance-модСлью ΠΌΠΎΠΆΠ½ΠΎ ΡƒΠ²ΠΈΠ΄Π΅Ρ‚ΡŒ Π½Π° Π²ΠΈΠ΄Π΅ΠΎ Π½ΠΈΠΆΠ΅.

Π’ΠΈΠ΄Π΅ΠΎ 1. На Π²ΠΈΠ΄Π΅ΠΎ слСва ΠΏΡ€ΠΈΠΌΠ΅Ρ€ Ρ€Π°Π±ΠΎΡ‚Ρ‹ Ρ‚Ρ€Π΅ΠΊΠ΅Ρ€Π° SORT: зСлСная Ρ€Π°ΠΌΠΊΠ° соотвСтствуСт Π΄Π΅Ρ‚Π΅ΠΊΡ‚ΠΎΡ€Ρƒ YOLO, ΠΊΠΎΡ‚ΠΎΡ€Ρ‹ΠΉ Π²Ρ‹Π΄Π°Π΅Ρ‚ сработку Ρ€Π°Π· Π² 3 ΠΊΠ°Π΄Ρ€Π°, красная Ρ€Π°ΠΌΠΊΠ° соотвСтствуСт ΠΎΡ†Π΅Π½ΠΊΠ΅ Ρ„ΠΈΠ»ΡŒΡ‚Ρ€ΠΎΠΌ Калмана ΠΌΠ΅ΠΆΠ΄Ρƒ дСтСкциями. На ΠΏΡ€Π°Π²ΠΎΠΌ Π²ΠΈΠ΄Π΅ΠΎ ΠΏΡ€ΠΈΠΌΠ΅Ρ€ Ρ‚Ρ€Π΅ΠΊΠ΅Ρ€Π° IVT: красная Ρ€Π°ΠΌΠΊΠ° соотвСтствуСт ΠΎΡ†Π΅Π½ΠΊΠ΅ Π½Π° основС appearance-ΠΌΠΎΠ΄Π΅Π»ΠΈ, Π±Π΅Π· всякой Π΄Π΅Ρ‚Π΅ΠΊΡ†ΠΈΠΈ.

Π”Π°Π»Π΅Π΅ ΠΌΡ‹ рассмотрим всС ΡΠΎΡΡ‚Π°Π²Π»ΡΡŽΡ‰ΠΈΠ΅ IVT Ρ‚Ρ€Π΅ΠΊΠ΅Ρ€Π°, спСрва ΠΏΠΎ ΠΎΡ‚Π΄Π΅Π»ΡŒΠ½ΠΎΡΡ‚ΠΈ, Π·Π°Ρ‚Π΅ΠΌ соСдинив всС вмСстС.

ΠšΠΎΠΌΠΏΠ°ΠΊΡ‚Π½ΠΎΠ΅ прСдставлСниС ΠΎΠ±ΡŠΠ΅ΠΊΡ‚Π°

ВмСсто Ρ‚ΠΎΠ³ΠΎ, Ρ‡Ρ‚ΠΎΠ±Ρ‹ Ρ€Π°Π±ΠΎΡ‚Π°Ρ‚ΡŒ с ΠΈΠ·ΠΎΠ±Ρ€Π°ΠΆΠ΅Π½ΠΈΠ΅ΠΌ ΠΎΠ±ΡŠΠ΅ΠΊΡ‚Π° Π½Π°ΠΏΡ€ΡΠΌΡƒΡŽ, ΠΊΠ°ΠΊ с Π½Π°Π±ΠΎΡ€ΠΎΠΌ нСзависимых пиксСлСй, Ρ€Π°Π·ΡƒΠΌΠ½Π΅Π΅ ΠΎΠΏΠΈΡΠ°Ρ‚ΡŒ Π΅Π³ΠΎ Π½Π΅ΠΊΠΈΠΌ Π½Π°Π±ΠΎΡ€ΠΎΠΌ ΠΏΡ€ΠΈΠ·Π½Π°ΠΊΠΎΠ², Ρ…Π°Ρ€Π°ΠΊΡ‚Π΅Ρ€ΠΈΠ·ΡƒΡŽΡ‰ΠΈΡ… этот ΠΎΠ±ΡŠΠ΅ΠΊΡ‚. Π’Π°ΠΊ ΠΌΡ‹ ΡƒΠΌΠ΅Π½ΡŒΡˆΠΈΠΌ Ρ€Π°Π·ΠΌΠ΅Ρ€Π½ΠΎΡΡ‚ΡŒ оставив Ρ‚ΠΎΠ»ΡŒΠΊΠΎ ΡΠ°ΠΌΡƒΡŽ Ρ€Π΅Π»Π΅Π²Π°Π½Ρ‚Π½ΡƒΡŽ ΠΈΠ½Ρ„ΠΎΡ€ΠΌΠ°Ρ†ΠΈΡŽ ΠΎΠ± ΠΎΠ±ΡŠΠ΅ΠΊΡ‚Π΅. Π’ качСствС ΠΏΡ€ΠΈΠ·Π½Π°ΠΊΠΎΠ²ΠΎΠ³ΠΎ описания ΠΎΠ±ΡŠΠ΅ΠΊΡ‚Π° ΠΌΠΎΠΆΠ΅Ρ‚ Π±Ρ‹Ρ‚ΡŒ использовано Π½Π΅ Ρ‚ΠΎΠ»ΡŒΠΊΠΎ Ρ†Π²Π΅Ρ‚ΠΎΠ²ΠΎΠ΅ распрСдСлСниС, ΠΏΡ€ΠΈΠΌΠ΅Ρ€ ΠΊΠΎΡ‚ΠΎΡ€ΠΎΠ³ΠΎ Π±Ρ‹Π» ΠΏΡ€ΠΈΠ²Π΅Π΄Π΅Π½ Π²Ρ‹ΡˆΠ΅, Π½ΠΎ ΠΈ Π΅Π³ΠΎ ΠΊΠΎΠ½Ρ‚ΡƒΡ€, тСкстура ΠΈΠ»ΠΈ Π±ΠΎΠ»Π΅Π΅ абстрактныС ΠΏΡ€ΠΈΠ·Π½Π°ΠΊΠΈ, Ρ‚Π°ΠΊΠΈΠ΅ ΠΊΠ°ΠΊ Π²Π΅ΠΊΡ‚ΠΎΡ€ эмбСддингов ΠΈΠ»ΠΈ базис Π³Π»Π°Π²Π½Ρ‹Ρ… ΠΊΠΎΠΌΠΏΠΎΠ½Π΅Π½Ρ‚.

ΠœΡ‹ Π±ΡƒΠ΄Π΅ΠΌ ΠΏΡ€Π΅Π΄ΡΡ‚Π°Π²Π»ΡΡ‚ΡŒ ΠΎΠ±ΡŠΠ΅ΠΊΡ‚ Π² (ΠΏΠΎΠ΄)пространствС ΠΌΠ°Π»ΠΎΠΉ размСрности — собствСнном базисС. Он ΠΆΠ΅ базис Π³Π»Π°Π²Π½Ρ‹Ρ… ΠΊΠΎΠΌΠΏΠΎΠ½Π΅Π½Ρ‚, ΠΎΠ½ ΠΆΠ΅ eigenbasis. Π­Ρ‚ΠΎΡ‚ базис содСрТит Π±Γ³Π»ΡŒΡˆΡƒΡŽ Ρ‡Π°ΡΡ‚ΡŒ всСй Ρ€Π΅Π»Π΅Π²Π°Π½Ρ‚Π½ΠΎΠΉ ΠΈΠ½Ρ„ΠΎΡ€ΠΌΠ°Ρ†ΠΈΠΈ ΠΎΠ± ΠΎΠ±ΡŠΠ΅ΠΊΡ‚Π΅ нСсмотря Π½Π° Ρ‚ΠΎ, Ρ‡Ρ‚ΠΎ Π΅Π³ΠΎ Ρ€Π°Π·ΠΌΠ΅Ρ€Π½ΠΎΡΡ‚ΡŒ Π³ΠΎΡ€Π°Π·Π΄ΠΎ Π½ΠΈΠΆΠ΅ исходной. ΠœΠΎΠ΄Π΅Π»ΠΈΡ€ΠΎΠ²Π°Ρ‚ΡŒ этот базис ΠΌΡ‹ Π±ΡƒΠ΄Π΅ΠΌ с ΠΏΠΎΠΌΠΎΡ‰ΡŒΡŽ ΠΌΠ΅Ρ‚ΠΎΠ΄Π° Π³Π»Π°Π²Π½Ρ‹Ρ… ΠΊΠΎΠΌΠΏΠΎΠ½Π΅Π½Ρ‚ (principal component analysis) ΠΈΡΠΏΠΎΠ»ΡŒΠ·ΡƒΡ ΠΏΠ΅Ρ€Π²Ρ‹Π΅ ΠΈΠ·ΠΎΠ±Ρ€Π°ΠΆΠ΅Π½ΠΈΠΉ ΠΎΠ±ΡŠΠ΅ΠΊΡ‚Π° Π½Π° Π½Π°Ρ‡Π°Π»ΡŒΠ½Ρ‹Ρ… ΠΊΠ°Π΄Ρ€Π°Ρ….

ΠšΡ€Π°Ρ‚ΠΊΠΎ напомню, Ρ‡Ρ‚ΠΎ ΡΡƒΡ‚ΡŒ ΠΌΠ΅Ρ‚ΠΎΠ΄Π° Π³Π»Π°Π²Π½Ρ‹Ρ… ΠΊΠΎΠΌΠΏΠΎΠ½Π΅Π½Ρ‚ состоит Π² Ρ‚ΠΎΠΌ, Ρ‡Ρ‚ΠΎΠ±Ρ‹ Π½Π°ΠΉΡ‚ΠΈ для исходных Π΄Π°Π½Π½Ρ‹Ρ… Ρ‚Π°ΠΊΡƒΡŽ систСму ΠΊΠΎΠΎΡ€Π΄ΠΈΠ½Π°Ρ‚Π½Ρ‹Ρ… осСй (Π³Π»Π°Π²Π½Ρ‹Ρ… ΠΊΠΎΠΌΠΏΠΎΠ½Π΅Π½Ρ‚), которая Π΄Π°Π²Π°Π»Π° Π±Ρ‹ Π½Π°ΠΈΠ±ΠΎΠ»ΡŒΡˆΡƒΡŽ Π΄ΠΈΡΠΏΠ΅Ρ€ΡΠΈΡŽ расстояний ΠΌΠ΅ΠΆΠ΄Ρƒ проСкциями Π½Π° эту систСму. ΠŸΡ€ΠΎΠ΅Ρ†ΠΈΡ€ΡƒΡ исходный Π²Π΅ΠΊΡ‚ΠΎΡ€ Π΄Π°Π½Π½Ρ‹Ρ… Π½Π° Π½ΠΎΠ²ΡƒΡŽ систСму ΠΌΡ‹ ΠΏΠΎΠ»ΡƒΡ‡Π°Π΅ΠΌ Π½ΠΎΠ²Ρ‹ΠΉ Π²Π΅ΠΊΡ‚ΠΎΡ€ , ΠΊΠΎΡ‚ΠΎΡ€Ρ‹ΠΉ ΠΌΡ‹ ΠΌΠΎΠΆΠ΅ΠΌ ΡƒΡ€Π΅Π·Π°Ρ‚ΡŒ Π²ΠΏΠ»ΠΎΡ‚ΡŒ Π΄ΠΎ , Π³Π΄Π΅ ΠΎΡΡ‚Π°Π²ΡˆΠΈΡ…ΡΡ Π·Π½Π°Ρ‡Π΅Π½ΠΈΠΉ ΡΠΎΠΎΡ‚Π²Π΅Ρ‚ΡΡ‚Π²ΡƒΡŽΡ‚ осям наибольшСй диспСрсии.

На рисунках Π½ΠΈΠΆΠ΅ ΠΏΡ€ΠΈΠ²Π΅Π΄Π΅Π½Ρ‹ ΠΏΡ€ΠΈΠΌΠ΅Ρ€Ρ‹ ΡƒΠΌΠ΅Π½ΡŒΡˆΠ΅Π½ΠΈΡ размСрности.

Рис. 1. Визуализация ΡƒΠΌΠ΅Π½ΡŒΡˆΠ΅Π½ΠΈΡ размСрности.

Рис. 1. Визуализация ΡƒΠΌΠ΅Π½ΡŒΡˆΠ΅Π½ΠΈΡ размСрности Π² Π΄Π²ΡƒΠΌΠ΅Ρ€Π½ΠΎΠΌ пространствС. Π˜ΡΡ…ΠΎΠ΄Π½Ρ‹Π΅ Π΄Π°Π½Π½Ρ‹Π΅ ΠΏΡ€Π΅Π΄ΡΡ‚Π°Π²Π»ΡΡŽΡ‚ собой массив Π²Π΅ΠΊΡ‚ΠΎΡ€ΠΎΠ² , ΠΊΠ°ΠΆΠ΄Ρ‹ΠΉ ΠΈΠ· ΠΊΠΎΡ‚ΠΎΡ€Ρ‹Ρ… изобраТаСтся Ρ‚ΠΎΡ‡ΠΊΠΎΠΉ Π½Π° Π³Ρ€Π°Ρ„ΠΈΠΊΠ΅ с ΠΊΠΎΠΎΡ€Π΄ΠΈΠ½Π°Ρ‚Π°ΠΌΠΈ . Рисунок слСва ΠΈΠ»Π»ΡŽΡΡ‚Ρ€ΠΈΡ€ΡƒΠ΅Ρ‚ Π½ΠΎΠ²ΡƒΡŽ ΠΎΡ€Ρ‚ΠΎΠ³ΠΎΠ½Π°Π»ΡŒΠ½ΡƒΡŽ систСму ΠΊΠΎΠΎΡ€Π΄ΠΈΠ½Π°Ρ‚ с Ρ†Π΅Π½Ρ‚Ρ€ΠΎΠΌ Π² . Рисунок справа ΠΈΠ»Π»ΡŽΡΡ‚Ρ€ΠΈΡ€ΡƒΠ΅Ρ‚ ΠΏΡ€ΠΎΠ΅ΠΊΡ†ΠΈΡŽ исходных Π΄Π°Π½Π½Ρ‹Ρ… (красныС Ρ‚ΠΎΡ‡ΠΊΠΈ) Π½Π° систСму, ΡΠΎΡΡ‚ΠΎΡΡ‰ΡƒΡŽ Ρ‚ΠΎΠ»ΡŒΠΊΠΎ ΠΈΠ· ΠΏΠ΅Ρ€Π²ΠΎΠΉ Π³Π»Π°Π²Π½ΠΎΠΉ ΠΊΠΎΠΌΠΏΠΎΠ½Π΅Π½Ρ‚Ρ‹ . Π’Π°ΠΊ ΠΊΠ°ΠΊ разброс ΠΏΡ€ΠΎΠ΅ΠΊΡ†ΠΈΠΉ для оси Π·Π°ΠΌΠ΅Ρ‚Π½ΠΎ мСньшС разброса оси , ΠΌΡ‹ ΠΌΠΎΠΆΠ΅ΠΌ ΠΎΡ‚ΡΠ΅Ρ‡ΡŒ Π²Ρ‚ΠΎΡ€ΡƒΡŽ ось, ΠΏΠΎΠΆΠ΅Ρ€Ρ‚Π²ΠΎΠ²Π°Ρ‚ΡŒ Π½Π΅ΠΊΠΎΡ‚ΠΎΡ€ΠΎΠΉ ΠΈΠ½Ρ„ΠΎΡ€ΠΌΠ°Ρ†ΠΈΠ΅ΠΉ, сохранив ΠΏΡ€ΠΈ этом ΠΎΡΠ½ΠΎΠ²Π½ΡƒΡŽ. Π’Π°ΠΊΠΈΠΌ ΠΎΠ±Ρ€Π°Π·ΠΎΠΌ проСкция прСдставляСтся ΠΎΠ΄Π½ΠΈΠΌ числом ΠΈ ΠΌΡ‹ сократили Ρ€Π°Π·ΠΌΠ΅Ρ€Π½ΠΎΡΡ‚ΡŒ с 2 Π΄ΠΎ 1.

Рис. 2. ΠšΡ€Π°ΡΠΈΠ²Π°Ρ анимация поиска оси с наибольшСй диспСрсиСй. Π˜ΡΡ‚ΠΎΡ‡Π½ΠΈΠΊ https://builtin.com/data-science/step-step-explanation-principal-component-analysis.

Π’Π΅ΡΡŒ Π°ΠΏΠΏΠ°Ρ€Π°Ρ‚, ΠΏΡ€ΠΎΠΈΠ»Π»ΡŽΡΡ‚Ρ€ΠΈΡ€ΠΎΠ²Π°Π½Π½Ρ‹ΠΉ Π½Π° рисунках 1 ΠΈ 2 для Π΄Π²ΡƒΠΌΠ΅Ρ€Π½ΠΎΠ³ΠΎ случая Π±ΡƒΠ΄Π΅Ρ‚ Ρ‚Π°ΠΊΠΆΠ΅ справСдлив для пространства любой размСрности. Π’Π°ΠΊ ΠΊΠ°ΠΊ исходными Π΄Π°Π½Π½Ρ‹ΠΌΠΈ Π² нашСм случаС являСтся одноканальноС ΠΈΠ·ΠΎΠ±Ρ€Π°ΠΆΠ΅Π½ΠΈΠ΅ ΠΎΠ±ΡŠΠ΅ΠΊΡ‚Π° ΠΈΠ»ΠΈ , Π²Ρ‹Ρ€Π΅Π·Π°Π½Π½ΠΎΠ΅ ΠΈΠ· ΠΎΡ€ΠΈΠ³ΠΈΠ½Π°Π»ΡŒΠ½ΠΎΠ³ΠΎ ΠΊΠ°Π΄Ρ€Π°, Ρ‚ΠΎ ΠΌΡ‹ Π±ΡƒΠ΄Π΅ΠΌ ΡΡ‡ΠΈΡ‚Π°Ρ‚ΡŒ, Ρ‡Ρ‚ΠΎ проСкция Π½Π° базис , Π³Π΄Π΅ , прСдставляСт собой Π²Π΅ΠΊΡ‚ΠΎΡ€ ΠΏΡ€ΠΈΠ·Π½Π°ΠΊΠΎΠ² этого ΠΎΠ±ΡŠΠ΅ΠΊΡ‚Π°. Π’ ΠΏΡ€ΠΎΠ³Ρ€Π°ΠΌΠΌΠ½ΠΎΠΉ Ρ€Π΅Π°Π»ΠΈΠ·Π°Ρ†ΠΈΠΈ, ΠΎ ΠΊΠΎΡ‚ΠΎΡ€ΠΎΠΉ Π±ΡƒΠ΄Π΅Ρ‚ рассказано Π½ΠΈΠΆΠ΅, Π±ΡƒΠ΄ΡƒΡ‚ ΠΈΡΠΏΠΎΠ»ΡŒΠ·ΠΎΠ²Π°Π½Ρ‹ ΠΏΠ΅Ρ€Π²Ρ‹Π΅ 16 Π³Π»Π°Π²Π½Ρ‹Ρ… ΠΊΠΎΠΌΠΏΠΎΠ½Π΅Π½Ρ‚ ΠΏΡ€ΠΈ исходной Π΄Π»ΠΈΠ½Π΅ Π²Π΅ΠΊΡ‚ΠΎΡ€Π° 32×32=1024. Богласимся, Ρ‡Ρ‚ΠΎ ΠΎΠΏΠ΅Ρ€ΠΈΡ€ΠΎΠ²Π°Ρ‚ΡŒ ΠΌΠ°Ρ‚Ρ€ΠΈΡ†Π°ΠΌΠΈ Π³ΠΎΡ€Π°Π·Π΄ΠΎ Π²Ρ‹Π³ΠΎΠ΄Π½Π΅Π΅ Ρ‡Π΅ΠΌ ΠΌΠ°Ρ‚Ρ€ΠΈΡ†Π°ΠΌΠΈ . На рисункС Π½ΠΈΠΆΠ΅ Π²ΠΈΠ·ΡƒΠ°Π»ΡŒΠ½ΠΎ прСдставлСны ΠΏΠ΅Ρ€Π²Ρ‹Π΅ Ρ‡Π΅Ρ‚Ρ‹Ρ€Π΅ Π³Π»Π°Π²Π½Ρ‹Π΅ ΠΊΠΎΠΌΠΏΠΎΠ½Π΅Π½Ρ‚Ρ‹ для ΠΈΠ·ΠΎΠ±Ρ€Π°ΠΆΠ΅Π½ΠΈΠΉ вСлосипСда.

Рис. 3. Визуализация ΠΏΠ΅Ρ€Π²Ρ‹Ρ… Π³Π»Π°Π²Π½Ρ‹Ρ… ΠΊΠΎΠΌΠΏΠΎΠ½Π΅Π½Ρ‚ для пятидСсяти ΠΈΠ·ΠΎΠ±Ρ€Π°ΠΆΠ΅Π½ΠΈΠΉ вСлосипСда. Π‘Π»Π΅Π²Π°-Π½Π°ΠΏΡ€Π°Π²ΠΎ: ΠΎΡ€ΠΈΠ³ΠΈΠ½Π°Π»ΡŒΠ½Ρ‹ΠΉ ΠΊΠ°Π΄Ρ€, срСднСС ΠΏΠΎ послСдним пятидСсяти ΠΊΠ°Π΄Ρ€Π°ΠΌ, Π³Π»Π°Π²Π½Ρ‹Π΅ ΠΊΠΎΠΌΠΏΠΎΠ½Π΅Π½Ρ‚Ρ‹ с ΠΏΠ΅Ρ€Π²ΠΎΠΉ ΠΏΠΎ Ρ‡Π΅Ρ‚Π²Π΅Ρ€Ρ‚ΡƒΡŽ.

Π Π°Π½Π΅Π΅ ΠΌΡ‹ Π³ΠΎΠ²ΠΎΡ€ΠΈΠ»ΠΈ, Ρ‡Ρ‚ΠΎ appearance-based ΠΏΠΎΠ΄Ρ…ΠΎΠ΄ ΠΏΡ€Π΅Π΄ΠΏΠΎΠ»Π°Π³Π°Π΅Ρ‚ отслСТиваниС ΠΎΠ±ΡŠΠ΅ΠΊΡ‚Π° Π½Π° ΠΏΠΎΡΠ»Π΅Π΄ΡƒΡŽΡ‰ΠΈΡ… ΠΊΠ°Π΄Ρ€Π°Ρ… с ΠΏΠΎΠΌΠΎΡ‰ΡŒΡŽ ΠΏΡ€ΠΈΠ·Π½Π°ΠΊΠΎΠ²ΠΎΠ³ΠΎ описания ΠΎΠ±ΡŠΠ΅ΠΊΡ‚Π°. Каким ΠΆΠ΅ ΠΎΠ±Ρ€Π°Π·ΠΎΠΌ ΠΏΡ€ΠΈΠ·Π½Π°ΠΊΠΎΠ²ΠΎΠ΅ описаниС Π² пространствС Π³Π»Π°Π²Π½Ρ‹Ρ… ΠΊΠΎΠΌΠΏΠΎΠ½Π΅Π½Ρ‚ ΠΌΠΎΠΆΠ΅Ρ‚ Π±Ρ‹Ρ‚ΡŒ использовано Π½Π°ΠΌΠΈ для отслСТивания ΠΎΠ±ΡŠΠ΅ΠΊΡ‚Π°? ΠœΠ΅Ρ…Π°Π½ΠΈΠ·ΠΌ для этого прост — ΠΌΡ‹ Π΄ΠΎΠ»ΠΆΠ½Ρ‹ Π½Π°ΠΉΡ‚ΠΈ участок ΠΊΠ°Π΄Ρ€Π° , ΠΊΠΎΡ‚ΠΎΡ€Ρ‹ΠΉ Π»ΡƒΡ‡ΡˆΠ΅ всСго проСцируСтся Π½Π° базис . Π§Ρ‚ΠΎ Π·Π½Π°Ρ‡ΠΈΡ‚ β€œΠ»ΡƒΡ‡ΡˆΠ΅ всСго”?

Если Ρ€Π°ΡΡΠΌΠ°Ρ‚Ρ€ΠΈΠ²Π°Ρ‚ΡŒ собствСнный базис Ρ‡Π΅Ρ€Π΅Π· вСроятностный ΠΏΠΎΠ΄Ρ…ΠΎΠ΄, Ρ‚ΠΎ ΠΌΡ‹ ΠΌΠΎΠΆΠ΅ΠΌ ΠΎΠΏΡ€Π΅Π΄Π΅Π»ΠΈΡ‚ΡŒ Π²Π΅Ρ€ΠΎΡΡ‚Π½ΠΎΡΡ‚ΡŒ Ρ‚ΠΎΠ³ΠΎ, Ρ‡Ρ‚ΠΎ ΠΎΠ±ΡŠΠ΅ΠΊΡ‚ ΠΏΡ€ΠΈΠ½Π°Π΄Π»Π΅ΠΆΠΈΡ‚ Π΄Π°Π½Π½ΠΎΠΌΡƒ базису. Π­Ρ‚Π° Π²Π΅Ρ€ΠΎΡΡ‚Π½ΠΎΡΡ‚ΡŒ ΠΎΠ±Ρ€Π°Ρ‚Π½ΠΎ ΠΏΡ€ΠΎΠΏΠΎΡ€Ρ†ΠΈΠΎΠ½Π°Π»ΡŒΠ½Π° Ρ€Π°ΡΡΡ‚ΠΎΡΠ½ΠΈΡŽ ΠΎΡ‚ спроСцированного ΠΎΠ±ΡŠΠ΅ΠΊΡ‚Π° Π΄ΠΎ Ρ†Π΅Π½Ρ‚Ρ€Π° базиса . Π’ ΡΠ»Π΅Π΄ΡƒΡŽΡ‰ΠΈΡ… Ρ€Π°Π·Π΄Π΅Π»Π°Ρ… Π±ΡƒΠ΄Π΅Ρ‚ ΠΏΠΎΠΊΠ°Π·Π°Π½ΠΎ, Ρ‡Ρ‚ΠΎ всС Ρ‡ΡƒΡ‚ΡŒ слоТнСС, Π½ΠΎ для Ρ‚Π΅ΠΊΡƒΡ‰Π΅Π³ΠΎ пояснСния этого Π±ΡƒΠ΄Π΅Ρ‚ достаточно. ΠžΡ‡Π΅Π²ΠΈΠ΄Π½ΠΎ, Ρ‡Ρ‚ΠΎ ΠΎΠ±ΡŠΠ΅ΠΊΡ‚Ρ‹, ΠΏΡ€ΠΈΠ½Π°Π΄Π»Π΅ΠΆΠ°Ρ‰ΠΈΠ΅ базису Π±ΡƒΠ΄ΡƒΡ‚ Ρ€Π°ΡΠΏΠΎΠ»Π°Π³Π°Ρ‚ΡŒΡΡ Π±Π»ΠΈΠ·ΠΊΠΎ ΠΊ Ρ†Π΅Π½Ρ‚Ρ€Ρƒ, Π² Ρ‚ΠΎ врСмя ΠΊΠ°ΠΊ ΠΈΠ½Ρ‹Π΅ ΠΎΠ±ΡŠΠ΅ΠΊΡ‚Ρ‹ Π±ΡƒΠ΄ΡƒΡ‚ Π½Π°Ρ…ΠΎΠ΄ΠΈΡ‚ΡŒΡΡ Π΄Π°Π»Π΅ΠΊΠΎ. ΠŸΡ€Π΅Π΄ΡΡ‚Π°Π²ΡŒΡ‚Π΅, Ρ‡Ρ‚ΠΎ Π½Π° рисункС 1 ΠΌΡ‹ пытаСмся ΡΠΏΡ€ΠΎΠ΅Ρ†ΠΈΡ€ΠΎΠ²Π°Ρ‚ΡŒ Ρ‚ΠΎΡ‡ΠΊΡƒ Π½Π° Π³Π»Π°Π²Π½ΡƒΡŽ ось . ΠŸΠΎΠ½ΡΡ‚Π½ΠΎ, Ρ‡Ρ‚ΠΎ проСкция Π±ΡƒΠ΄Π΅Ρ‚ Π½Π°Ρ…ΠΎΠ΄ΠΈΡ‚ΡŒΡΡ Π΄Π°Π»Π΅ΠΊΠΎ ΠΎΡ‚ Ρ†Π΅Π½Ρ‚Ρ€Π° , Π·Π½Π°Ρ‡ΠΈΡ‚ маловСроятно, Ρ‡Ρ‚ΠΎ Ρ‚ΠΎΡ‡ΠΊΠ° ΠΏΡ€ΠΈΠ½Π°Π΄Π»Π΅ΠΆΠΈΡ‚ базису. Π’Π΅ΠΏΠ΅Ρ€ΡŒ ΠΏΡ€Π΅Π΄ΠΏΠΎΠ»ΠΎΠΆΠΈΠΌ, Ρ‡Ρ‚ΠΎ ΠΌΡ‹ Ρ…ΠΎΡ‚ΠΈΠΌ Π½Π°ΠΉΡ‚ΠΈ участок ΠΊΠ°Π΄Ρ€Π°, Π² ΠΊΠΎΡ‚ΠΎΡ€ΠΎΠΌ находится ΠΎΠ±ΡŠΠ΅ΠΊΡ‚. Для этого я Π΄ΠΎΠ»ΠΆΠ΅Π½ Π½Π°ΠΉΡ‚ΠΈ участок ΠΊΠ°Π΄Ρ€Π°, ΠΊΠΎΡ‚ΠΎΡ€Ρ‹ΠΉ Π»ΡƒΡ‡ΡˆΠ΅ всСго проСцируСтся Π½Π° базис ΠΎΠ±ΡŠΠ΅ΠΊΡ‚Π° ; ΠΈΠ½Ρ‹ΠΌΠΈ словами, расстояниС ΠΎΡ‚ ΠΊΠΎΡ‚ΠΎΡ€ΠΎΠ³ΠΎ Π΄ΠΎ базиса минимально.

Π§Ρ‚ΠΎΠ±Ρ‹ Π»ΡƒΡ‡ΡˆΠ΅ ΠΏΠΎΠ½ΡΡ‚ΡŒ, ΠΊΠ°ΠΊ это Π±ΡƒΠ΄Π΅Ρ‚ Π²Ρ‹Π³Π»ΡΠ΄Π΅Ρ‚ΡŒ, ΠΏΠΎΠΏΡ€ΠΎΠ±ΡƒΠ΅ΠΌ ΠΏΡ€Π΅Π΄ΡΡ‚Π°Π²ΠΈΡ‚ΡŒ ΠΏΡ€ΠΎΠ΅ΠΊΡ†ΠΈΡŽ Π½Π° базис Π² качСствС коррСляционной Ρ„ΡƒΠ½ΠΊΡ†ΠΈΠΈ , ΠΏΡ€ΠΈΠ½ΠΈΠΌΠ°ΡŽΡ‰ΡƒΡŽ Π½Π° Π²Ρ…ΠΎΠ΄ участок изобраТСния, ΠΎΠΊΠ½ΠΎ, фиксированного Ρ€Π°Π·ΠΌΠ΅Ρ€Π° ΠΈ Π²ΠΎΠ·Π²Ρ€Π°Ρ‰Π°ΡŽΡ‰ΡƒΡŽ ΡΡ‚Π΅ΠΏΠ΅Π½ΡŒ принадлСТности этого участка базису . Π’Π΅ΠΏΠ΅Ρ€ΡŒ заставим Ρ„ΡƒΠ½ΠΊΡ†ΠΈΡŽ ΠΏΡ€ΠΎΠ±Π΅ΠΆΠ°Ρ‚ΡŒ всС ΠΈΠ·ΠΎΠ±Ρ€Π°ΠΆΠ΅Π½ΠΈΠ΅, сдвигая нашС ΠΎΠΊΠ½ΠΎ ΠΏΠΎ Π³ΠΎΡ€ΠΈΠ·ΠΎΠ½Ρ‚Π°Π»ΠΈ ΠΈ Π²Π΅Ρ€Ρ‚ΠΈΠΊΠ°Π»ΠΈ. Π’ΠΎΠ³Π΄Π° Ρ€Π΅Π·ΡƒΠ»ΡŒΡ‚Π°Ρ‚ΠΎΠΌ Ρ€Π°Π±ΠΎΡ‚Ρ‹ Π±ΡƒΠ΄Π΅Ρ‚ Π½ΠΎΠ²ΠΎΠ΅ ΠΈΠ·ΠΎΠ±Ρ€Π°ΠΆΠ΅Π½ΠΈΠ΅, Π² ΠΊΠ°ΠΆΠ΄ΠΎΠΉ Ρ‚ΠΎΡ‡ΠΊΠ΅ ΠΊΠΎΡ‚ΠΎΡ€ΠΎΠ³ΠΎ Π±ΡƒΠ΄Π΅Ρ‚ записано расстояниС ΠΎΡ‚ участка с Ρ†Π΅Π½Ρ‚Ρ€ΠΎΠΌ Π² Π΄ΠΎ базиса.

Рис. 4. Π‘Π»Π΅Π²Π° ΠΎΡ€ΠΈΠ³ΠΈΠ½Π°Π»ΡŒΠ½Ρ‹ΠΉ ΠΊΠ°Π΄Ρ€. Π‘ΠΏΡ€Π°Π²Π° Ρ€Π΅Π·ΡƒΠ»ΡŒΡ‚Π°Ρ‚ ΠΏΡ€ΠΎΠ³ΠΎΠ½Π° коррСляционной Ρ„ΡƒΠ½ΠΊΡ†ΠΈΠΈ ΠΏΠΎ ΠΎΡ€ΠΈΠ³ΠΈΠ½Π°Π»ΡŒΠ½ΠΎΠΌΡƒ ΠΊΠ°Π΄Ρ€Ρƒ. Для удобства Π²ΠΈΠ·ΡƒΠ°Π»ΠΈΠ·Π°Ρ†ΠΈΠΈ ΠΊΠ°Π΄Ρ€ Π½ΠΎΡ€ΠΌΠΈΡ€ΠΎΠ²Π°Π½ ΠΎΡ‚ 0 Π΄ΠΎ 255, Π³Π΄Π΅ 255 — Π½Π°ΠΈΠ±ΠΎΠ»Π΅Π΅ Π±Π»ΠΈΠ·ΠΊΠΎΠ΅ ΠΊ базису ΠΎΠΊΠ½ΠΎ. Базис Π³Π»Π°Π²Π½Ρ‹Ρ… ΠΊΠΎΠΌΠΏΠΎΠ½Π΅Π½Ρ‚ ΠΎΠ±ΡƒΡ‡Π΅Π½ Π½Π° изобраТСниях ΠΌΠ°Π½Π΅ΠΊΠ΅Π½Π°, стоящСго Π² Ρ†Π΅Π½Ρ‚Ρ€Π΅, поэтому коррСляционная функция ΠΎΠΆΠΈΠ΄Π°Π΅ΠΌΠΎ Π²Ρ‹Π΄Π°Π»Π° наибольшСС Π·Π½Π°Ρ‡Π΅Π½ΠΈΠ΅ Ρ‚ΠΎΡ‡ΠΊΠ΅, Π² ΠΊΠΎΡ‚ΠΎΡ€ΠΎΠΉ этот ΠΌΠ°Π½Π΅ΠΊΠ΅Π½ находится.

Π—Π°Π΄Π°Ρ‡Π° Ρ‚Ρ€Π΅ΠΊΠΈΠ½Π³Π° Π² ΠΎΠ±Ρ‰Π΅ΠΌ Π²ΠΈΠ΄Π΅ слоТна ΠΈ ΠΏΠΎΠ΄Π²Π΅Ρ€ΠΆΠ΅Π½Π° ΠΌΠ½ΠΎΠ³ΠΈΠΌ ошибкам связанных ΠΊΠ°ΠΊ с Ρ„Π°ΠΊΡ‚ΠΎΡ€Π°ΠΌΠΈ окруТСния, Ρ‚Π°ΠΊ ΠΈ с ΠΏΠΎΠ²Π΅Π΄Π΅Π½ΠΈΠ΅ΠΌ самого ΠΎΠ±ΡŠΠ΅ΠΊΡ‚Π°. К ΠΏΠ΅Ρ€Π²Ρ‹ΠΌ относятся ΠΈΠ·ΠΌΠ΅Π½Π΅Π½ΠΈΠ΅ освСщСния, Π΄Π²ΠΈΠΆΠ΅Π½ΠΈΠ΅ ΠΊΠ°ΠΌΠ΅Ρ€Ρ‹ ΠΈ всСвозмоТныС пСрСкрытия Ρ†Π΅Π»ΠΈ Π΄Ρ€ΡƒΠ³ΠΈΠΌΠΈ ΠΎΠ±ΡŠΠ΅ΠΊΡ‚Π°ΠΌΠΈ. Ко Π²Ρ‚ΠΎΡ€Ρ‹ΠΌ относятся ΠΈΠ·ΠΌΠ΅Π½Π΅Π½ΠΈΠ΅ ΠΏΠΎΠ·ΠΈΡ†ΠΈΠΈ ΠΈ очСртания ΠΎΠ±ΡŠΠ΅ΠΊΡ‚Π°. Для Ρ‚ΠΎΠ³ΠΎ, Ρ‡Ρ‚ΠΎΠ±Ρ‹ ΡƒΡΡ‚Ρ€Π°Π½ΠΈΡ‚ΡŒ влияниС этих Ρ„Π°ΠΊΡ‚ΠΎΡ€ΠΎΠ² ΠΈΠ»ΠΈ ΠΌΠΈΠ½ΠΈΠΌΠΈΠ·ΠΈΡ€ΠΎΠ²Π°Ρ‚ΡŒ ΠΈΡ… послСдствия нашС ΠΏΡ€ΠΈΠ·Π½Π°ΠΊΠΎΠ²ΠΎΠ΅ описаниС (базис Π³Π»Π°Π²Π½Ρ‹Ρ… ΠΊΠΎΠΌΠΏΠΎΠ½Π΅Π½Ρ‚) Π½Π΅ Π΄ΠΎΠ»ΠΆΠ½ΠΎ Π±Ρ‹Ρ‚ΡŒ постоянным, Π° Π΄ΠΎΠ»ΠΆΠ½ΠΎ ΡƒΠΌΠ΅Ρ‚ΡŒ Π°Π΄Π°ΠΏΡ‚ΠΈΡ€ΠΎΠ²Π°Ρ‚ΡŒΡΡ ΠΊ измСнСниям. И IVT Ρ‚Ρ€Π΅ΠΊΠ΅Ρ€ ΡƒΠΌΠ΅Π΅Ρ‚ это Π΄Π΅Π»Π°Ρ‚ΡŒ.

Π’ΠΈΠ΄Π΅ΠΎ 2. Π›Π΅Π²ΠΎΠ΅ Π²ΠΈΠ΄Π΅ΠΎ: потСря Ρ‚Ρ€Π΅ΠΊΠ° Π½Π° 275 ΠΊΠ°Π΄Ρ€Π΅ ΠΈΠ·-Π·Π° константного базиса. ΠŸΡ€Π°Π²ΠΎΠ΅ Π²ΠΈΠ΄Π΅ΠΎ: ΡƒΡΠΏΠ΅ΡˆΠ½ΠΎΠ΅ отслСТиваниС Π°Π²Ρ‚ΠΎ Π΄ΠΎ ΠΊΠΎΠ½Ρ†Π° с ΠΏΠΎΠΌΠΎΡ‰ΡŒΡŽ ΠΈΠ½ΠΊΡ€Π΅ΠΌΠ΅Π½Ρ‚Π½ΠΎΠ³ΠΎ обучСния базиса.

Π’Ρ€ΡŽΠΊ с Π°Π΄Π°ΠΏΡ‚Π°Ρ†ΠΈΠ΅ΠΉ базиса являСтся ΠΊΠ»ΡŽΡ‡Π΅Π²ΠΎΠΉ ΠΎΡΠΎΠ±Π΅Π½Π½ΠΎΡΡ‚ΡŒΡŽ Ρ‚Ρ€Π΅ΠΊΠ΅Ρ€Π°, Ρ‡Ρ‚ΠΎ, Π² своС врСмя, ΠΏΠΎΠ·Π²ΠΎΠ»ΠΈΠ»ΠΎ Π΅ΠΌΡƒ Π²Ρ‹Π΄Π΅Π»ΠΈΡ‚ΡŒΡΡ срСди Π΄Ρ€ΡƒΠ³ΠΈΡ… Ρ‚Ρ€Π΅ΠΊΠ΅Ρ€ΠΎΠ². Основной Π²ΠΊΠ»Π°Π΄ Π°Π²Ρ‚ΠΎΡ€ΠΎΠ² Π΄Π°Π½Π½ΠΎΠ³ΠΎ Ρ‚Ρ€Π΅ΠΊΠ΅Ρ€Π° состоит Π² Ρ‚ΠΎΠΌ, Ρ‡Ρ‚ΠΎ ΠΎΠ½ΠΈ Ρ€Π°Π·Ρ€Π°Π±ΠΎΡ‚Π°Π»ΠΈ ΡΡ„Ρ„Π΅ΠΊΡ‚ΠΈΠ²Π½ΡƒΡŽ ΠΏΡ€ΠΎΡ†Π΅Π΄ΡƒΡ€Ρƒ обновлСния базиса Π³Π»Π°Π²Π½Ρ‹Ρ… ΠΊΠΎΠΌΠΏΠΎΠ½Π΅Π½Ρ‚ ΠΏΠΎ ΠΌΠ΅Ρ€Π΅ накоплСния ΠΈΠ·ΠΌΠ΅Π½Π΅Π½ΠΈΠΉ Π² ΠΊΠ°Π΄Ρ€Π΅. Π­Ρ‚Ρƒ ΠΏΡ€ΠΎΡ†Π΅Π΄ΡƒΡ€Ρƒ ΠΎΠ½ΠΈ Π½Π°Π·Π²Π°Π»ΠΈ Incremental PCA. Π’Π°ΠΊΠΈΠΌ ΠΎΠ±Ρ€Π°Π·ΠΎΠΌ, вмСсто ΠΎΠ΄Π½ΠΎΠΊΡ€Π°Ρ‚Π½ΠΎΠ³ΠΎ обучСния базиса Π² Π½Π°Ρ‡Π°Π»Π΅ Ρ‚Ρ€Π΅ΠΊΠΈΠ½Π³Π° ΠΌΡ‹ способны пСриодичСски ΠΎΠ±Π½ΠΎΠ²Π»ΡΡ‚ΡŒ этот базис ΠΏΠΎ ΠΌΠ΅Ρ€Π΅ двиТСния.

Π˜Π½ΠΊΡ€Π΅ΠΌΠ΅Π½Ρ‚Π½ΠΎΠ΅ ΠΎΠ±ΡƒΡ‡Π΅Π½ΠΈΠ΅ — эффСктивноС ΠΎΠ±Π½ΠΎΠ²Π»Π΅Π½ΠΈΠ΅ базиса Π³Π»Π°Π²Π½Ρ‹Ρ… ΠΊΠΎΠΌΠΏΠΎΠ½Π΅Π½Ρ‚

Π—Π΄Π΅ΡΡŒ ΠΈ Π² дальнСйшСм Π±ΡƒΠ΄ΡƒΡ‚ ΠΈΡΠΏΠΎΠ»ΡŒΠ·ΠΎΠ²Π°Π½Ρ‹ Ρ‚ΠΎΠ»ΡŒΠΊΠΎ основныС матСматичСскиС Π²Ρ‹ΠΊΠ»Π°Π΄ΠΊΠΈ, Π½Π΅ΠΎΠ±Ρ…ΠΎΠ΄ΠΈΠΌΡ‹Π΅ для понимания происходящСго. ВсС, ΠΊΡ‚ΠΎ ΠΈΠ½Ρ‚Π΅Ρ€Π΅ΡΡƒΡŽΡ‚ΡΡ Ρ‚Π΅ΠΎΡ€ΠΈΠ΅ΠΉ ΠΈ Π΄ΠΎΠΊΠ°Π·Π°Ρ‚Π΅Π»ΡŒΡΡ‚Π²Π°ΠΌΠΈ, ΠΌΠΎΠ³ΡƒΡ‚ ΠΎΠ·Π½Π°ΠΊΠΎΠΌΠΈΡ‚ΡŒΡΡ с Π½ΠΈΠΌΠΈ Π² ΠΎΡ€ΠΈΠ³ΠΈΠ½Π°Π»ΡŒΠ½ΠΎΠΉ ΡΡ‚Π°Ρ‚ΡŒΠ΅ [1].

Π—Π°Π΄Π°Ρ‡Π° обновлСния базиса описываСтся ΡΠ»Π΅Π΄ΡƒΡŽΡ‰ΠΈΠΌ ΠΎΠ±Ρ€Π°Π·ΠΎΠΌ. Π”Π°Π½Π° ΠΌΠ°Ρ‚Ρ€ΠΈΡ†Π° собствСнных Π²Π΅ΠΊΡ‚ΠΎΡ€ΠΎΠ² ΠΈ диагональная ΠΌΠ°Ρ‚Ρ€ΠΈΡ†Π° собствСнных Π·Π½Π°Ρ‡Π΅Π½ΠΈΠΉ , ΠΎΠ±ΡƒΡ‡Π΅Π½Π½Ρ‹Ρ… Π½Π° ΠΏΠ΅Ρ€Π²ΠΈΡ‡Π½Ρ‹Ρ… изобраТСниях ΠΎΠ±ΡŠΠ΅ΠΊΡ‚Π° . ВрСбуСтся ΠΎΠ±ΡƒΡ‡ΠΈΡ‚ΡŒ Π½ΠΎΠ²Ρ‹ΠΉ Π½Π°Π±ΠΎΡ€ ΠΈ Π½Π° ΠΏΠ΅Ρ€Π²ΠΈΡ‡Π½Ρ‹Ρ… изобраТСниях ΠΎΠ±ΡŠΠ΅ΠΊΡ‚Π° плюс Π½ΠΎΠ²Ρ‹Ρ… Π²Ρ…ΠΎΠ΄Π½Ρ‹Ρ… изобраТСниях .

Π’Ρ€ΠΈΠ²ΠΈΠ°Π»ΡŒΠ½ΠΎΠ΅ Ρ€Π΅ΡˆΠ΅Π½ΠΈΠ΅ с ΠΏΠ΅Ρ€Π΅ΠΎΠ±ΡƒΡ‡Π΅Π½ΠΈΠ΅ΠΌ базиса ΠΏΡ€ΠΈ ΠΊΠ°ΠΆΠ΄ΠΎΠΌ поступлСнии Π½ΠΎΠ²Ρ‹Ρ… Π΄Π°Π½Π½Ρ‹Ρ… ΠΈΠ»ΠΈ Π΄Π°ΠΆΠ΅ ΠΏΡ€ΠΈ ΠΏΡ€ΠΈ поступлСнии Π½ΠΎΠ²Ρ‹Ρ… Π΄Π°Π½Π½Ρ‹Ρ… выглядит Π½Π΅Ρ€Π°Π·ΡƒΠΌΠ½ΠΎ, Ρ‚Π°ΠΊ ΠΊΠ°ΠΊ с ΡƒΠ²Π΅Π»ΠΈΡ‡Π΅Π½ΠΈΠ΅ΠΌ количСства Π΄Π°Π½Π½Ρ‹Ρ… растСт Ρ€Π°Π·ΠΌΠ΅Ρ€ выдСляСмой для Π½ΠΈΡ… ΠΎΠΏΠ΅Ρ€Π°Ρ‚ΠΈΠ²Π½ΠΎΠΉ памяти ΠΈ врСмя вычислСния.

ΠŸΡ€Π΅Π΄Π»Π°Π³Π°Π΅ΠΌΡ‹ΠΉ Π°Π²Ρ‚ΠΎΡ€Π°ΠΌΠΈ ΠΌΠ΅Ρ‚ΠΎΠ΄ ΠΈΠ½ΠΊΡ€Π΅ΠΌΠ΅Π½Ρ‚Π½ΠΎΠ³ΠΎ обучСния/обновлСния собствСнного базиса, основанный Π½Π° ΠΏΠΎΡΠ»Π΅Π΄ΠΎΠ²Π°Ρ‚Π΅Π»ΡŒΠ½ΠΎΠΌ ΠœΠ΅Ρ‚ΠΎΠ΄Π΅ ΠšΠ°Ρ€ΡƒΠ½Π΅Π½Π°-Лоэва (Sequential Karhunen-Loeve) [4], ΡƒΠΌΠ΅Π΅Ρ‚ ΠΎΠ±Π½ΠΎΠ²Π»ΡΡ‚ΡŒ базис ΠΈΡΠΏΠΎΠ»ΡŒΠ·ΡƒΡ Ρ‚ΠΎΠ»ΡŒΠΊΠΎ послСдниС наблюдСний, Π½Π΅ уничтоТая влияниС ΠΏΡ€Π΅Π΄Ρ‹Π΄ΡƒΡ‰ΠΈΡ… наблюдСний, ΡΠ»Π΅Π΄ΠΎΠ²Π°Ρ‚Π΅Π»ΡŒΠ½ΠΎ врСмя вычислСния всСгда постоянно. Π‘Π»ΠΎΠΆΠ½ΠΎΡΡ‚ΡŒ Π΄Π°Π½Π½ΠΎΠ³ΠΎ Ρ€Π΅ΡˆΠ΅Π½ΠΈΡ составляСт , Π³Π΄Π΅ — Ρ€Π°Π·ΠΌΠ΅Ρ€Π½ΠΎΡΡ‚ΡŒ Π²Ρ…ΠΎΠ΄Π°, Π² Ρ‚ΠΎ врСмя ΠΊΠ°ΠΊ ΡΠ»ΠΎΠΆΠ½ΠΎΡΡ‚ΡŒ Ρ‚Ρ€ΠΈΠ²ΠΈΠ°Π»ΡŒΠ½ΠΎΠ³ΠΎ Ρ€Π΅ΡˆΠ΅Π½ΠΈΡ составляСт . ΠšΡ€ΠΎΠΌΠ΅ этого, Π°Π²Ρ‚ΠΎΡ€Ρ‹ вводят Π² уравнСния обновлСния коэффициСнт затухания (forgetting factor), ΡΠ½ΠΈΠΆΠ°ΡŽΡ‰ΠΈΠΉ влияниС Π±ΠΎΠ»Π΅Π΅ старых Π΄Π°Π½Π½Ρ‹Ρ… Π½Π° Π½ΠΎΠ²Ρ‹ΠΉ базис, удСляя большС внимания Π±ΠΎΠ»Π΅Π΅ Π½ΠΎΠ²Ρ‹ΠΌ Π΄Π°Π½Π½Ρ‹ΠΌ. ПолноС описаниС Π°Π»Π³ΠΎΡ€ΠΈΡ‚ΠΌΠ° с Ρ„ΠΎΡ€ΠΌΡƒΠ»Π°ΠΌΠΈ для вычислСния ΠΏΡ€ΠΈΠ²Π΅Π΄Π΅Π½ΠΎ Π² ΡΡ‚Π°Ρ‚ΡŒΠ΅. На Π²ΠΈΠ΄Π΅ΠΎ 2 прСдставлСн ΠΏΡ€ΠΈΠΌΠ΅Ρ€ обновлСния срСднСго ΠΈ собствСнного базиса.

Π˜Ρ‚Π°ΠΊ, ΠΌΡ‹ поняли, Ρ‡Ρ‚ΠΎ Ρ†Π΅Π»Π΅Π²ΠΎΠΉ ΠΎΠ±ΡŠΠ΅ΠΊΡ‚ Π±ΡƒΠ΄Π΅Ρ‚ ΠΏΡ€Π΅Π΄ΡΡ‚Π°Π²Π»ΡΡ‚ΡŒΡΡ Ρ‡Π΅Ρ€Π΅Π· базис Π³Π»Π°Π²Π½Ρ‹Ρ… ΠΊΠΎΠΌΠΏΠΎΠ½Π΅Π½Ρ‚ ΠΈ Ρ‡Ρ‚ΠΎ это базис Π±ΡƒΠ΄Π΅Ρ‚ пСриодичСски ΠΎΠ±Π½ΠΎΠ²Π»ΡΡ‚ΡŒΡΡ для Ρ‚ΠΎΠ³ΠΎ, Ρ‡Ρ‚ΠΎΠ±Ρ‹ Π°Π΄Π°ΠΏΡ‚ΠΈΡ€ΠΎΠ²Π°Ρ‚ΡŒΡΡ ΠΊ измСнСниям. ΠŸΠ΅Ρ€Π΅ΠΉΠ΄Π΅ΠΌ Ρ‚Π΅ΠΏΠ΅Ρ€ΡŒ ΠΊ Ρ„ΠΎΡ€ΠΌΠ°Π»ΡŒΠ½ΠΎΠΉ постановки Π·Π°Π΄Π°Ρ‡ΠΈ.

Π€ΠΎΡ€ΠΌΠ°Π»ΡŒΠ½Π°Ρ постановка Π·Π°Π΄Π°Ρ‡ΠΈ Ρ‚Ρ€Π΅ΠΊΠΈΠ½Π³Π°

ΠœΡ‹ Π±ΡƒΠ΄Π΅ΠΌ Ρ€Π°ΡΡΠΌΠ°Ρ‚Ρ€ΠΈΠ²Π°Ρ‚ΡŒ Π·Π°Π΄Π°Ρ‡Ρƒ Ρ‚Ρ€Π΅ΠΊΠΈΠ½Π³Π° Π² Ρ€Π°ΠΌΠΊΠ°Ρ… стохастичСского марковского процСсса, Π»Π΅ΠΆΠ°Ρ‰Π΅Π³ΠΎ Π² основС двиТСния ΠΎΠ±ΡŠΠ΅ΠΊΡ‚Π°. Π‘ΡƒΠ΄Π΅ΠΌ ΡΡ‡ΠΈΡ‚Π°Ρ‚ΡŒ, Ρ‡Ρ‚ΠΎ ΠΌΡ‹ Π²Π»Π°Π΄Π΅Π΅ΠΌ Π°ΠΏΡ€ΠΈΠΎΡ€Π½Ρ‹ΠΌ Π·Π½Π°Π½ΠΈΠ΅ΠΌ Ρ‚ΠΎΠ³ΠΎ, Π³Π΄Π΅ находится ΠΎΠ±ΡŠΠ΅ΠΊΡ‚ Π² Π½Π°Ρ‡Π°Π»ΡŒΠ½ΠΎΠΌ ΠΊΠ°Π΄Ρ€Π΅. Π­Ρ‚ΠΎ Π°ΠΏΡ€ΠΈΠΎΡ€Π½ΠΎΠ΅ Π·Π½Π°Π½ΠΈΠ΅ ΠΌΠΎΠΆΠ½ΠΎ ΠΏΠΎΠ»ΡƒΡ‡ΠΈΡ‚ΡŒ с ΠΏΠΎΠΌΠΎΡ‰ΡŒΡŽ нСйросСтСвого Π΄Π΅Ρ‚Π΅ΠΊΡ‚ΠΎΡ€Π°. Π’ процСссС двиТСния ΠΌΡ‹ наблюдаСм Π΄Π°Π½Π½Ρ‹Π΅ ΠΈΠ· ΠΊΠΎΡ‚ΠΎΡ€Ρ‹Ρ… ΠΌΡ‹ ΠΌΠΎΠΆΠ΅ΠΌ вывСсти апостСриорноС Π·Π½Π°Π½ΠΈΠ΅ ΠΎ Π½ΠΎΠ²ΠΎΠΌ мСстополоТСнии ΠΎΠ±ΡŠΠ΅ΠΊΡ‚Π°.

ПолоТСниС ΠΎΠ±ΡŠΠ΅ΠΊΡ‚Π° Π² ΠΌΠΎΠΌΠ΅Π½Ρ‚ Π²Ρ€Π΅ΠΌΠ΅Π½ΠΈ Π±ΡƒΠ΄Π΅Ρ‚ Ρ€Π°ΡΡΠΌΠ°Ρ‚Ρ€ΠΈΠ²Π°Ρ‚ΡŒΡΡ ΠΊΠ°ΠΊ скрытоС состояниС . Π­Ρ‚ΠΎ состояниС ΠΌΠΎΠΆΠ΅Ρ‚ Π±Ρ‹Ρ‚ΡŒ Π·Π°Π΄Π°Π½ΠΎ ΠΌΠ½ΠΎΠ³ΠΈΠΌΠΈ способами Π² зависимости ΠΎΡ‚ Ρ„ΠΈΠ³ΡƒΡ€Ρ‹ ΠΎΠ±ΡŠΠ΅ΠΊΡ‚Π°. Для сохранСния общности Π±ΡƒΠ΄Π΅ΠΌ ΠΏΡ€Π΅Π΄ΠΏΠΎΠ»Π°Π³Π°Ρ‚ΡŒ, Ρ‡Ρ‚ΠΎ ΠΎΠ±ΡŠΠ΅ΠΊΡ‚ Π·Π°ΠΊΠ»ΡŽΡ‡Π΅Π½ Π² Ρ‡Π΅Ρ‚Ρ‹Ρ€Π΅Ρ…ΡƒΠ³ΠΎΠ»ΡŒΠ½ΡƒΡŽ Ρ€Π°ΠΌΠΊΡƒ. Π’ΠΎΠ³Π΄Π° состояниС прСдставляСт собой Π°Ρ„Ρ„ΠΈΠ½Π½ΠΎΠ΅ ΠΏΡ€Π΅ΠΎΠ±Ρ€Π°Π·ΠΎΠ²Π°Π½ΠΈΠ΅ для Ρ‡Π΅Ρ‚Ρ‹Ρ€Π΅Ρ…ΡƒΠ³ΠΎΠ»ΡŒΠ½ΠΈΠΊΠ°, Π³Π΄Π΅ ΠΏΠ°Ρ€Π°ΠΌΠ΅Ρ‚Ρ€Ρ‹ Π² скобках ΠΎΠ·Π½Π°Ρ‡Π°ΡŽΡ‚ ΠΊΠΎΠΎΡ€Π΄ΠΈΠ½Π°Ρ‚Ρ‹ Ρ†Π΅Π½Ρ‚Ρ€Π° Ρ€Π°ΠΌΠΊΠΈ, ΡƒΠ³ΠΎΠ» ΠΏΠΎΠ²ΠΎΡ€ΠΎΡ‚Π°, ΠΌΠ°ΡΡˆΡ‚Π°Π±, ΡΠΎΠΎΡ‚Π½ΠΎΡˆΠ΅Π½ΠΈΠ΅ сторон ΠΈ Π½Π°ΠΊΠ»ΠΎΠ½ соотвСтствСнно.

НаблюдаСмой Π²Π΅Π»ΠΈΡ‡ΠΈΠ½ΠΎΠΉ Π±ΡƒΠ΄Π΅Ρ‚ ΡΠ²Π»ΡΡ‚ΡŒΡΡ ΠΈΠ·ΠΎΠ±Ρ€Π°ΠΆΠ΅Π½ΠΈΠ΅ ΠΎΠ±ΡŠΠ΅ΠΊΡ‚Π° с ΠΊΠ°ΠΌΠ΅Ρ€Ρ‹.

ГрафичСская модСль марковского процСсса ΠΏΡ€ΠΎΠΈΠ»Π»ΡŽΡΡ‚Ρ€ΠΈΡ€ΠΎΠ²Π°Π½Π° Π½ΠΈΠΆΠ΅.

Рис. 5. ГрафичСскоС прСдставлСниС скрытой марковской ΠΌΠΎΠ΄Π΅Π»ΠΈ.

Π˜Ρ‚Π°ΠΊ, Π½Π°ΠΌ Π΄Π°Π½ΠΎ Π½Π°Ρ‡Π°Π»ΡŒΠ½ΠΎΠ΅ ΠΏΠΎΠ»ΠΎΠΆΠ΅Π½ΠΈΠ΅ ΠΎΠ±ΡŠΠ΅ΠΊΡ‚Π° . ΠœΡ‹ ΠΏΠΎΠ»ΡƒΡ‡ΠΈΠ»ΠΈ наблюдСниС . ВрСбуСтся ΠΏΡ€Π΅Π΄ΡΠΊΠ°Π·Π°Ρ‚ΡŒ Π½ΠΎΠ²ΠΎΠ΅ мСстополоТСниС ΠΎΠ±ΡŠΠ΅ΠΊΡ‚Π° Π² ΠΌΠΎΠΌΠ΅Π½Ρ‚ Π²Ρ€Π΅ΠΌΠ΅Π½ΠΈ . На языкС Ρ‚Π΅ΠΎΡ€ΠΈΠΈ вСроятностСй это выраТаСтся ΠΊΠ°ΠΊ — Π²Π΅Ρ€ΠΎΡΡ‚Π½ΠΎΡΡ‚ΡŒ ΠΏΡ€ΠΈ условии ΠΈ . ΠŸΡ€ΠΈΠΌΠ΅Π½ΡΡ Ρ‚Π΅ΠΎΡ€Π΅ΠΌΡƒ БайСса Π½Π°Ρ…ΠΎΠ΄ΠΈΠΌ

ΠžΠΏΡƒΡΠΊΠ°Ρ лишниС зависимости (ΠΏΠΎΠ»ΡŒΠ·ΡƒΡΡΡŒ марковским свойством) ΠΈ отсСкая Π½ΠΎΡ€ΠΌΠΈΡ€ΠΎΠ²ΠΎΡ‡Π½Ρ‹ΠΉ коэффициСнт ΠΏΠΎΠ»ΡƒΡ‡Π°Π΅ΠΌ

Ρ‡Ρ‚ΠΎ искомая Π²Π΅Ρ€ΠΎΡΡ‚Π½ΠΎΡΡ‚ΡŒ ΠΏΡ€ΠΎΠΏΠΎΡ€Ρ†ΠΈΠΎΠ½Π°Π»ΡŒΠ½Π° ΠΏΡ€ΠΎΠΈΠ·Π²Π΅Π΄Π΅Π½ΠΈΡŽ Π΄Π²ΡƒΡ… ΠΌΠ½ΠΎΠΆΠΈΡ‚Π΅Π»Π΅ΠΉ. ΠŸΠ΅Ρ€Π²Ρ‹ΠΉ ΠΌΠ½ΠΎΠΆΠΈΡ‚Π΅Π»ΡŒ это модСль двиТСния ΠΈΠ»ΠΈ β€œΠ³Π΄Π΅ Π±ΡƒΠ΄Π΅Ρ‚ Π½Π°Ρ…ΠΎΠ΄ΠΈΡ‚ΡŒΡΡ ΠΎΠ±ΡŠΠ΅ΠΊΡ‚ Π² ΠΌΠΎΠΌΠ΅Π½Ρ‚ Π²Ρ€Π΅ΠΌΠ΅Π½ΠΈ , ΠΏΡ€ΠΈ условии, Ρ‡Ρ‚ΠΎ Π² ΠΌΠΎΠΌΠ΅Π½Ρ‚ Π²Ρ€Π΅ΠΌΠ΅Π½ΠΈ ΠΎΠ±ΡŠΠ΅ΠΊΡ‚ находился Π² . Π’Ρ‚ΠΎΡ€ΠΎΠΉ ΠΌΠ½ΠΎΠΆΠΈΡ‚Π΅Π»ΡŒ это модСль наблюдСния ΠΈΠ»ΠΈ β€œΠ½Π°ΡΠΊΠΎΠ»ΡŒΠΊΠΎ ΠΏΡ€Π°Π²Π΄ΠΎΠΏΠΎΠ΄ΠΎΠ±Π½ΠΎ Π±Ρ‹Π»ΠΎ Π±Ρ‹ наблюдСниС , Ссли Π±Ρ‹ Π² ΠΌΠΎΠΌΠ΅Π½Ρ‚ Π²Ρ€Π΅ΠΌΠ΅Π½ΠΈ ΠΎΠ±ΡŠΠ΅ΠΊΡ‚ находился Π² . Π’ ΡΠ»Π΅Π΄ΡƒΡŽΡ‰ΠΈΡ… Ρ€Π°Π·Π΄Π΅Π»Π°Ρ… ΠΌΡ‹ рассмотрим эти ΠΌΠ½ΠΎΠΆΠΈΡ‚Π΅Π»ΠΈ ΠΏΠΎΠ΄Ρ€ΠΎΠ±Π½Π΅Π΅.

Π’Π΅ΠΏΠ΅Ρ€ΡŒ ΠΌΡ‹ ΠΎΠ±Π»Π°Π΄Π°Π΅ΠΌ Π·Π½Π°Π½ΠΈΠ΅ΠΌ ΠΎ мСстополоТСнии ΠΎΠ±ΡŠΠ΅ΠΊΡ‚Π° Π² ΠΌΠΎΠΌΠ΅Π½Ρ‚ Π²Ρ€Π΅ΠΌΠ΅Π½ΠΈ . Π‘Π΄Π΅Π»Π°Π΅ΠΌ ΠΎΠ΄ΠΈΠ½ шаг Π²ΠΏΠ΅Ρ€Π΅Π΄ ΠΊ ΠΈ ΠΏΡ€ΠΎΠ½Π°Π±Π»ΡŽΠ΄Π°Π΅ΠΌ . Как Ρ‚Π΅ΠΏΠ΅Ρ€ΡŒ Π½Π°ΠΌ Π½Π°ΠΉΡ‚ΠΈ мСстополоТСниС ΠΎΠ±ΡŠΠ΅ΠΊΡ‚Π° Π½Π° Π½ΠΎΠ²ΠΎΠΌ шагС? На самом Π΄Π΅Π»Π΅ всС ΠΎΡ‡Π΅Π²ΠΈΠ΄Π½ΠΎ. ВспоминаСм, Ρ‡Ρ‚ΠΎ для опрСдСлСния Ρ‚Π΅ΠΊΡƒΡ‰Π΅Π³ΠΎ полоТСния Π½Π°ΠΌ достаточно Π·Π½Π°Ρ‚ΡŒ ΠΏΡ€Π΅Π΄Ρ‹Π΄ΡƒΡ‰Π΅Π΅ ΠΏΠΎΠ»ΠΎΠΆΠ΅Π½ΠΈΠ΅ ΠΎΠ±ΡŠΠ΅ΠΊΡ‚Π° ΠΈ Ρ‚Π΅ΠΊΡƒΡ‰Π΅Π΅ наблюдСниС. Но ΠΌΡ‹ ΡƒΠΆΠ΅ Π·Π½Π°Π΅ΠΌ ΠΏΡ€Π΅Π΄Ρ‹Π΄ΡƒΡ‰Π΅Π΅ мСстополоТСниС ΠΎΠ±ΡŠΠ΅ΠΊΡ‚Π°, ΠΌΡ‹ Π²Ρ‹Π²Π΅Π»ΠΈ Π΅Π³ΠΎ Π½Π° шагС , поэтому всС, Ρ‡Ρ‚ΠΎ Π½ΡƒΠΆΠ½ΠΎ ΡΠ΄Π΅Π»Π°Ρ‚ΡŒ, это ΠΏΠΎΠ΄ΡΡ‚Π°Π²ΠΈΡ‚ΡŒ Π΅Π³ΠΎ Π² Ρ„ΠΎΡ€ΠΌΡƒΠ»Ρƒ Π² качСствС ΠΏΡ€Π΅Π΄Ρ‹Π΄ΡƒΡ‰Π΅Π³ΠΎ наблюдСния, Ρ‚ΠΎ Π΅ΡΡ‚ΡŒ .

ВсС Π³ΠΎΡ‚ΠΎΠ²ΠΎ ΠΊ Ρ‚ΠΎΠΌΡƒ, Ρ‡Ρ‚ΠΎΠ±Ρ‹ Π·Π°ΠΏΠΈΡΠ°Ρ‚ΡŒ ΠΎΠ±Ρ‰ΡƒΡŽ Ρ€Π΅ΠΊΡƒΡ€ΡΠΈΠ²Π½ΡƒΡŽ Ρ„ΠΎΡ€ΠΌΡƒΠ»Ρƒ для ΠΏΡ€ΠΎΠΈΠ·Π²ΠΎΠ»ΡŒΠ½ΠΎΠ³ΠΎ шага

МоТно Π·Π°ΠΌΠ΅Ρ‚ΠΈΡ‚ΡŒ, насколько элСгантно Π² ΠΊΡ€Π°Ρ‚ΠΊΠΎΠΉ Ρ„ΠΎΡ€ΠΌΡƒΠ»Π΅ записываСтся Ρ€Π΅ΡˆΠ΅Π½ΠΈΠ΅ нашСй Π·Π°Π΄Π°Ρ‡ΠΈ. По сути, Π² Π²Ρ‹Ρ€Π°ΠΆΠ΅Π½ΠΈΠΈ содСрТатся (Ρ…ΠΎΡ‚ΡŒ ΠΈ Π² ΠΏΡ€ΠΎΠΈΠ½Ρ‚Π΅Π³Ρ€ΠΈΡ€ΠΎΠ²Π°Π½Π½ΠΎΠΌ Π²ΠΈΠ΄Π΅) всС ΠΏΡ€Π΅Π΄Ρ‹Π΄ΡƒΡ‰ΠΈΠ΅ знания ΠΎ ΠΏΠ΅Ρ€Π΅ΠΌΠ΅Ρ‰Π΅Π½ΠΈΠΈ ΠΎΠ±ΡŠΠ΅ΠΊΡ‚Π° (ΠΎΠ½ΠΎ соотвСтствуСт Π²Π΅Ρ€Ρ…Π½Π΅ΠΉ Π³ΠΎΡ€ΠΈΠ·ΠΎΠ½Ρ‚Π°Π»ΡŒΠ½ΠΎΠΉ стрСлочкС Π² графичСской ΠΌΠΎΠ΄Π΅Π»ΠΈ Π½Π° рисункС 5). На самом Π΄Π΅Π»Π΅ байСсовский Π²Ρ‹Π²ΠΎΠ΄ Π²ΠΎΠ·Π½ΠΈΠΊΠ°Π΅Ρ‚ СстСствСнным ΠΎΠ±Ρ€Π°Π·ΠΎΠΌ для ΠΌΠΎΠ΄Π΅Π»Π΅ΠΉ ΠΏΠΎΡΠ»Π΅Π΄ΠΎΠ²Π°Ρ‚Π΅Π»ΡŒΠ½ΠΎΠΉ ΠΎΠ±Ρ€Π°Π±ΠΎΡ‚ΠΊΠΈ Π΄Π°Π½Π½Ρ‹Ρ…, ΠΊΠ°ΠΊΠΎΠ²ΠΎΠΉ являСтся ΠΈ наша марковская модСль, ΠΈΠ·-Π·Π° способности ΠΎΠ±Π½ΠΎΠ²Π»ΡΡ‚ΡŒ апостСриорноС Π·Π½Π°Π½ΠΈΠ΅ ΠΏΠΎ ΠΌΠ΅Ρ€Π΅ поступлСния Π½ΠΎΠ²ΠΎΠΉ ΠΈΠ½Ρ„ΠΎΡ€ΠΌΠ°Ρ†ΠΈΠΈ.

МодСль двиТСния (dynamical model)

Π˜ΡΡ‚ΠΎΡ‡Π½ΠΈΠΊ http://www.anuncommonlab.com/articles/how-kalman-filters-work/.

МодСль двиТСния Π·Π°Π΄Π°Π΅Ρ‚ Π·Π°ΠΊΠΎΠ½, ΠΊΠΎΡ‚ΠΎΡ€ΠΎΠΌΡƒ подчиняСтся Π΄Π²ΠΈΠΆΠ΅Π½ΠΈΠ΅ ΠΎΠ±ΡŠΠ΅ΠΊΡ‚Π°, Ρ‚ΠΎ Π΅ΡΡ‚ΡŒ ΠΏΠ΅Ρ€Π΅Ρ…ΠΎΠ΄ ΠΎΡ‚ состояния ΠΊ ΡΠΎΡΡ‚ΠΎΡΠ½ΠΈΡŽ . Он опрСдСляСтся Π·Π°Ρ€Π°Π½Π΅Π΅ Π½Π° основС Π½Π΅ΠΊΠΈΡ… тСорСтичСских свСдСний ΠΎΠ± ΠΎΠ±ΡŠΠ΅ΠΊΡ‚Π΅. Π­Ρ‚ΠΎ ΠΌΠΎΠΆΠ΅Ρ‚ Π±Ρ‹Ρ‚ΡŒ, Π½Π°ΠΏΡ€ΠΈΠΌΠ΅Ρ€, линСйная модСль ΠΈΠ»ΠΈ Π΄ΠΈΡ„Ρ„Π΅Ρ€Π΅Π½Ρ†ΠΈΠ°Π»ΡŒΠ½ΠΎΠ΅ ΡƒΡ€Π°Π²Π½Π΅Π½ΠΈΠ΅ ΠΏΡ€ΠΎΠΈΠ·Π²ΠΎΠ»ΡŒΠ½ΠΎΠ³ΠΎ порядка. Π’Π°ΠΊ ΠΊΠ°ΠΊ Π² этой ΡΡ‚Π°Ρ‚ΡŒΠ΅ ΠΌΡ‹ рассматриваСм отслСТиваниС ΠΏΡ€ΠΎΠΈΠ·Π²ΠΎΠ»ΡŒΠ½ΠΎΠ³ΠΎ ΠΎΠ±ΡŠΠ΅ΠΊΡ‚Π°, Π΄ΠΈΠ½Π°ΠΌΠΈΠΊΠ° ΠΊΠΎΡ‚ΠΎΡ€ΠΎΠ³ΠΎ Π½Π°ΠΌ нСизвСстна, Ρ‚ΠΎ Π² качСствС ΠΌΠΎΠ΄Π΅Π»ΠΈ двиТСния ΠΌΡ‹ возьмСм случайноС ΠΏΠ΅Ρ€Π΅ΠΌΠ΅Ρ‰Π΅Π½ΠΈΠ΅ ΠΎΠ±ΡŠΠ΅ΠΊΡ‚Π° Π² любом ΠΈΠ· Π½Π°ΠΏΡ€Π°Π²Π»Π΅Π½ΠΈΠΉ — БроуновскоС Π΄Π²ΠΈΠΆΠ΅Π½ΠΈΠ΅. ΠœΡ‹ ΠΌΠΎΠΆΠ΅ΠΌ Π·Π°ΠΏΠΈΡΠ°Ρ‚ΡŒ это Π΄Π²ΠΈΠΆΠ΅Π½ΠΈΠ΅ Π² Π²ΠΈΠ΄Π΅ ΠΌΠ½ΠΎΠ³ΠΎΠΌΠ΅Ρ€Π½ΠΎΠ³ΠΎ Π½ΠΎΡ€ΠΌΠ°Π»ΡŒΠ½ΠΎΠ³ΠΎ распрСдСлСния , Π³Π΄Π΅ — диагональная ковариационная ΠΌΠ°Ρ‚Ρ€ΠΈΡ†Π°, ΠΊΠ°ΠΆΠ΄Ρ‹ΠΉ элСмСнт ΠΊΠΎΡ‚ΠΎΡ€ΠΎΠΉ Π½Π° Π³Π»Π°Π²Π½ΠΎΠΉ Π΄ΠΈΠ°Π³ΠΎΠ½Π°Π»ΠΈ Ρ€Π°Π²Π΅Π½ диспСрсии ΠΎΠ΄Π½ΠΎΠ³ΠΎ ΠΈΠ· ΠΏΠ°Ρ€Π°ΠΌΠ΅Ρ‚Ρ€ΠΎΠ², Ρ‚ΠΎ Π΅ΡΡ‚ΡŒ . Π˜Π½Ρ‹ΠΌΠΈ словами, ΠΊΠ°ΠΆΠ΄Ρ‹ΠΉ ΠΏΠ°Ρ€Π°ΠΌΠ΅Ρ‚Ρ€ распрСдСлСн Π½ΠΎΡ€ΠΌΠ°Π»ΡŒΠ½ΠΎ Π²ΠΎΠΊΡ€ΡƒΠ³ своСго Ρ†Π΅Π½Ρ‚Ρ€Π° (ΠΏΡ€Π΅Π΄Ρ‹Π΄ΡƒΡ‰Π΅Π³ΠΎ ΠΏΠ°Ρ€Π°ΠΌΠ΅Ρ‚Ρ€Π°) со срСднСквадратичным ΠΎΡ‚ΠΊΠ»ΠΎΠ½Π΅Π½ΠΈΠ΅ΠΌ .

ГСомСтричСски, ΠΏΠ΅Ρ€Π΅Ρ…ΠΎΠ΄ ΠΎΡ‚ ΠΏΡ€Π΅Π΄Ρ‹Π΄ΡƒΡ‰Π΅Π³ΠΎ состояния Π² ΡΠ»Π΅Π΄ΡƒΡŽΡ‰ΠΈΠΉ Π±ΡƒΠ΄Π΅Ρ‚ ΠΎΠ±Ρ€Π°Π·ΠΎΠ²Ρ‹Π²Π°Ρ‚ΡŒ ΠΎΠ±Π»Π°ΠΊΠΎ Ρ‡Π΅Ρ‚Ρ‹Ρ€Π΅Ρ…ΡƒΠ³ΠΎΠ»ΡŒΠ½ΠΈΠΊΠΎΠ² Π²ΠΎΠΊΡ€ΡƒΠ³ ΠΏΡ€Π΅Π΄Ρ‹Π΄ΡƒΡ‰Π΅Π³ΠΎ состояния. НовоС мСстополоТСниС ΠΎΠ±ΡŠΠ΅ΠΊΡ‚Π°, согласно нашСй ΠΌΠΎΠ΄Π΅Π»ΠΈ, Π΄ΠΎΠ»ΠΆΠ½ΠΎ ΠΎΠΊΠ°Π·Π°Ρ‚ΡŒΡΡ Π² ΠΎΠ΄Π½ΠΎΠΌ ΠΈΠ· Ρ‡Π΅Ρ‚Ρ‹Ρ€Π΅Ρ…ΡƒΠ³ΠΎΠ»ΡŒΠ½ΠΈΠΊΠΎΠ². Π’ ΠΊΠ°ΠΊΠΎΠΌ ΠΈΠΌΠ΅Π½Π½ΠΎ — Ρ€Π΅ΡˆΠΈΡ‚ модСль наблюдСния.

Рис. 6. Разброс состояний Π²ΠΎΠΊΡ€ΡƒΠ³ ΠΏΡ€Π΅Π΄Ρ‹Π΄ΡƒΡ‰Π΅Π³ΠΎ состояния (красный ΠΏΡ€ΡΠΌΠΎΡƒΠ³ΠΎΠ»ΡŒΠ½ΠΈΠΊ). ΠšΠ°ΠΆΠ΄Ρ‹ΠΉ синий Ρ‡Π΅Ρ‚Ρ‹Ρ€Π΅Ρ…ΡƒΠ³ΠΎΠ»ΡŒΠ½ΠΈΠΊ это Π³ΠΈΠΏΠΎΡ‚Π΅Π·Π° ΠΎ мСстонахоТдСнии ΠΎΠ±ΡŠΠ΅ΠΊΡ‚Π°.

МодСль наблюдСния (observation model)

КаТдоС Π½ΠΎΠ²ΠΎΠ΅ наблюдСниС вносит Π½Π΅ΠΊΡƒΡŽ ΠΈΠ½Ρ„ΠΎΡ€ΠΌΠ°Ρ†ΠΈΡŽ, Π½Π° основС ΠΊΠΎΡ‚ΠΎΡ€ΠΎΠΉ ΠΌΡ‹ ΠΌΠΎΠΆΠ΅ΠΌ Π΄Π΅Π»Π°Ρ‚ΡŒ Π²Ρ‹Π²ΠΎΠ΄ ΠΎ мСстонахоТдСнии ΠΎΠ±ΡŠΠ΅ΠΊΡ‚Π°. Для Ρ‚ΠΎΠ³ΠΎ, Ρ‡Ρ‚ΠΎΠ±Ρ‹ Π²ΠΊΠ»ΡŽΡ‡ΠΈΡ‚ΡŒ эту ΠΈΠ½Ρ„ΠΎΡ€ΠΌΠ°Ρ†ΠΈΡŽ Π² модСль ΠΌΡ‹ Π΄ΠΎΠ»ΠΆΠ½Ρ‹ Π·Π°Π΄Π°Ρ‚ΡŒ связь ΠΌΠ΅ΠΆΠ΄Ρƒ наблюдСниСм ΠΈ состояниСм . Π­Ρ‚ΠΎ ΠΈ Π΅ΡΡ‚ΡŒ модСль наблюдСния. Π’ байСсовской ΠΈΠ½Ρ‚Π΅Ρ€ΠΏΡ€Π΅Ρ‚Π°Ρ†ΠΈΠΈ модСль наблюдСния задаСтся распрСдСлСниСм — ΠΏΡ€Π°Π²Π΄ΠΎΠΏΠΎΠ΄ΠΎΠ±ΠΈΠ΅ΠΌ Ρ‚ΠΎΠ³ΠΎ, Ρ‡Ρ‚ΠΎ Π½Π°Ρ…ΠΎΠ΄ΡΡΡŒ Π² ΠΏΠΎΠ·ΠΈΡ†ΠΈΠΈ , ΠΌΡ‹ ΠΏΡ€ΠΎΠ½Π°Π±Π»ΡŽΠ΄Π°Π΅ΠΌ . Π’Π°ΠΊ ΠΊΠ°ΠΊ для наблюдаСмого ΠΎΠ±ΡŠΠ΅ΠΊΡ‚Π° ΠΌΡ‹ ΠΌΠΎΠ΄Π΅Π»ΠΈΡ€ΡƒΠ΅ΠΌ базис с Ρ†Π΅Π½Ρ‚Ρ€ΠΎΠΌ , Ρ‚ΠΎ ΠΌΡ‹ ΠΏΡ€Π΅Π΄ΠΏΠΎΠ»Π°Π³Π°Π΅ΠΌ, Ρ‡Ρ‚ΠΎ наблюдСниС ΠΏΠΎΠ»ΡƒΡ‡Π΅Π½ΠΎ ΠΈΠ· этого базиса. Π’ этом случаС распрСдСлСниС ΠΈΠΌΠ΅Π΅Ρ‚ ΡΠ»Π΅Π΄ΡƒΡŽΡ‰ΠΈΠΉ смысл: насколько вСроятно ΠΏΠΎΠ»ΡƒΡ‡ΠΈΡ‚ΡŒ ΠΈΠ· пространства нашСго ΠΎΠ±ΡŠΠ΅ΠΊΡ‚Π°. Π­Ρ‚Π° Π²Π΅Ρ€ΠΎΡΡ‚Π½ΠΎΡΡ‚ΡŒ ΠΎΠ±Ρ€Π°Ρ‚Π½ΠΎ ΠΏΡ€ΠΎΠΏΠΎΡ€Ρ†ΠΈΠΎΠ½Π°Π»ΡŒΠ½Π° Ρ€Π°ΡΡΡ‚ΠΎΡΠ½ΠΈΡŽ ΠΎΡ‚ наблюдСния Π΄ΠΎ Ρ†Π΅Π½Ρ‚Ρ€Π° . Π­Ρ‚ΠΎ расстояниС ΠΌΠΎΠΆΠ½ΠΎ Ρ€Π°Π·Π»ΠΎΠΆΠΈΡ‚ΡŒ Π½Π° Π΄Π²Π°: расстояниС ΠΎΡ‚ наблюдСния Π΄ΠΎ пространства ΠΈ расстояниС Π²Π½ΡƒΡ‚Ρ€ΠΈ самого пространства ΠΎΡ‚ спроСцированного Π΄ΠΎ Ρ†Π΅Π½Ρ‚Ρ€Π°. Но для Π½Π°ΡˆΠΈΡ… Ρ†Π΅Π»Π΅ΠΉ достаточно Π±ΡƒΠ΄Π΅Ρ‚ ΠΏΠΎΡΡ‡ΠΈΡ‚Π°Ρ‚ΡŒ Ρ‚ΠΎΠ»ΡŒΠΊΠΎ расстояниС, ΠΏΡ€ΠΎΠΏΠΎΡ€Ρ†ΠΈΠΎΠ½Π°Π»ΡŒΠ½ΠΎΠ΅ :

Π’ этом Π²Ρ‹Ρ€Π°ΠΆΠ΅Π½ΠΈΠΈ Π΅ΡΡ‚ΡŒ просто исходноС ΠΈΠ·ΠΎΠ±Ρ€Π°ΠΆΠ΅Π½ΠΈΠ΅ ΠΎΠ±ΡŠΠ΅ΠΊΡ‚Π° Π·Π° Π²Ρ‹Ρ‡Π΅Ρ‚ΠΎΠΌ срСднСго. Π‘Π»Π°Π³Π°Π΅ΠΌΠΎΠ΅ Π΅ΡΡ‚ΡŒ рСконструкция исходного изобраТСния, Ρ‚ΠΎ Π΅ΡΡ‚ΡŒ проСкция изобраТСния Π½Π° базис ΠΈ ΠΎΠ±Ρ€Π°Ρ‚Π½ΠΎ. Π’ΠΈΠ·ΡƒΠ°Π»ΡŒΠ½ΠΎ ΠΏΠΎΡΠΌΠΎΡ‚Ρ€Π΅Ρ‚ΡŒ Π½Π° Ρ‡Ρ‚ΠΎ ΠΏΠΎΡ…ΠΎΠΆΠ° рСконструкция ΠΌΠΎΠΆΠ½ΠΎ Π½Π° рисункС 4 Π² Ρ€Π°ΠΌΠΊΠ΅ ΠΏΠΎΠ΄ Π½Π°Π·Π²Π°Π½ΠΈΠ΅ΠΌ recon. Π’ΠΈΠ΄Π½ΠΎ, Ρ‡Ρ‚ΠΎ исходноС ΠΈΠ·ΠΎΠ±Ρ€Π°ΠΆΠ΅Π½ΠΈΠ΅ нСсколько искаТСно, ΠΈΠ·-Π·Π° ΠΏΠΎΡ‚Π΅Ρ€ΠΈ Π½Π΅ΠΊΠΎΡ‚ΠΎΡ€ΠΎΠΉ ΡΠΎΡΡ‚Π°Π²Π»ΡΡŽΡ‰Π΅ΠΉ ΠΏΡ€ΠΈ ΠΏΡ€ΠΎΠ΅Ρ†ΠΈΡ€ΠΎΠ²Π°Π½ΠΈΠΈ, Π½ΠΎ основной ΠΎΠ±Ρ€Π°Π· сохранСн. Π’ΠΎΠ³Π΄Π° Ρ€Π°Π·Π½ΠΎΡΡ‚ΡŒ , называСмая Ρ‚Π°ΠΊΠΆΠ΅ Π²Π΅ΠΊΡ‚ΠΎΡ€ΠΎΠΌ нСвязки, Π΅ΡΡ‚ΡŒ просто Ρ€Π°Π·Π½ΠΈΡ†Π° ΠΌΠ΅ΠΆΠ΄Ρƒ исходным ΠΈΠ·ΠΎΠ±Ρ€Π°ΠΆΠ΅Π½ΠΈΠ΅ΠΌ ΠΈ Π΅Π³ΠΎ рСконструкциСй, Π° -Π½ΠΎΡ€ΠΌΠ° Π²Π΅ΠΊΡ‚ΠΎΡ€Π° нСвязки Π²Ρ‹Ρ€Π°ΠΆΠ°Π΅Ρ‚ количСство ΠΈΠ½Ρ„ΠΎΡ€ΠΌΠ°Ρ†ΠΈΠΈ, ΠΊΠΎΡ‚ΠΎΡ€ΡƒΡŽ ΠΌΡ‹ Π½Π΅ ΠΌΠΎΠΆΠ΅ΠΌ Π²ΠΎΡΡΡ‚Π°Π½ΠΎΠ²ΠΈΡ‚ΡŒ. Ясно, Ρ‡Ρ‚ΠΎ Ρ‡Π΅ΠΌ большС расстояниС , Ρ‚Π΅ΠΌ Π½ΠΈΠΆΠ΅ Π²Π΅Ρ€ΠΎΡΡ‚Π½ΠΎΡΡ‚ΡŒ ΠΈ Ρ‚Π΅ΠΌ Ρ…ΡƒΠΆΠ΅ Π±ΡƒΠ΄Π΅Ρ‚ качСство восстановлСнного изобраТСния.

Наглядно расстояния ΠΈ ΠΈΠ·ΠΎΠ±Ρ€Π°ΠΆΠ΅Π½Ρ‹ Π½Π° рисункС Π½ΠΈΠΆΠ΅.

Рис. 7. Расстояния Π΄ΠΎ базиса ΠΈ Π²Π½ΡƒΡ‚Ρ€ΠΈ базиса.

Рис. 7. РасстояниС, ΠΏΡ€ΠΎΠΏΠΎΡ€Ρ†ΠΈΠΎΠ½Π°Π»ΡŒΠ½ΠΎΠ΅ . Для наглядности базис Π³Π»Π°Π²Π½Ρ‹Ρ… ΠΊΠΎΠΌΠΏΠΎΠ½Π΅Π½Ρ‚ прСдставлСн Π½Π° плоскости. ΠœΡ‹ΡΠ»Π΅Π½Π½ΠΎ Ρ€Π°ΡΡˆΠΈΡ€ΡΠ΅ΠΌ базис Π΄ΠΎ .

ПолСзной Π½Π°Ρ…ΠΎΠ΄ΠΊΠΎΠΉ Π°Π²Ρ‚ΠΎΡ€ΠΎΠ² оказалось использованиС робастной Ρ„ΡƒΠ½ΠΊΡ†ΠΈΠΈ вмСсто -Π½ΠΎΡ€ΠΌΡ‹ для ΠΌΠΈΠ½ΠΈΠΌΠΈΠ·Π°Ρ†ΠΈΠΈ влияния ΡˆΡƒΠΌΠΎΠ²Ρ‹Ρ… пиксСлСй Π½Π° ΠΎΡ†Π΅Π½ΠΊΡƒ вСроятности . Π”Π΅ΠΉΡΡ‚Π²ΠΈΡ‚Π΅Π»ΡŒΠ½ΠΎ, Ссли Ρ†Π΅Π»Π΅Π²ΠΎΠΉ ΠΎΠ±ΡŠΠ΅ΠΊΡ‚ ΠΈΠΌΠ΅Π΅Ρ‚ ΠΊΡ€ΡƒΠ³Π»ΡƒΡŽ Ρ„ΠΎΡ€ΠΌΡƒ ΠΈ, ΠΏΡ€ΠΈ этом, Π·Π°ΠΊΠ»ΡŽΡ‡Π΅Π½ Π² ΠΏΡ€ΡΠΌΠΎΡƒΠ³ΠΎΠ»ΡŒΠ½ΡƒΡŽ Ρ€Π°ΠΌΠΊΡƒ, Ρ‚ΠΎ ΠΊΡ€Π°Π΅Π²Ρ‹Π΅ пиксСли Π²Π½ΡƒΡ‚Ρ€ΠΈ Ρ€Π°ΠΌΠΊΠΈ, выходящиС Π·Π° ΠΏΠ΅Ρ€ΠΈΠΌΠ΅Ρ‚Ρ€ ΠΊΡ€ΡƒΠ³Π°, Π±ΡƒΠ΄ΡƒΡ‚ явно ΠΏΠΎΠΌΠ΅Ρ…ΠΎΠΉ. Бмысл ΠΏΠ°Ρ€Π°ΠΌΠ΅Ρ‚Ρ€Π° Π² Ρ‚ΠΎΠΌ, Ρ‡Ρ‚ΠΎ ΠΎΠ½ Π·Π°Π΄Π°Π΅Ρ‚ ΠΊΡ€ΠΈΡ‚ΠΈΡ‡Π΅ΡΠΊΡƒΡŽ ΠΎΠ±Π»Π°ΡΡ‚ΡŒ, послС ΠΊΠΎΡ‚ΠΎΡ€ΠΎΠΉ влияниС ΡˆΡƒΠΌΠΎΠ² Π½Π° модСль наблюдСния Π½Π°Ρ‡ΠΈΠ½Π°Π΅Ρ‚ ΡƒΠΌΠ΅Π½ΡŒΡˆΠ°Ρ‚ΡŒΡΡ [2].

БэмплированиС — ΠΊΠ°ΠΊ Π²Ρ‹Ρ‡ΠΈΡΠ»ΠΈΡ‚ΡŒ ΠΏΡ€ΠΎΠΈΠ·Π²ΠΎΠ»ΡŒΠ½ΠΎΠ΅ распрСдСлСниС

ЕдинствСнноС, Π½Π° Ρ‡Π΅ΠΌ ΠΌΡ‹ ΠΏΠΎΠΊΠ° Π½Π΅ заостряли внимания, это Π½Π° Ρ‚ΠΎΠΌ, ΠΊΠ°ΠΊΡƒΡŽ Ρ„ΠΎΡ€ΠΌΡƒ Π΄ΠΎΠ»ΠΆΠ½ΠΎ ΠΈΠΌΠ΅Ρ‚ΡŒ распрСдСлСниС . ΠŸΠΎΠ½Π°Ρ‡Π°Π»Ρƒ это распрСдСлСниС ΠΏΡ€ΠΈΠ±Π»ΠΈΠΆΠ°Π»ΠΈ ΠΎΠ±Ρ‹Ρ‡Π½Ρ‹ΠΌ гауссианом прСдполагая, Ρ‡Ρ‚ΠΎ сущСствуСт Ρ‚ΠΎΠ»ΡŒΠΊΠΎ ΠΎΠ΄Π½Π° Π½Π°ΠΈΠ±ΠΎΠ»Π΅Π΅ вСроятная Ρ‚ΠΎΡ‡ΠΊΠ°, Π² ΠΊΠΎΡ‚ΠΎΡ€ΠΎΠΉ Π΄ΠΎΠ»ΠΆΠ΅Π½ Π½Π°Ρ…ΠΎΠ΄ΠΈΡ‚ΡŒΡΡ ΠΎΠ±ΡŠΠ΅ΠΊΡ‚. ΠŸΡ€Π΅Π΄ΡΠΊΠ°Π·Π°Π½ΠΈΠ΅ мСстополоТСния Ρ‚Π°ΠΊΠΈΠΌ ΠΎΠ±Ρ€Π°Π·ΠΎΠΌ сводилось ΠΊ ΠΎΡ†Π΅Π½ΠΊΠ΅ ΠΏΠ°Ρ€Π°ΠΌΠ΅Ρ‚Ρ€ΠΎΠ² двиТСния Ρ„ΠΈΠ»ΡŒΡ‚Ρ€ΠΎΠΌ Калмана. Однако, нСсмотря Π½Π° Ρ‚ΠΎ, Ρ‡Ρ‚ΠΎ это ΡƒΠ΄ΠΎΠ±Π½ΠΎΠ΅ срСдство модСлирования, Π½Π° ΠΏΡ€Π°ΠΊΡ‚ΠΈΠΊΠ΅ ΠΎΠ½ΠΎ Π½Π΅ всСгда Π°Π΄Π΅ΠΊΠ²Π°Ρ‚Π½ΠΎ описываСт процСсс. Из-Π·Π° наличия слоТного Ρ„ΠΎΠ½Π° ΠΈ нСпрСдсказуСмой Π΄ΠΈΠ½Π°ΠΌΠΈΠΊΠΈ двиТСния ΠΎΠ±ΡŠΠ΅ΠΊΡ‚Π° Π±Ρ‹Π»ΠΎ Π±Ρ‹ ΠΏΡ€Π°Π²ΠΈΠ»ΡŒΠ½Π΅Π΅ Π²Ρ‹Π΄Π²ΠΈΠ³Π°Ρ‚ΡŒ сразу нСсколько Π³ΠΈΠΏΠΎΡ‚Π΅Π· ΠΎ Ρ‚ΠΎΠΌ, Π³Π΄Π΅ ΠΌΠΎΠΆΠ΅Ρ‚ Π½Π°Ρ…ΠΎΠ΄ΠΈΡ‚ΡŒΡΡ ΠΎΠ±ΡŠΠ΅ΠΊΡ‚ ΠΈ ΠΏΡ€ΠΈΠ½ΠΈΠΌΠ°Ρ‚ΡŒ Π½Π°ΠΈΠ±ΠΎΠ»Π΅Π΅ Π²Π΅Ρ€ΠΎΡΡ‚Π½ΡƒΡŽ ΠΈΠ· Π½ΠΈΡ…. ΠŸΠΎΡΡ‚ΠΎΠΌΡƒ ΠΌΡ‹ Π±ΡƒΠ΄Π΅ΠΌ ΡΡ‡ΠΈΡ‚Π°Ρ‚ΡŒ, Ρ‡Ρ‚ΠΎ распрСдСлСниС ΠΈΠΌΠ΅Π΅Ρ‚ нСсколько Π²Π΅Ρ€ΡˆΠΈΠ½.

Рис. 8. АпостСриорноС распространСниС для x-ΠΊΠΎΠΎΡ€Π΄ΠΈΠ½Π°Ρ‚Ρ‹ ΠΏΠΎ дискрСтным отсчСтам t. Π’ΠΈΠ΄Π½ΠΎ, Ρ‡Ρ‚ΠΎ Π² Π½Π°Ρ‡Π°Π»Π΅ модСль ΡƒΠ²Π΅Ρ€Π΅Π½Π° Π² ΠΏΠΎΠ»ΠΎΠΆΠ΅Π½ΠΈΠΈ ΠΎΠ±ΡŠΠ΅ΠΊΡ‚Π° (Π»ΠΈΡ†ΠΎ Ρ‡Π΅Π»ΠΎΠ²Π΅ΠΊΠ° Π² красной Ρ€Π°ΠΌΠΊΠ΅), Π½ΠΎ ΠΏΠΎ ΠΌΠ΅Ρ€Π΅ двиТСния Π½Π°Ρ‡ΠΈΠ½Π°ΡŽΡ‚ ΠΎΠ±Ρ€Π°Π·ΠΎΠ²Ρ‹Π²Π°Ρ‚ΡŒΡΡ нСсколько Π³ΠΈΠΏΠΎΡ‚Π΅Π·. НапримСр, Π½Π° ΠΎΡ‚ΠΌΠ΅Ρ‚ΠΊΠ΅ t=20 Π½Π° Π»Π΅Π²ΠΎΠΌ Π³Ρ€Π°Ρ„ΠΈΠΊΠ΅ Π²ΠΈΠ΄Π½ΠΎ Ρ‚Ρ€ΠΈ Π²Π΅Ρ€ΡˆΠΈΠ½Ρ‹. Бамая лСвая Π²Π΅Ρ€ΡˆΠΈΠ½Π° соотвСтствуСт Ρ†Π΅Π»ΠΈ, Π²Π΅Ρ€ΡˆΠΈΠ½Π° посСрСдинС соотвСтствуСт ΠΏΠΎΡ…ΠΎΠΆΠ΅ΠΌΡƒ Ρ‡Π΅Π»ΠΎΠ²Π΅ΠΊΡƒ справа ΠΎΡ‚ Ρ†Π΅Π»ΠΈ ΠΈ правая Π²Π΅Ρ€ΡˆΠΈΠ½Π° соотвСтствуСт Ρ‡Π΅Π»ΠΎΠ²Π΅ΠΊΡƒ Π² синСй ΠΆΠΈΠ»Π΅Ρ‚ΠΊΠ΅ справа. Π’Π°ΠΊ ΠΊΠ°ΠΊ Ρ‡Π΅Π»ΠΎΠ²Π΅ΠΊ справа находится Π΄Π°Π»Π΅ΠΊΠΎ ΠΎΡ‚ Ρ†Π΅Π»ΠΈ, Ρ‚ΠΎ модСль справСдливо Π΄Π°Π΅Ρ‚ Π΅ΠΌΡƒ наимСньший вСс полагая, Ρ‡Ρ‚ΠΎ Ρ†Π΅Π»ΡŒ Π½Π΅ смоТСт Ρ‚Π°ΠΊ Ρ€Π΅Π·ΠΊΠΎ ΡΠ΄Π²ΠΈΠ½ΡƒΡ‚ΡŒΡΡ Π²ΠΏΡ€Π°Π²ΠΎ.

ΠŸΠΎΡΠΊΠΎΠ»ΡŒΠΊΡƒ Ρ‚Π΅ΠΏΠ΅Ρ€ΡŒ распрСдСлСниС ΠΈΠΌΠ΅Π΅Ρ‚ ΠΏΡ€ΠΎΠΈΠ·Π²ΠΎΠ»ΡŒΠ½ΡƒΡŽ Ρ„ΠΎΡ€ΠΌΡƒ, ΠΎΡ‚Π»ΠΈΡ‡Π½ΡƒΡŽ ΠΎΡ‚ гауссиана, ΠΌΡ‹ тСряСм Π²ΠΎΠ·ΠΌΠΎΠΆΠ½ΠΎΡΡ‚ΡŒ Π²Ρ‹Ρ‡ΠΈΡΠ»ΠΈΡ‚ΡŒ Π΅Π³ΠΎ аналитичСски. ΠŸΠΎΡΡ‚ΠΎΠΌΡƒ ΠΌΡ‹ ΠΏΡ€ΠΈΠΌΠ΅Π½ΠΈΠΌ Ρ‚Π΅Ρ…Π½ΠΈΠΊΡƒ Ρ„ΠΈΠ»ΡŒΡ‚Ρ€Π° частиц (particle filter), ΠΏΠΎΠ·Π²ΠΎΠ»ΡΡŽΡ‰ΡƒΡŽ ΠΎΡ†Π΅Π½ΠΈΡ‚ΡŒ ΠΏΠ°Ρ€Π°ΠΌΠ΅Ρ‚Ρ€Ρ‹ искомого ΠΏΡ€ΠΎΠΈΠ·Π²ΠΎΠ»ΡŒΠ½ΠΎΠ³ΠΎ распрСдСлСния. МоТно ΡΡ‡ΠΈΡ‚Π°Ρ‚ΡŒ, Ρ‡Ρ‚ΠΎ это ΠΎΠ±ΠΎΠ±Ρ‰Π΅Π½ΠΈΠ΅ Ρ„ΠΈΠ»ΡŒΡ‚Ρ€Π° Калмана Π½Π° случай нСгауссовских процСссов. Под частицСй понимаСтся элСмСнт ΠΈΠ· Π²Ρ‹Π±ΠΎΡ€ΠΎΡ‡Π½ΠΎΠΉ совокупности распрСдСлСния с вСсом, ΠΏΡ€ΠΎΠΏΠΎΡ€Ρ†ΠΈΠΎΠ½Π°Π»ΡŒΠ½Ρ‹ΠΌ вСроятности ΠΏΠΎΠ»ΡƒΡ‡ΠΈΡ‚ΡŒ эту частицу ΠΈΠ· распрСдСлСния. На рисункС 8 частицСй являСтся каТдая Ρ‚Π΅ΠΌΠ½ΠΎ-синяя Ρ‚ΠΎΡ‡ΠΊΠ° Π½Π° Π»Π΅Π²ΠΎΠΌ Π³Ρ€Π°Ρ„ΠΈΠΊΠ΅. Если ΡƒΡΡ‚Ρ€Π΅ΠΌΠΈΡ‚ΡŒ количСство частиц Π² Π±Π΅ΡΠΊΠΎΠ½Π΅Ρ‡Π½ΠΎΡΡ‚ΡŒ, Ρ‚ΠΎ β€œΡ€Π²Π°Π½Ρ‹ΠΉβ€ Π³Ρ€Π°Ρ„ΠΈΠΊ Π±ΡƒΠ΄Π΅Ρ‚ ΡΡ‚Π°Π½ΠΎΠ²ΠΈΡ‚ΡŒΡΡ Π±ΠΎΠ»Π΅Π΅ Π³Π»Π°Π΄ΠΊΠΈΠΌ, Π° Π² ΠΏΡ€Π΅Π΄Π΅Π»Π΅ станСт Π½Π΅ΠΏΡ€Π΅Ρ€Ρ‹Π²Π½Ρ‹ΠΌ. Π—Π°ΠΌΠ΅Ρ‚Π½ΠΎ, Ρ‡Ρ‚ΠΎ частица с наибольшим вСсом соотвСтствуСт Π½Π°ΠΈΠ±ΠΎΠ»Π΅Π΅ вСроятному ΠΌΠ΅ΡΡ‚ΠΎΠΏΠΎΠ»ΠΎΠΆΠ΅Π½ΠΈΡŽ ΠΎΠ±ΡŠΠ΅ΠΊΡ‚Π°. Для Π³Π΅Π½Π΅Ρ€Π°Ρ†ΠΈΠΈ ΠΈ распространСния частиц Π²ΠΎ Π²Ρ€Π΅ΠΌΠ΅Π½ΠΈ ΠΌΡ‹ Π²ΠΎΡΠΏΠΎΠ»ΡŒΠ·ΡƒΠ΅ΠΌΡΡ Π°Π»Π³ΠΎΡ€ΠΈΡ‚ΠΌΠΎΠΌ сэмплирования CONDENSATION [3].

Алгоритм CONDENSATION ΠΈΡΠΏΠΎΠ»ΡŒΠ·ΡƒΠ΅Ρ‚ Ρ‚Π΅Ρ…Π½ΠΈΠΊΡƒ сэмплирования с ΡƒΡ‡Π΅Ρ‚ΠΎΠΌ Π΄ΠΈΠ½Π°ΠΌΠΈΠΊΠΈ двиТСния ΠΎΠ±ΡŠΠ΅ΠΊΡ‚Π° для Π³Π΅Π½Π΅Ρ€Π°Ρ†ΠΈΠΈ Π²Ρ‹Π±ΠΎΡ€ΠΊΠΈ ΠΈΠ· частиц подчиняСмых Ρ€Π°ΡΠΏΡ€Π΅Π΄Π΅Π»Π΅Π½ΠΈΡŽ . Π”Π°Π½Π½Ρ‹ΠΉ Π°Π»Π³ΠΎΡ€ΠΈΡ‚ΠΌ ΠΈΡ‚Π΅Ρ€Π°Ρ‚ΠΈΠ²Π½Ρ‹ΠΉ. Π­Ρ‚ΠΎ Π·Π½Π°Ρ‡ΠΈΡ‚, Ρ‡Ρ‚ΠΎ ΠΏΠΎΠ»ΡƒΡ‡ΠΈΠ² Π²Ρ‹Π±ΠΎΡ€ΠΊΡƒ ΠΈΠ· апостСриорного распрСдСлСния ΠΌΡ‹ ΠΌΠΎΠΆΠ΅ΠΌ Ρ€Π°ΡΠΏΡ€ΠΎΡΡ‚Ρ€Π°Π½ΠΈΡ‚ΡŒ эту Π²Ρ‹Π±ΠΎΡ€ΠΊΡƒ для вычислСния Π½ΠΎΠ²ΠΎΠ³ΠΎ апостСриорного распрСдСлСния Π½Π° ΡΠ»Π΅Π΄ΡƒΡŽΡ‰Π΅ΠΌ шагС ΠΈΡΠΏΠΎΠ»ΡŒΠ·ΡƒΡ ΠΏΡ€Π΅Π΄Ρ‹Π΄ΡƒΡ‰Π΅Π΅ распрСдСлСниС Π² качСствС Π°ΠΏΡ€ΠΈΠΎΡ€Π½ΠΎΠ³ΠΎ. Π­Ρ‚Π°ΠΏ распространСния ΠΏΡ€ΠΈ этом ΠΏΠ΅Ρ€Π΅ΠΆΠΈΠ²Π°ΡŽΡ‚ Ρ‚ΠΎΠ»ΡŒΠΊΠΎ частицы с наибольшим вСсом, Π² Ρ‚ΠΎ врСмя ΠΊΠ°ΠΊ маловСроятныС частицы ΠΎΡ‚ΡΠ΅ΠΈΠ²Π°ΡŽΡ‚ΡΡ. Π’Ρ‹Ρ‡ΠΈΡΠ»ΠΈΡ‚Π΅Π»ΡŒΠ½Π°Ρ ΡΠ»ΠΎΠΆΠ½ΠΎΡΡ‚ΡŒ Π°Π»Π³ΠΎΡ€ΠΈΡ‚ΠΌΠ° оцСниваСтся ΠΊΠ°ΠΊ .

Π£ΠΏΡ€ΠΎΡ‰Π΅Π½Π½ΠΎ, схСму CONDENSATION ΠΌΠΎΠΆΠ½ΠΎ ΠΏΡ€Π΅Π΄ΡΡ‚Π°Π²ΠΈΡ‚ΡŒ Π² ΡΠ»Π΅Π΄ΡƒΡŽΡ‰Π΅ΠΌ Π²ΠΈΠ΄Π΅:

  1. Π“Π΅Π½Π΅Ρ€ΠΈΡ€ΡƒΠ΅ΠΌ ΡΠ»ΡƒΡ‡Π°ΠΉΠ½ΡƒΡŽ Π²Ρ‹Π±ΠΎΡ€ΠΊΡƒ ΠΈΠ· распрСдСлСния .

  2. ΠšΠ°ΠΆΠ΄ΠΎΠΌΡƒ элСмСнту Π²Ρ‹Π±ΠΎΡ€ΠΊΠΈ присваиваСм вСс , Ρ€Π°Π²Π½Ρ‹ΠΉ .

  3. НормализуСм Π²Ρ‹Π±ΠΎΡ€ΠΊΡƒ для удовлСтворСния условия .

  4. ВычисляСм Π½Π΅ΠΎΠ±Ρ…ΠΎΠ΄ΠΈΠΌΡ‹Π΅ статистики, Π½Π°ΠΏΡ€ΠΈΠΌΠ΅Ρ€

    Нам интСрСсСн послСдний случай. Π”Π°Π½Π½Ρ‹Π΅ статистики асимптотичСски нСсмСщСнныС.

  5. ΠŸΠ΅Ρ€Π΅Ρ…ΠΎΠ΄ΠΈΠΌ ΠΊ ΡΠ»Π΅Π΄ΡƒΡŽΡ‰Π΅ΠΌΡƒ Ρ‚Π°ΠΊΡ‚Ρƒ ΠΈ повторяСм ΠΏΡ€ΠΎΡ†Π΅Π΄ΡƒΡ€Ρƒ с ΠΏΠ΅Ρ€Π²ΠΎΠ³ΠΎ шага принимая Π·Π° (см. ΠΏΠ΅Ρ€Π²Ρ‹ΠΉ шаг) Π²Ρ‹Π±ΠΎΡ€ΠΊΡƒ .

Π’Π°ΠΊΠΈΠΌ ΠΎΠ±Ρ€Π°Π·ΠΎΠΌ, нСсмотря Π½Π° Ρ‚ΠΎ, Ρ‡Ρ‚ΠΎ ΠΌΡ‹ Π½Π΅ Π² состоянии числСнно Π²Ρ‹Ρ€Π°Π·ΠΈΡ‚ΡŒ распрСдСлСниС , ΠΌΡ‹ ΠΌΠΎΠΆΠ΅ΠΌ Π½Π°ΠΉΡ‚ΠΈ Π½Π΅ΠΎΠ±Ρ…ΠΎΠ΄ΠΈΠΌΡ‹Π΅ статистики этого распрСдСлСния Ρ‡Π΅Ρ€Π΅Π· Π²Ρ‹Π±ΠΎΡ€ΠΎΡ‡Π½ΡƒΡŽ ΡΠΎΠ²ΠΎΠΊΡƒΠΏΠ½ΠΎΡΡ‚ΡŒ . А для Ρ€Π΅ΡˆΠ΅Π½ΠΈΡ нашСй Π·Π°Π΄Π°Ρ‡ΠΈ этого достаточно.

БоСдиняСм всС вмСстС

НаконСц, ΠΌΡ‹ Π²Π»Π°Π΄Π΅Π΅ΠΌ всСй Π½Π΅ΠΎΠ±Ρ…ΠΎΠ΄ΠΈΠΌΠΎΠΉ ΠΈΠ½Ρ„ΠΎΡ€ΠΌΠ°Ρ†ΠΈΠ΅ΠΉ ΠΈ ΠΌΠΎΠΆΠ΅ΠΌ Π·Π°ΠΏΠΈΡΠ°Ρ‚ΡŒ ΠΏΠΎΠ»Π½ΡƒΡŽ схСму Ρ€Π°Π±ΠΎΡ‚Ρ‹ Ρ‚Ρ€Π΅ΠΊΠ΅Ρ€Π°:

  1. Π˜ΡΠΏΠΎΠ»ΡŒΠ·ΡƒΡ Π΄Π΅Ρ‚Π΅ΠΊΡ‚ΠΎΡ€ ΠΎΠ±ΡŠΠ΅ΠΊΡ‚ΠΎΠ² (нСйросСтСвой ΠΈΠ»ΠΈ статистичСский, Π½Π΅ ΠΈΠΌΠ΅Π΅Ρ‚ значСния) опрСдСляСм мСстополоТСниС ΠΎΠ±ΡŠΠ΅ΠΊΡ‚Π° Π½Π° Π½Π°Ρ‡Π°Π»ΡŒΠ½ΠΎΠΌ ΠΊΠ°Π΄Ρ€Π΅ . Π—Π°Π΄Π°Π΅ΠΌ всСм частицам Π·Π½Π°Ρ‡Π΅Π½ΠΈΠ΅ Π½Π°Ρ‡Π°Π»ΡŒΠ½ΠΎΠΉ ΠΏΠΎΠ·ΠΈΡ†ΠΈΠΈ ΠΎΠ±ΡŠΠ΅ΠΊΡ‚Π° с Ρ€Π°Π²Π½Ρ‹ΠΌ вСсом .

  2. Π—Π°Π΄Π°Π΅ΠΌ пустой базис со срСдним Π·Π½Π°Ρ‡Π΅Π½ΠΈΠ΅ΠΌ Ρ€Π°Π²Π½Ρ‹ΠΌ ΠΈΠ·ΠΎΠ±Ρ€Π°ΠΆΠ΅Π½ΠΈΡŽ ΠΎΠ±ΡŠΠ΅ΠΊΡ‚Π° Π½Π° Π½Π°Ρ‡Π°Π»ΡŒΠ½ΠΎΠΌ ΠΊΠ°Π΄Ρ€Π΅.

  3. ДвигаСмся ΠΊ ΡΠ»Π΅Π΄ΡƒΡŽΡ‰Π΅ΠΌΡƒ ΠΊΠ°Π΄Ρ€Ρƒ . Π“Π΅Π½Π΅Ρ€ΠΈΡ€ΡƒΠ΅ΠΌ Π½ΠΎΠ²Ρ‹Π΅ частицы/Π²ΠΎΠ·ΠΌΠΎΠΆΠ½Ρ‹Π΅ мСстополоТСния Π² соотвСтствии с динамичСской модСлью .

  4. Для ΠΊΠ°ΠΆΠ΄ΠΎΠΉ частицы ΠΈΠ·Π²Π»Π΅ΠΊΠ°Π΅ΠΌ ΡΠΎΠΎΡ‚Π²Π΅Ρ‚ΡΡ‚Π²ΡƒΡŽΡ‰ΠΈΠΉ ΠΊΡ€ΠΎΠΏ ΠΈΠ· Ρ‚Π΅ΠΊΡƒΡ‰Π΅Π³ΠΎ ΠΊΠ°Π΄Ρ€Π° ΠΈ вычисляСм Π΅Π΅ вСс Π² соотвСтствии с модСлью наблюдСния .

  5. БохраняСм частицу с наибольшим вСсом. ПослС Ρ‚ΠΎΠ³ΠΎ, ΠΊΠ°ΠΊ Π±ΡƒΠ΄Π΅Ρ‚ Π½Π°ΠΊΠΎΠΏΠ»Π΅Π½ΠΎ ΠΊΠ°Π΄Ρ€ΠΎΠ² осущСствляСм ΠΈΠ½ΠΊΡ€Π΅ΠΌΠ΅Π½Ρ‚Π½ΠΎΠ΅ ΠΎΠ±ΡƒΡ‡Π΅Π½ΠΈΠ΅ базиса ΠΈ срСднСго .

  6. ВозвращаСмся ΠΊ ΡˆΠ°Π³Ρƒ 3.

ΠžΡΠΎΠ±Π΅Π½Π½ΠΎΡΡ‚ΠΈ Ρ€Π΅Π°Π»ΠΈΠ·Π°Ρ†ΠΈΠΈ ΠΈ экспСримСнты

Для тСстирования Ρ‚Ρ€Π΅ΠΊΠ΅Ρ€Π° Π±Ρ‹Π»Π° Ρ€Π°Π·Ρ€Π°Π±ΠΎΡ‚Π°Π½Π° Π΄Π΅ΠΌΠΊΠ° Π½Π° C++, Π² основС ΠΊΠΎΡ‚ΠΎΡ€ΠΎΠΉ Π»Π΅ΠΆΠΈΡ‚ ΠΊΠΎΠ΄ Matlab ΠΎΡ‚ Π°Π²Ρ‚ΠΎΡ€ΠΎΠ² ΠΎΡ€ΠΈΠ³ΠΈΠ½Π°Π»ΡŒΠ½ΠΎΠΉ ΡΡ‚Π°Ρ‚ΡŒΠΈ.

Π’ Ρ€Π΅Π°Π»ΡŒΠ½Ρ‹Ρ… Π·Π°Π΄Π°Ρ‡Π°Ρ… ΠΌΡ‹ сталкиваСмся с физичСскими ограничСниями. Π’ΠΎ-ΠΏΠ΅Ρ€Π²Ρ‹Ρ…, ΠΌΡ‹ ΠΎΠ³Ρ€Π°Π½ΠΈΡ‡Π΅Π½Ρ‹ ΠΆΠ΅Π»Π΅Π·ΠΎΠΌ, Π²Π²ΠΈΠ΄Ρƒ Ρ‡Π΅Π³ΠΎ ΠΌΡ‹ Π½Π΅ ΠΌΠΎΠΆΠ΅ΠΌ Π°ΠΏΠΏΡ€ΠΎΠΊΡΠΈΠΌΠΈΡ€ΠΎΠ²Π°Ρ‚ΡŒ распрСдСлСниС с любой Ρ‚ΠΎΡ‡Π½ΠΎΡΡ‚ΡŒΡŽ ΠΈ Π²Ρ‹Π½ΡƒΠΆΠ΄Π΅Π½Ρ‹ ΠΎΠ³Ρ€Π°Π½ΠΈΡ‡ΠΈΠ²Π°Ρ‚ΡŒ количСство Π³Π΅Π½Π΅Ρ€ΠΈΡ€ΡƒΠ΅ΠΌΡ‹Ρ… частиц . Π•Ρ‰Π΅ ΠΎΠ΄Π½ΠΎ ΠΎΠ³Ρ€Π°Π½ΠΈΡ‡Π΅Π½ΠΈΠ΅ связано с Ρ‚Π΅ΠΌ, Ρ‡Ρ‚ΠΎ ΠΏΡ€ΠΈ ΠΏΠ΅Ρ€Π΅Ρ…ΠΎΠ΄Π΅ ΠΊΠΎΠΎΡ€Π΄ΠΈΠ½Π°Ρ‚Ρ‹ Π½Π΅ΠΊΠΎΡ‚ΠΎΡ€Ρ‹Ρ… частиц ΠΌΠΎΠ³ΡƒΡ‚ β€œΠ²Ρ‹Π»Π΅Ρ‚Π΅Ρ‚ΡŒβ€ Π·Π° Ρ€Π°ΠΌΠΊΠΈ ΠΊΠ°Π΄Ρ€Π°. ΠœΡ‹ Π½Π΅ Π±ΡƒΠ΄Π΅ΠΌ ΡƒΡ‚Ρ€ΡƒΠΆΠ΄Π°Ρ‚ΡŒ сСбя ΠΎΡ‚Π΄Π΅Π»ΡŒΠ½ΠΎΠΉ ΠΎΠ±Ρ€Π°Π±ΠΎΡ‚ΠΊΠΎΠΉ Ρ‚Π°ΠΊΠΈΡ… случаСв, Π° Π±ΡƒΠ΄Π΅ΠΌ просто Π²ΠΎΠ·Π²ΠΎΠ΄ΠΈΡ‚ΡŒ Ρ‚Π°ΠΊΠΈΠ΅ частицы Π² Π½ΡƒΠ»Π΅Π²ΠΎΠΉ Π²Π΅ΠΊΡ‚ΠΎΡ€, Ρ‡Ρ‚ΠΎ эквивалСнтно ΠΎΠ±Π½ΡƒΠ»Π΅Π½ΠΈΡŽ вСса частицы.

Для сокращСния Π²Ρ€Π΅ΠΌΠ΅Π½ΠΈ вычислСний ΠΌΡ‹ Π½Π΅ Π±ΡƒΠ΄Π΅ΠΌ ΠΈΡΠΏΠΎΠ»ΡŒΠ·ΠΎΠ²Π°Ρ‚ΡŒ ΠΏΠ°Ρ€Π°ΠΌΠ΅Ρ‚Ρ€Ρ‹ ΠΏΠΎΠ²ΠΎΡ€ΠΎΡ‚Π° ΠΈ сдвига для состояния ΠΈ оставим Ρ‚ΠΎΠ»ΡŒΠΊΠΎ Ρ‡Π΅Ρ‚Ρ‹Ρ€Π΅ ΠΏΠ°Ρ€Π°ΠΌΠ΅Ρ‚Ρ€Π° : ΠΊΠΎΠΎΡ€Π΄ΠΈΠ½Π°Ρ‚Ρ‹ Ρ†Π΅Π½Ρ‚Ρ€Π°, ΠΌΠ°ΡΡˆΡ‚Π°Π± ΠΈ ΡΠΎΠΎΡ‚Π½ΠΎΡˆΠ΅Π½ΠΈΠ΅ сторон; с ΠΈΡ… ΠΏΠΎΠΌΠΎΡ‰ΡŒΡŽ ΠΌΠΎΠΆΠ½ΠΎ Π·Π°Π΄Π°Ρ‚ΡŒ любой Π½Π΅ΠΏΠΎΠ²ΠΎΡ€ΠΎΡ‚Π½Ρ‹ΠΉ ΠΏΡ€ΡΠΌΠΎΡƒΠ³ΠΎΠ»ΡŒΠ½ΠΈΠΊ.

ИмСя Π² качСствС Π½Π°Ρ‡Π°Π»ΡŒΠ½ΠΎΠΉ ΠΊΠΎΠ½Ρ„ΠΈΠ³ΡƒΡ€Π°Ρ†ΠΈΠΈ Ρ‚Ρ€Π΅ΠΊΠ΅Ρ€Π° Ρ€Π°Π·ΠΌΠ΅Ρ€ ΠΎΠΊΠ½Π° 32 Π½Π° 32 ΠΌΡ‹ ΠΏΠΎΠ»ΡƒΡ‡Π°Π΅ΠΌ ΠΎΡ‚ 125 Π΄ΠΎ 8 ΠΊΠ°Π΄Ρ€ΠΎΠ² Π² сСкунду Π½Π° ΡΡ‚Π°Ρ€Π΅Π½ΡŒΠΊΠΎΠΌ Intel Core i7 4700HQ 2.4 ΠœΠ³Ρ† ΠΏΡ€ΠΈ количСствС частиц ΠΎΡ‚ 100 Π΄ΠΎ 1000 соотвСтствСнно. Π”Ρ€ΡƒΠ³ΠΈΠ΅ ΠΏΠ°Ρ€Π°ΠΌΠ΅Ρ‚Ρ€Ρ‹ Π½Π΅ сильно ΡΠΊΠ°Π·Ρ‹Π²Π°ΡŽΡ‚ΡΡ Π½Π° ΠΏΡ€ΠΎΠΈΠ·Π²ΠΎΠ΄ΠΈΡ‚Π΅Π»ΡŒΠ½ΠΎΡΡ‚ΠΈ. ΠŸΠΎΠ΄Ρ€ΠΎΠ±Π½Ρ‹ΠΉ Ρ‚Ρ€Π΅Π½Π΄ прСдставлСн Π½Π° рисунках Π½ΠΈΠΆΠ΅.

Рис. 9. Показания срСднСго Π²Ρ€Π΅ΠΌΠ΅Π½ΠΈ вычислСния.

На Π²ΠΈΠ΄Π΅ΠΎ Π½ΠΈΠΆΠ΅ ΠΏΡ€ΠΈΠ²Π΅Π΄Π΅Π½Ρ‹ Π½Π΅ΠΊΠΎΡ‚ΠΎΡ€Ρ‹Π΅ ΠΏΡ€ΠΈΠΌΠ΅Ρ€Ρ‹ Ρ€Π°Π±ΠΎΡ‚Ρ‹ Ρ‚Ρ€Π΅ΠΊΠ΅Ρ€Π° Π½Π° сцСнах Ρ€Π°Π·Π»ΠΈΡ‡Π½ΠΎΠΉ слоТности, ΠΊΠ°ΠΊ ΡƒΠ΄Π°Ρ‡Π½Ρ‹Π΅ Ρ‚Π°ΠΊ ΠΈ Π½Π΅ΡƒΠ΄Π°Ρ‡Π½Ρ‹Π΅.

Π‘Ρ‚ΠΎΠΈΡ‚ ΠΎΡ‚ΠΌΠ΅Ρ‚ΠΈΡ‚ΡŒ, Ρ‡Ρ‚ΠΎ, ΠΊΠΎΠ½Π΅Ρ‡Π½ΠΎ, Π²Π΅ΠΊΡ‚ΠΎΡ€Ρ‹ коэффициСнтов Π² базисС Π³Π»Π°Π²Π½Ρ‹Ρ… ΠΊΠΎΠΌΠΏΠΎΠ½Π΅Π½Ρ‚ Π½Π΅ ΡΠ²Π»ΡΡŽΡ‚ΡΡ Π½Π°ΡΡ‚ΠΎΠ»ΡŒΠΊΠΎ Π²Ρ‹Ρ€Π°Π·ΠΈΡ‚Π΅Π»ΡŒΠ½Ρ‹ΠΌΠΈ ΠΏΡ€ΠΈΠ·Π½Π°ΠΊΠ°ΠΌΠΈ ΠΊΠ°ΠΊ ΠΏΡ€ΠΈΠ·Π½Π°ΠΊΠΈ аппроксимированныС Π³Π»ΡƒΠ±ΠΎΠΊΠΎΠΉ Π½Π΅ΠΉΡ€ΠΎΡΠ΅Ρ‚ΡŒΡŽ. ΠŸΡ€ΠΈ тСстировании Ρ‚Ρ€Π΅ΠΊΠ΅Ρ€Π° это Π΄Π°Π΅Ρ‚ ΠΎ сСбС Π·Π½Π°Ρ‚ΡŒ. НапримСр, Π½Π΅Ρ€Π΅Π΄ΠΊΠΎ Π²ΠΎΠ·Π½ΠΈΠΊΠ°Π΅Ρ‚ ситуация ΠΊΠΎΠ³Π΄Π° состояниС ΠΌΠΎΠΆΠ΅Ρ‚ β€œΠΏΠ΅Ρ€Π΅ΡΠΊΠΎΡ‡ΠΈΡ‚ΡŒβ€ Π½Π° Π΄Ρ€ΡƒΠ³ΠΎΠΉ ΠΎΠ±ΡŠΠ΅ΠΊΡ‚, посчитав Π΅Π³ΠΎ Π·Π° Ρ†Π΅Π»Π΅Π²ΠΎΠΉ (см. отслСТиваниС сСвСрного олСня Π½Π° Π²ΠΈΠ΄Π΅ΠΎ Π²Ρ‹ΡˆΠ΅). Другая ΠΏΡ€ΠΎΠ±Π»Π΅ΠΌΠ° связана с Ρ‚Π΅ΠΌ, Ρ‡Ρ‚ΠΎ ΠΎΠ±ΡŠΠ΅ΠΊΡ‚Ρ‹ с ΠΏΡ€ΠΈΠΌΠΈΡ‚ΠΈΠ²Π½ΠΎΠΉ тСкстурой, Π±Π΅Π· ярко Π²Ρ‹Ρ€Π°ΠΆΠ΅Π½Π½Ρ‹Ρ… Π²ΠΈΠ·ΡƒΠ°Π»ΡŒΠ½Ρ‹Ρ… ΠΏΡ€ΠΈΠ·Π½Π°ΠΊΠΎΠ², ΠΏΠ»ΠΎΡ…ΠΎ ΠΌΠΎΠ΄Π΅Π»ΠΈΡ€ΡƒΡŽΡ‚ΡΡ базисом. БлСдствиСм этого являСтся Ρ‚ΠΎ, Ρ‡Ρ‚ΠΎ модСль наблюдСния Π½Π΅ ΠΌΠΎΠΆΠ΅Ρ‚ Π΄Π°Ρ‚ΡŒ особого прСдпочтСния для ΠΊΠ°ΠΊΠΎΠΉ-Ρ‚ΠΎ ΠΈΠ· Π³ΠΈΠΏΠΎΡ‚Π΅Π· ΠΈ состояниС Π½Π΅ ΠΌΠΎΠΆΠ΅Ρ‚ Π·Π°Ρ†Π΅ΠΏΠΈΡ‚ΡŒΡΡ Π·Π° ΠΊΠΎΠ½ΠΊΡ€Π΅Ρ‚Π½Ρ‹ΠΉ ΠΎΠ±ΡŠΠ΅ΠΊΡ‚, Π° Π½Π°Ρ‡ΠΈΠ½Π°Π΅Ρ‚ случайно Π±Π»ΡƒΠΆΠ΄Π°Ρ‚ΡŒ ΠΏΠΎ сцСнС (см. отслСТиваниС УсСйна Π‘ΠΎΠ»Ρ‚Π° Π½Π° Π²ΠΈΠ΄Π΅ΠΎ Π²Ρ‹ΡˆΠ΅).

Π’Π°ΠΊ ΠΆΠ΅, Ρ‡Ρ‚ΠΎ Ρ…Π°Ρ€Π°ΠΊΡ‚Π΅Ρ€Π½ΠΎ для всСх Π°Π½Π°Π»ΠΎΠ³ΠΈΡ‡Π½Ρ‹Ρ… Ρ‚Ρ€Π΅ΠΊΠ΅Ρ€ΠΎΠ² Ρ‚ΠΎΠ³ΠΎ Π²Ρ€Π΅ΠΌΠ΅Π½ΠΈ, для Ρ€Π°Π±ΠΎΡ‚Ρ‹ Π² ΠΊΠΎΠ½ΠΊΡ€Π΅Ρ‚Π½Ρ‹Ρ… условиях Π΅Π³ΠΎ Π½ΡƒΠΆΠ½ΠΎ Π½Π°ΡΡ‚Ρ€Π°ΠΈΠ²Π°Ρ‚ΡŒ. Но ΠΏΠΎ ΠΎΠΏΡ‹Ρ‚Ρƒ ΠΌΠΎΠΆΠ½ΠΎ ΡΠΊΠ°Π·Π°Ρ‚ΡŒ, Ρ‡Ρ‚ΠΎ настройки ΠΏΠΎ ΡƒΠΌΠΎΠ»Ρ‡Π°Π½ΠΈΡŽ (ΠΈΡ… ΠΌΠΎΠΆΠ½ΠΎ ΠΏΠΎΠ΄ΡΠΌΠΎΡ‚Ρ€Π΅Ρ‚ΡŒ Π² Ρ€Π΅ΠΏΠΎΠ·ΠΈΡ‚ΠΎΡ€ΠΈΠΈ) ΠΏΠΎΠΊΡ€Ρ‹Π²Π°ΡŽΡ‚ Π±Γ³Π»ΡŒΡˆΡƒΡŽ Ρ‡Π°ΡΡ‚ΡŒ сцСнариСв ΠΈ всю настройку ΠΌΠΎΠΆΠ½ΠΎ свСсти ΠΊ ΠΏΠΎΠ΄Π³ΠΎΠ½ΠΊΠ΅ ΠΌΠΎΠ΄Π΅Π»ΠΈ двиТСния.

Подводя практичСский ΠΈΡ‚ΠΎΠ³ ΠΌΠΎΠΆΠ½ΠΎ ΡΠΊΠ°Π·Π°Ρ‚ΡŒ, Ρ‡Ρ‚ΠΎ Ρ‚Ρ€Π΅ΠΊΠ΅Ρ€ Ρ…ΠΎΡ€ΠΎΡˆΠΎ справляСтся с отслСТиваниСм ΠΎΠ±ΡŠΠ΅ΠΊΡ‚ΠΎΠ² слоТной тСкстуры Π½Π° краткосрочных ΠΈ срСднСсрочных Ρ‚Ρ€Π΅ΠΊΠ°Ρ…, Ρ‚Π°ΠΊΠΈΡ… ΠΊΠ°ΠΊ Π»ΠΈΡ†Π°, транспортныС срСдства. ΠŸΡ€ΠΈ наблюдСнии Π·Π° ΠΎΠ±ΡŠΠ΅ΠΊΡ‚ΠΎΠΌ Π½Π° Π΄Π»ΠΈΠ½Π½Ρ‹Ρ… Π²Ρ€Π΅ΠΌΠ΅Π½Π½Ρ‹Ρ… дистанциях Π²Π΅Π»ΠΈΠΊ риск ΡƒΠΏΡƒΡΡ‚ΠΈΡ‚ΡŒ истинноС ΠΏΠΎΠ»ΠΎΠΆΠ΅Π½ΠΈΠ΅ ΠΎΠ±ΡŠΠ΅ΠΊΡ‚Π° ΠΈ Π½Π°Ρ‡Π°Ρ‚ΡŒ ΠΎΡ‚ΡΠ»Π΅ΠΆΠΈΠ²Π°Ρ‚ΡŒ Π½Π΅ Ρ‚ΠΎ, Ρ‡Ρ‚ΠΎ Π½ΡƒΠΆΠ½ΠΎ. ΠŸΡ€ΠΈ частичном ΠΏΠ΅Ρ€Π΅ΠΊΡ€Ρ‹Ρ‚ΠΈΠΈ ΠΎΠ±ΡŠΠ΅ΠΊΡ‚Π° Ρ‚Ρ€Π΅ΠΊΠ΅Ρ€ способСн ΡƒΠ΄Π΅Ρ€ΠΆΠΈΠ²Π°Ρ‚ΡŒ ΠΏΠΎΠ·ΠΈΡ†ΠΈΡŽ, Π½ΠΎ ΠΏΡ€ΠΈ сильном ΠΏΠ΅Ρ€Π΅ΠΊΡ€Ρ‹Ρ‚ΠΈΠΈ тСряСт ΠΎΠ±ΡŠΠ΅ΠΊΡ‚ ΠΈΠ· Π²ΠΈΠ΄Ρƒ.

НСсомнСнным прСимущСством Ρ‚Ρ€Π΅ΠΊΠ΅Ρ€Π° являСтся Ρ‚ΠΎ, Ρ‡Ρ‚ΠΎ Π½Π°ΠΌ Π½Π΅ ΠΎΠ±ΡΠ·Π°Ρ‚Π΅Π»ΡŒΠ½ΠΎ ΠΈΠΌΠ΅Ρ‚ΡŒ ΠΏΡ€Π΅Π΄ΠΎΠ±ΡƒΡ‡Π΅Π½Π½Ρ‹ΠΉ базис, ΠΊΠ°ΠΊ этого Ρ‚Ρ€Π΅Π±ΡƒΠ΅Ρ‚, Π½Π°ΠΏΡ€ΠΈΠΌΠ΅Ρ€, DeepSORT, хотя Π½ΠΈΠΊΡ‚ΠΎ Π½Π΅ Π·Π°ΠΏΡ€Π΅Ρ‰Π°Π΅Ρ‚ ΠΏΡ€Π΅Π΄Π²Π°Ρ€ΠΈΡ‚Π΅Π»ΡŒΠ½ΠΎ ΠΎΠ±ΡƒΡ‡ΠΈΡ‚ΡŒ базис Π½Π° Ρ†Π΅Π»Π΅Π²ΠΎΠΌ ΠΎΠ±ΡŠΠ΅ΠΊΡ‚Π΅ ΠΈ ΠΈΡΠΏΠΎΠ»ΡŒΠ·ΠΎΠ²Π°Ρ‚ΡŒ Π΅Π³ΠΎ Π² качСствС Π½Π°Ρ‡Π°Π»ΡŒΠ½ΠΎΠ³ΠΎ, вмСсто пустого (см. шаг 2 ΠΎΠ±Ρ‰Π΅Π³ΠΎ Π°Π»Π³ΠΎΡ€ΠΈΡ‚ΠΌΠ°).

Π—Π°ΠΊΠ»ΡŽΡ‡Π΅Π½ΠΈΠ΅

НСсмотря Π½Π° Ρ‚ΠΎ, Ρ‡Ρ‚ΠΎ этот Ρ‚Ρ€Π΅ΠΊΠ΅Ρ€ ΡƒΠΆΠ΅ Π½Π΅ соврСмСнный ΠΈ с появлСниСм нСйросСтСвых ΠΌΠΎΠ΄Π΅Π»Π΅ΠΉ постСпСнно ΠΎΡ‚Ρ…ΠΎΠ΄ΠΈΡ‚ Π½Π° Π²Ρ‚ΠΎΡ€ΠΎΠΉ ΠΏΠ»Π°Π½, это всС ΠΆΠ΅ интСрСсный ΠΏΡ€ΠΈΠΌΠ΅Ρ€ использования ΠΌΠ΅Ρ‚ΠΎΠ΄Π° Π³Π»Π°Π²Π½Ρ‹Ρ… ΠΊΠΎΠΌΠΏΠΎΠ½Π΅Π½Ρ‚, ΠΊΠΎΡ‚ΠΎΡ€Ρ‹ΠΉ ΠΏΠΎΠ»Π΅Π·Π½ΠΎ Ρ€Π°ΡΡΠΌΠΎΡ‚Ρ€Π΅Ρ‚ΡŒ ΠΊΠ°ΠΊ ΠΌΠΈΠ½ΠΈΠΌΡƒΠΌ Π² ΠΎΠ±Ρ€Π°Π·ΠΎΠ²Π°Ρ‚Π΅Π»ΡŒΠ½Ρ‹Ρ… цСлях ΠΈ ΠΊΠ°ΠΊ максимум ΠΏΡ€ΠΈΠΌΠ΅Π½ΠΈΡ‚ΡŒ Π΅Π³ΠΎ Π² Ρ‚Π΅Ρ… случаях, ΠΊΠΎΠ³Π΄Π° нСйросСтСвая ΠΎΠ±Ρ€Π°Π±ΠΎΡ‚ΠΊΠ° слишком Π΄ΠΎΡ€ΠΎΠ³Π° ΠΈΠ»ΠΈ ΠΊΠΎΠ³Π΄Π° трСбуСтся высокая частота ΠΎΠ±Ρ€Π°Π±ΠΎΡ‚ΠΊΠΈ ΠΊΠ°Π΄Ρ€ΠΎΠ². ИдСю ΠΆΠ΅ ΠΈΠ½ΠΊΡ€Π΅ΠΌΠ΅Π½Ρ‚Π½ΠΎΠ³ΠΎ обучСния ΠΌΠΎΠΆΠ½ΠΎ ΠΏΠΎΠ΄Ρ…Π²Π°Ρ‚ΠΈΡ‚ΡŒ ΠΏΡ€ΠΈ Ρ€Π΅Π°Π»ΠΈΠ·Π°Ρ†ΠΈΠΈ Π΄Ρ€ΡƒΠ³ΠΈΡ… CV-Π°Π»Π³ΠΎΡ€ΠΈΡ‚ΠΌΠΎΠ².

НадСюсь ΠΌΠ°Ρ‚Π΅Ρ€ΠΈΠ°Π» оказался Π½Π΅ слишком Π½Π°Π³Ρ€ΡƒΠΆΠ΅Π½Π½Ρ‹ΠΌ ΠΈ ΠΊΠ°ΠΆΠ΄Ρ‹ΠΉ нашСл Π² Π½Π΅ΠΌ для сСбя Ρ‡Ρ‚ΠΎ-Ρ‚ΠΎ ΠΏΠΎΠ»Π΅Π·Π½ΠΎΠ΅.

Бсылки

  1. D. Ross, J. Lim, R. S. Lin, M. H. Yang.Β  Incremental Learning for Robust Visual Tracking. 2008.

  2. M. J. Black and A. D. Jepson. Eigentracking: Robust matching and tracking of articulated objects using view-based representation. 1996.

  3. M. Isard and A. Blake. Contour tracking by stochastic propagation of conditional density. 1996.

  4. A. Levy and M. Lindenbaum. Sequential Karhunen-Loeve basis extraction and its application to images. 2000.

3-8 9 ΠžΡ†Π΅Π½ΠΈΡ‚ΡŒ ΠΊΠ²Π°Π΄Ρ€Π°Ρ‚Π½Ρ‹ΠΉ ΠΊΠΎΡ€Π΅Π½ΡŒ ΠΈΠ· 12 10 ΠžΡ†Π΅Π½ΠΈΡ‚ΡŒ ΠΊΠ²Π°Π΄Ρ€Π°Ρ‚Π½Ρ‹ΠΉ ΠΊΠΎΡ€Π΅Π½ΡŒ ΠΈΠ· 20 11 ΠžΡ†Π΅Π½ΠΈΡ‚ΡŒ ΠΊΠ²Π°Π΄Ρ€Π°Ρ‚Π½Ρ‹ΠΉ ΠΊΠΎΡ€Π΅Π½ΡŒ ΠΈΠ· 50 94 18 ΠžΡ†Π΅Π½ΠΈΡ‚ΡŒ ΠΊΠ²Π°Π΄Ρ€Π°Ρ‚Π½Ρ‹ΠΉ ΠΊΠΎΡ€Π΅Π½ΡŒ ΠΈΠ· 45 19 ΠžΡ†Π΅Π½ΠΈΡ‚ΡŒ ΠΊΠ²Π°Π΄Ρ€Π°Ρ‚Π½Ρ‹ΠΉ ΠΊΠΎΡ€Π΅Π½ΡŒ ΠΈΠ· 32
20 ΠžΡ†Π΅Π½ΠΈΡ‚ΡŒ ΠΊΠ²Π°Π΄Ρ€Π°Ρ‚Π½Ρ‹ΠΉ ΠΊΠΎΡ€Π΅Π½ΡŒ ΠΈΠ· 18 93-8 9 ΠžΡ†Π΅Π½ΠΈΡ‚ΡŒ ΠΊΠ²Π°Π΄Ρ€Π°Ρ‚Π½Ρ‹ΠΉ ΠΊΠΎΡ€Π΅Π½ΡŒ ΠΈΠ· 12 10 ΠžΡ†Π΅Π½ΠΈΡ‚ΡŒ ΠΊΠ²Π°Π΄Ρ€Π°Ρ‚Π½Ρ‹ΠΉ ΠΊΠΎΡ€Π΅Π½ΡŒ ΠΈΠ· 20 11 ΠžΡ†Π΅Π½ΠΈΡ‚ΡŒ ΠΊΠ²Π°Π΄Ρ€Π°Ρ‚Π½Ρ‹ΠΉ ΠΊΠΎΡ€Π΅Π½ΡŒ ΠΈΠ· 50 94 18 ΠžΡ†Π΅Π½ΠΈΡ‚ΡŒ ΠΊΠ²Π°Π΄Ρ€Π°Ρ‚Π½Ρ‹ΠΉ ΠΊΠΎΡ€Π΅Π½ΡŒ ΠΈΠ· 45 19 ΠžΡ†Π΅Π½ΠΈΡ‚ΡŒ ΠΊΠ²Π°Π΄Ρ€Π°Ρ‚Π½Ρ‹ΠΉ ΠΊΠΎΡ€Π΅Π½ΡŒ ΠΈΠ· 32 20 ΠžΡ†Π΅Π½ΠΈΡ‚ΡŒ ΠΊΠ²Π°Π΄Ρ€Π°Ρ‚Π½Ρ‹ΠΉ ΠΊΠΎΡ€Π΅Π½ΡŒ ΠΈΠ· 18 92

Π€ΡƒΠ½ΠΊΡ†ΠΈΠΈ: Π΄ΠΎΠΌΠ΅Π½, Π΄ΠΎΠΌΠ΅Π½ Π² Π΄ΠΈΠ°ΠΏΠ°Π·ΠΎΠ½Π΅.

..🎢

Π€ΡƒΠ½ΠΊΡ†ΠΈΠΈ ΠΈ ΠΎΡ‚Π½ΠΎΡˆΠ΅Π½ΠΈΡ

Purplemath

ВСрнСмся ΠΊ Ρ‚Π΅ΠΌΠ΅ Π΄ΠΎΠΌΠ΅Π½ΠΎΠ² ΠΈ Π΄ΠΈΠ°ΠΏΠ°Π·ΠΎΠ½ΠΎΠ².

ΠŸΡ€ΠΈ ΠΏΠ΅Ρ€Π²ΠΎΠΌ знакомствС с функциями Π²Π°ΠΌ, вСроятно, придСтся ΠΈΠΌΠ΅Ρ‚ΡŒ Π΄Π΅Π»ΠΎ с Π½Π΅ΠΊΠΎΡ‚ΠΎΡ€Ρ‹ΠΌΠΈ ΡƒΠΏΡ€ΠΎΡ‰Π΅Π½Π½Ρ‹ΠΌΠΈ «функциями» ΠΈ ΠΎΡ‚Π½ΠΎΡˆΠ΅Π½ΠΈΡΠΌΠΈ, ΠΎΠ±Ρ‹Ρ‡Π½ΠΎ ΠΏΡ€Π΅Π΄ΡΡ‚Π°Π²Π»ΡΡŽΡ‰ΠΈΠΌΠΈ собой просто Π½Π°Π±ΠΎΡ€Ρ‹ Ρ‚ΠΎΡ‡Π΅ΠΊ. Π­Ρ‚ΠΎ Π½Π΅ Π±ΡƒΠ΄ΡƒΡ‚ ΠΎΡ‡Π΅Π½ΡŒ ΠΏΠΎΠ»Π΅Π·Π½Ρ‹Π΅ ΠΈΠ»ΠΈ интСрСсныС Ρ„ΡƒΠ½ΠΊΡ†ΠΈΠΈ ΠΈ ΠΎΡ‚Π½ΠΎΡˆΠ΅Π½ΠΈΡ, Π½ΠΎ ваш тСкст Ρ…ΠΎΡ‡Π΅Ρ‚, Ρ‡Ρ‚ΠΎΠ±Ρ‹ Π²Ρ‹ ΠΏΠΎΠ»ΡƒΡ‡ΠΈΠ»ΠΈ прСдставлСниС ΠΎ Π΄ΠΎΠΌΠ΅Π½Π΅ ΠΈ Π΄ΠΈΠ°ΠΏΠ°Π·ΠΎΠ½Π΅ Ρ„ΡƒΠ½ΠΊΡ†ΠΈΠΈ.

Π‘ΠΎΠ΄Π΅Ρ€ΠΆΠ°Π½ΠΈΠ΅ продолТаСтся Π½ΠΈΠΆΠ΅

MathHelp.com

Π”ΠΎΠΌΠ΅Π½ ΠΈ Π΄ΠΈΠ°ΠΏΠ°Π·ΠΎΠ½

Π§Ρ‚ΠΎ Ρ‚Π°ΠΊΠΎΠ΅ Π΄ΠΎΠΌΠ΅Π½ ΠΈ Π΄ΠΈΠ°ΠΏΠ°Π·ΠΎΠ½?

ΠžΠ±Π»Π°ΡΡ‚ΡŒ опрСдСлСния ΠΎΡ‚Π½ΠΎΡˆΠ΅Π½ΠΈΡ (Π° Π·Π½Π°Ρ‡ΠΈΡ‚, ΠΈ Ρ„ΡƒΠ½ΠΊΡ†ΠΈΠΈ) β€” это Π½Π°Π±ΠΎΡ€ допустимых Π²Ρ…ΠΎΠ΄Π½Ρ‹Ρ… Π΄Π°Π½Π½Ρ‹Ρ…; это всС значСния x Π² Ρ‚ΠΎΡ‡ΠΊΠ°Ρ… ( x , y ), опрСдСляСмыС ΡΠΎΠΎΡ‚Π½ΠΎΡˆΠ΅Π½ΠΈΠ΅ΠΌ. Π”ΠΈΠ°ΠΏΠ°Π·ΠΎΠ½ ΠΎΡ‚Π½ΠΎΡˆΠ΅Π½ΠΈΡ (ΠΈ, ΡΠ»Π΅Π΄ΠΎΠ²Π°Ρ‚Π΅Π»ΡŒΠ½ΠΎ, Ρ‚Π°ΠΊΠΆΠ΅ Ρ„ΡƒΠ½ΠΊΡ†ΠΈΠΈ) β€” это Π½Π°Π±ΠΎΡ€ Ρ€Π΅Π·ΡƒΠ»ΡŒΡ‚ΠΈΡ€ΡƒΡŽΡ‰ΠΈΡ… Π²Ρ‹Ρ…ΠΎΠ΄ΠΎΠ²; это всС значСния y Π² ( x , y ) Ρ‚ΠΎΡ‡Π΅ΠΊ, опрСдСляСмых ΡΠΎΠΎΡ‚Π½ΠΎΡˆΠ΅Π½ΠΈΠ΅ΠΌ.

Π•ΡΡ‚ΡŒ Π»ΠΈ ΠΌΡƒΠ·Ρ‹ΠΊΠ°Π»ΡŒΠ½Ρ‹ΠΉ способ Π·Π°ΠΏΠΎΠΌΠ½ΠΈΡ‚ΡŒ, Π³Π΄Π΅ Π΄ΠΎΠΌΠ΅Π½, Π° Π³Π΄Π΅ Π΄ΠΈΠ°ΠΏΠ°Π·ΠΎΠ½?

Π•ΡΡ‚ΡŒ старая ковбойская пСсня, Π² ΠΊΠΎΡ‚ΠΎΡ€ΠΎΠΉ начинаСтся ΠΏΡ€ΠΈΠΏΠ΅Π²: Β«Π”ΠΎΠΌΠΎΠΉ, Π΄ΠΎΠΌΠΎΠΌ Π½Π° пастбищС / Π“Π΄Π΅ ΠΈΠ³Ρ€Π°ΡŽΡ‚ ΠΎΠ»Π΅Π½ΠΈ ΠΈ Π°Π½Ρ‚ΠΈΠ»ΠΎΠΏΡ‹Β»; Π²Ρ‹, вСроятно, ΡΠ»Ρ‹ΡˆΠΈΡ‚Π΅ это Π² своСй Π³ΠΎΠ»ΠΎΠ²Π΅ прямо сСйчас. ВмСсто этого ΠΏΠΎΠΉΡ‚Π΅ ΠΏΡ€ΠΈΠΏΠ΅Π² ΠΊΠ°ΠΊ Β«Π”ΠΎΠΌΠ΅Π½, Π΄ΠΎΠΌΠ΅Π½ Π² Π΄ΠΈΠ°ΠΏΠ°Π·ΠΎΠ½Π΅Β», ΠΈ это ΠΏΠΎΠΌΠΎΠΆΠ΅Ρ‚ Π²Π°ΠΌ ΠΏΠΎΠ½ΡΡ‚ΡŒ, Ρ‡Ρ‚ΠΎ Π΅ΡΡ‚ΡŒ Ρ‡Ρ‚ΠΎ.

ΠŸΡ€Π΅Π΄ΡΡ‚Π°Π²ΡŒΡ‚Π΅, Ρ‡Ρ‚ΠΎ Π²Ρ‹ ΠΆΠΈΠ²Π΅Ρ‚Π΅ Π² малСнькой ΡƒΡΠ°Π΄ΡŒΠ±Π΅ посрСди большого ΠΎΡ‚ΠΊΡ€Ρ‹Ρ‚ΠΎΠ³ΠΎ пространства. Π’Π°Ρˆ Π΄ΠΎΠΌ β€” это ваш Π΄ΠΎΠΌΠ΅Π½; это мСсто, Π³Π΄Π΅ Π²Ρ‹ Π½Π°Ρ‡ΠΈΠ½Π°Π΅Ρ‚Π΅ свой дСнь. Как Ρ‚ΠΎΠ»ΡŒΠΊΠΎ Π²Ρ‹ встанСтС, Π²Ρ‹ Π±Π΅Ρ€Π΅Ρ‚Π΅ лошадь ΠΈ Π½Π°ΠΏΡ€Π°Π²Π»ΡΠ΅Ρ‚Π΅ΡΡŒ Π² ΡˆΠΈΡ€ΠΎΠΊΠΎΠ΅ ΠΎΡ‚ΠΊΡ€Ρ‹Ρ‚ΠΎΠ΅ пространство, ΡΠ²Π»ΡΡŽΡ‰Π΅Π΅ΡΡ пастбищами Ρ€Π°Π²Π½ΠΈΠ½. Π”ΠΎΠΌΠ΅Π½ β€” это мСсто, Π³Π΄Π΅ Π½Π°Ρ‡ΠΈΠ½Π°ΡŽΡ‚ΡΡ ΠΎΡ‚Π½ΠΎΡˆΠ΅Π½ΠΈΡ; Π΄ΠΈΠ°ΠΏΠ°Π·ΠΎΠ½, Π³Π΄Π΅ ΠΎΠ½ ΠΈΠ΄Π΅Ρ‚ Π½Π° Ρ€Π°Π±ΠΎΡ‚Ρƒ.

(Π­ΠΉ, ΠΌΡƒΠ·Ρ‹ΠΊΠ°Π»ΡŒΠ½Π°Ρ ΡˆΡ‚ΡƒΠΊΠ° ΠΌΠΎΠΆΠ΅Ρ‚ Π±Ρ‹Ρ‚ΡŒ Π³Π»ΡƒΠΏΠΎΠΉ, Π½ΠΎ ΠΎΠ½Π° Ρ€Π°Π±ΠΎΡ‚Π°Π΅Ρ‚ для Π½Π΅ΠΊΠΎΡ‚ΠΎΡ€Ρ‹Ρ… ΠΈΠ· нас, Ρ…ΠΎΡ€ΠΎΡˆΠΎ?)

НСбольшиС Π½Π°Π±ΠΎΡ€Ρ‹, содСрТащиС всСго нСсколько Ρ‚ΠΎΡ‡Π΅ΠΊ, ΠΎΠ±Ρ‹Ρ‡Π½ΠΎ ΠΏΡ€Π΅Π΄ΡΡ‚Π°Π²Π»ΡΡŽΡ‚ собой самыС простыС Π²ΠΈΠ΄Ρ‹ ΠΎΡ‚Π½ΠΎΡˆΠ΅Π½ΠΈΠΉ, поэтому ваша ΠΊΠ½ΠΈΠ³Π° начинаСтся с Π½ΠΈΡ….

Π§Ρ‚ΠΎ являСтся ΠΏΡ€ΠΈΠΌΠ΅Ρ€ΠΎΠΌ опрСдСлСния Π΄ΠΎΠΌΠ΅Π½Π° ΠΈ Π΄ΠΈΠ°ΠΏΠ°Π·ΠΎΠ½Π° мноТСства Ρ‚ΠΎΡ‡Π΅ΠΊ?

{(2, βˆ’3), (4, 6), (3, βˆ’1), (6, 6), (2, 3)}

ΠŸΡ€ΠΈΠ²Π΅Π΄Π΅Π½Π½Ρ‹ΠΉ Π²Ρ‹ΡˆΠ΅ список Ρ‚ΠΎΡ‡Π΅ΠΊ, ΡΠ²Π»ΡΡŽΡ‰ΠΈΠΉΡΡ ΠΎΡ‚Π½ΠΎΡˆΠ΅Π½ΠΈΠ΅ΠΌ ΠΌΠ΅ΠΆΠ΄Ρƒ ΠΎΠΏΡ€Π΅Π΄Π΅Π»Π΅Π½Π½Ρ‹ΠΌΠΈ x ΠΈ ΠΎΠΏΡ€Π΅Π΄Π΅Π»Π΅Π½Π½Ρ‹ΠΌΠΈ ΠΈ , это ΠΎΡ‚Π½ΠΎΡˆΠ΅Π½ΠΈΠ΅. Π”ΠΎΠΌΠ΅Π½ β€” это всС значСния x , Π° Π΄ΠΈΠ°ΠΏΠ°Π·ΠΎΠ½ β€” всС значСния y . Π§Ρ‚ΠΎΠ±Ρ‹ ΡƒΠΊΠ°Π·Π°Ρ‚ΡŒ Π΄ΠΎΠΌΠ΅Π½ ΠΈ Π΄ΠΈΠ°ΠΏΠ°Π·ΠΎΠ½, я просто ΠΏΠ΅Ρ€Π΅Ρ‡ΠΈΡΠ»ΡΡŽ значСния Π±Π΅Π· дублирования:

Π΄ΠΎΠΌΠ΅Π½: {2, 3, 4, 6}

Π΄ΠΈΠ°ΠΏΠ°Π·ΠΎΠ½: {βˆ’3, βˆ’1, 3, 6}

(ΠΎΠ±Ρ‹Ρ‡Π½ΠΎ Ρ‡Ρ‚ΠΎΠ±Ρ‹ ΠΏΠ΅Ρ€Π΅Ρ‡ΠΈΡΠ»ΠΈΡ‚ΡŒ эти значСния Π² числовом порядкС, Π½ΠΎ трСбуСтся , Π° Π½Π΅ . Наборы ΠΏΠΎ ΠΎΠΏΡ€Π΅Π΄Π΅Π»Π΅Π½ΠΈΡŽ ΡΠ²Π»ΡΡŽΡ‚ΡΡ *нСупорядочСнными* списками, поэтому Π²Ρ‹ ΠΌΠΎΠΆΠ΅Ρ‚Π΅ ΠΏΠ΅Ρ€Π΅Ρ‡ΠΈΡΠ»ΡΡ‚ΡŒ числа Π² любом порядкС, ΠΊΠΎΡ‚ΠΎΡ€Ρ‹ΠΉ Π²Π°ΠΌ нравится. Π½ΠΎΡ€ΠΌΠ°Π»ΡŒΠ½ΠΎ Π² Π½Π°Π±ΠΎΡ€Π°Ρ…, Π½ΠΎ Π±ΠΎΠ»ΡŒΡˆΠΈΠ½ΡΡ‚Π²ΠΎ инструкторов Π·Π° это Π·Π°Ρ‡Ρ‚ΡƒΡ‚.)

Π₯отя Π΄Π°Π½Π½Ρ‹ΠΉ Π½Π°Π±ΠΎΡ€ Ρ‚ΠΎΡ‡Π΅ΠΊ Π΄Π΅ΠΉΡΡ‚Π²ΠΈΡ‚Π΅Π»ΡŒΠ½ΠΎ прСдставляСт ΠΎΡ‚Π½ΠΎΡˆΠ΅Π½ΠΈΠ΅ (ΠΏΠΎΡΠΊΠΎΠ»ΡŒΠΊΡƒ x ΠΈ y связаны Π΄Ρ€ΡƒΠ³ с Π΄Ρ€ΡƒΠ³ΠΎΠΌ), Π½Π°Π±ΠΎΡ€, ΠΊΠΎΡ‚ΠΎΡ€Ρ‹ΠΉ ΠΎΠ½ΠΈ ΠΌΠ½Π΅ Π΄Π°Π»ΠΈ, содСрТит Π΄Π²Π΅ Ρ‚ΠΎΡ‡ΠΊΠΈ с ΠΎΠ΄ΠΈΠ½Π°ΠΊΠΎΠ²Ρ‹ΠΌΠΈ x -Π·Π½Π°Ρ‡Π΅Π½ΠΈΠ΅: (2,Β βˆ’3) ΠΈ (2,Β 3). ΠŸΠΎΡΠΊΠΎΠ»ΡŒΠΊΡƒ x Β =Β 2 Π΄Π°Π΅Ρ‚ ΠΌΠ½Π΅ Π΄Π²Π° Π²ΠΎΠ·ΠΌΠΎΠΆΠ½Ρ‹Ρ… мСста назначСния (Ρ‚ΠΎ Π΅ΡΡ‚ΡŒ Π΄Π²Π° Π²ΠΎΠ·ΠΌΠΎΠΆΠ½Ρ‹Ρ… значСния y ), Ρ‚ΠΎ это ΠΎΡ‚Π½ΠΎΡˆΠ΅Π½ΠΈΠ΅ Π½Π΅ ΠΌΠΎΠΆΠ΅Ρ‚ Π±Ρ‹Ρ‚ΡŒ Ρ„ΡƒΠ½ΠΊΡ†ΠΈΠ΅ΠΉ.

И ΠΊΠΎΠ³Π΄Π° ΠΎΡ‚Π½ΠΎΡˆΠ΅Π½ΠΈΠ΅, ΠΊΠΎΡ‚ΠΎΡ€ΠΎΠ΅ ΠΎΠ½ΠΈ ΠΌΠ½Π΅ Π΄Π°Π»ΠΈ, прСдставляСт собой Π½Π°Π±ΠΎΡ€ Ρ‚ΠΎΡ‡Π΅ΠΊ, всС, Ρ‡Ρ‚ΠΎ ΠΌΠ½Π΅ Π½ΡƒΠΆΠ½ΠΎ ΡΠ΄Π΅Π»Π°Ρ‚ΡŒ, это ΠΏΡ€ΠΎΠ²Π΅Ρ€ΠΈΡ‚ΡŒ Ρ‚ΠΎΡ‡ΠΊΠΈ’ x — значСния; Ссли ΠΊΠ°ΠΊΠΎΠ΅-Π»ΠΈΠ±ΠΎ x появляСтся Π±ΠΎΠ»Π΅Π΅ ΠΎΠ΄Π½ΠΎΠ³ΠΎ Ρ€Π°Π·Π°, Ρ‚ΠΎ ΠΎΡ‚Π½ΠΎΡˆΠ΅Π½ΠΈΠ΅ Π½Π΅ являСтся Ρ„ΡƒΠ½ΠΊΡ†ΠΈΠ΅ΠΉ. Π­Ρ‚ΠΎ ΠΎΡ‚Π½ΠΎΡˆΠ΅Π½ΠΈΠ΅ повторяСтся, Ρ‚Π°ΠΊ Ρ‡Ρ‚ΠΎ ΠΎΠ½ΠΎ Π΅ΡΡ‚ΡŒ:

Π½Π΅ функция

ΠžΠ±Ρ€Π°Ρ‚ΠΈΡ‚Π΅ Π²Π½ΠΈΠΌΠ°Π½ΠΈΠ΅, Ρ‡Ρ‚ΠΎ всС, Ρ‡Ρ‚ΠΎ ΠΌΠ½Π΅ Π½ΡƒΠΆΠ½ΠΎ Π±Ρ‹Π»ΠΎ ΡΠ΄Π΅Π»Π°Ρ‚ΡŒ, Ρ‡Ρ‚ΠΎΠ±Ρ‹ ΠΏΡ€ΠΎΠ²Π΅Ρ€ΠΈΡ‚ΡŒ, являСтся Π»ΠΈ ΠΎΡ‚Π½ΠΎΡˆΠ΅Π½ΠΈΠ΅ Ρ„ΡƒΠ½ΠΊΡ†ΠΈΠ΅ΠΉ, это Π½Π°ΠΉΡ‚ΠΈ ΠΏΠΎΠ²Ρ‚ΠΎΡ€ΡΡŽΡ‰ΠΈΠ΅ΡΡ значСния x . Если Π²Ρ‹ Π½Π°ΠΉΠ΄Π΅Ρ‚Π΅ ΠΏΠΎΠ²Ρ‚ΠΎΡ€ΡΡŽΡ‰ΠΈΠ΅ΡΡ значСния x , Ρ‚ΠΎ Ρ€Π°Π·Π½Ρ‹Π΅ значСния y ΠΎΠ·Π½Π°Ρ‡Π°ΡŽΡ‚, Ρ‡Ρ‚ΠΎ Ρƒ вас Π½Π΅Ρ‚ Ρ„ΡƒΠ½ΠΊΡ†ΠΈΠΈ. ΠŸΠΎΠΌΠ½ΠΈΡ‚Π΅: Ρ‡Ρ‚ΠΎΠ±Ρ‹ ΠΎΡ‚Π½ΠΎΡˆΠ΅Π½ΠΈΠ΅ Π±Ρ‹Π»ΠΎ Ρ„ΡƒΠ½ΠΊΡ†ΠΈΠ΅ΠΉ, ΠΊΠ°ΠΆΠ΄ΠΎΠ΅ Π·Π½Π°Ρ‡Π΅Π½ΠΈΠ΅ Ρ€Π°Π·ΠΌΠ΅Ρ€ΠΎΠΌ x Π΄ΠΎΠ»ΠΆΠ½ΠΎ ΡΠΎΠΎΡ‚Π²Π΅Ρ‚ΡΡ‚Π²ΠΎΠ²Π°Ρ‚ΡŒ Π΅Π΄ΠΈΠ½ΠΈΡ†Π΅, ΠΈ Ρ‚ΠΎΠ»ΡŒΠΊΠΎ ΠΎΠ΄Π½ΠΎ Π·Π½Π°Ρ‡Π΅Π½ΠΈΠ΅ , ΠΈ .

{(βˆ’3, 5), (βˆ’2, 5), (βˆ’1, 5), (0, 5), (1, 5), (2, 5)}

ВсС, Ρ‡Ρ‚ΠΎ ΠΌΠ½Π΅ Π½ΡƒΠΆΠ½ΠΎ ΡΠ΄Π΅Π»Π°Ρ‚ΡŒ для части Π΄ΠΎΠΌΠ΅Π½Π° ΠΈ Π΄ΠΈΠ°ΠΏΠ°Π·ΠΎΠ½Π° Π² этом ΡƒΠΏΡ€Π°ΠΆΠ½Π΅Π½ΠΈΠΈ, это ΠΏΠ΅Ρ€Π΅Ρ‡ΠΈΡΠ»ΠΈΡ‚ΡŒ значСния x для Π΄ΠΎΠΌΠ΅Π½Π° ΠΈ значСния y для Π΄ΠΈΠ°ΠΏΠ°Π·ΠΎΠ½Π°. Π― помню, Ρ‡Ρ‚ΠΎ для ΠΊΠ°ΠΆΠ΄ΠΎΠ³ΠΎ ΠΈΠ· Π½ΠΈΡ… Π½ΡƒΠΆΠ½ΠΎ ΠΈΡΠΏΠΎΠ»ΡŒΠ·ΠΎΠ²Π°Ρ‚ΡŒ Ρ„ΠΈΠ³ΡƒΡ€Π½Ρ‹Π΅ скобки:

Π΄ΠΎΠΌΠ΅Π½: {βˆ’3, βˆ’2, βˆ’1, 0, 1, 2}

Π΄ΠΈΠ°ΠΏΠ°Π·ΠΎΠ½: {5}

Π­Ρ‚ΠΎ Π΅Ρ‰Π΅ ΠΎΠ΄ΠΈΠ½ ΠΏΡ€ΠΈΠΌΠ΅Ρ€ «скучного» ΠΊΠ°ΠΊ ΠΈ Π² ΠΏΡ€ΠΈΠΌΠ΅Ρ€Π΅ Π½Π° ΠΏΡ€Π΅Π΄Ρ‹Π΄ΡƒΡ‰Π΅ΠΉ страницС: ΠΊΠ°ΠΆΠ΄Ρ‹Π΅ послСдниС 9Π—Π½Π°Ρ‡Π΅Π½ΠΈΠ΅ 1827 x соотвСтствуСт Ρ‚ΠΎΡ‡Π½ΠΎ Ρ‚Π°ΠΊΠΎΠΌΡƒ ΠΆΠ΅ Π·Π½Π°Ρ‡Π΅Π½ΠΈΡŽ y . Но ΠΊΠ°ΠΆΠ΄ΠΎΠ΅ x Π·Π½Π°Ρ‡Π΅Π½ΠΈΠ΅ отличаСтся, поэтому, хотя ΠΈ скучно,

это ΠΎΡ‚Π½ΠΎΡˆΠ΅Π½ΠΈΠ΅ Π΄Π΅ΠΉΡΡ‚Π²ΠΈΡ‚Π΅Π»ΡŒΠ½ΠΎ являСтся Ρ„ΡƒΠ½ΠΊΡ†ΠΈΠ΅ΠΉ.

На самом Π΄Π΅Π»Π΅ эти Ρ‚ΠΎΡ‡ΠΊΠΈ Π»Π΅ΠΆΠ°Ρ‚ Π½Π° Π³ΠΎΡ€ΠΈΠ·ΠΎΠ½Ρ‚Π°Π»ΡŒΠ½ΠΎΠΉ Π»ΠΈΠ½ΠΈΠΈ y Β =Β 5.


«. Π’Π°ΠΊΠΈΠΌ ΠΎΠ±Ρ€Π°Π·ΠΎΠΌ, Π΄ΠΈΠ°ΠΏΠ°Π·ΠΎΠ½ ΠΌΠΎΠΆΠ΅Ρ‚ Ρ‚Π°ΠΊΠΆΠ΅ Π±Ρ‹Ρ‚ΡŒ сформулирован ΠΊΠ°ΠΊ Β«ΠΎΠ΄ΠΈΠ½ΠΎΡ‡ΠΊΠ° 5Β»


Π•ΡΡ‚ΡŒ Π΅Ρ‰Π΅ ΠΎΠ΄ΠΈΠ½ случай для нахоТдСния Π΄ΠΎΠΌΠ΅Π½Π° ΠΈ Π΄ΠΈΠ°ΠΏΠ°Π·ΠΎΠ½Π° Ρ„ΡƒΠ½ΠΊΡ†ΠΈΠΉ. Π’Π°ΠΌ Π΄Π°Π΄ΡƒΡ‚ Ρ„ΡƒΠ½ΠΊΡ†ΠΈΡŽ ΠΈ попросят Π½Π°ΠΉΡ‚ΠΈ Π΄ΠΎΠΌΠ΅Π½ (Π° ΠΌΠΎΠΆΠ΅Ρ‚ ΠΈ Π΄ΠΈΠ°ΠΏΠ°Π·ΠΎΠ½ Ρ‚ΠΎΠΆΠ΅). На Π΄Π°Π½Π½ΠΎΠΌ этапС вашСй матСматичСской ΠΊΠ°Ρ€ΡŒΠ΅Ρ€Ρ‹ я Π²ΠΈΠ΄Π΅Π» (ΠΈΠ»ΠΈ Π΄Π°ΠΆΠ΅ ΠΌΠΎΠ³Ρƒ Π΄ΡƒΠΌΠ°Ρ‚ΡŒ) Ρ‚ΠΎΠ»ΡŒΠΊΠΎ ΠΎ Π΄Π²ΡƒΡ… Π²Π΅Ρ‰Π°Ρ…, ΠΊΠΎΡ‚ΠΎΡ€Ρ‹Π΅ Π²Π°ΠΌ придСтся ΠΏΡ€ΠΎΠ²Π΅Ρ€ΠΈΡ‚ΡŒ, Ρ‡Ρ‚ΠΎΠ±Ρ‹ ΠΎΠΏΡ€Π΅Π΄Π΅Π»ΠΈΡ‚ΡŒ ΠΎΠ±Π»Π°ΡΡ‚ΡŒ опрСдСлСния Ρ„ΡƒΠ½ΠΊΡ†ΠΈΠΈ, ΠΊΠΎΡ‚ΠΎΡ€ΡƒΡŽ ΠΎΠ½ΠΈ Π²Π°ΠΌ Π΄Π°Π΄ΡƒΡ‚, ΠΈ эти Π΄Π²Π΅ Π²Π΅Ρ‰ΠΈ β€” Π·Π½Π°ΠΌΠ΅Π½Π°Ρ‚Π΅Π»ΠΈ ΠΈ ΠΊΠ²Π°Π΄Ρ€Π°Ρ‚Π½Ρ‹Π΅ ΠΊΠΎΡ€Π½ΠΈ.

Какой ΠΏΡ€ΠΈΠΌΠ΅Ρ€ нахоТдСния области опрСдСлСния ΠΈ области Π·Π½Π°Ρ‡Π΅Π½ΠΈΠΉ Ρ€Π°Ρ†ΠΈΠΎΠ½Π°Π»ΡŒΠ½ΠΎΠΉ Ρ„ΡƒΠ½ΠΊΡ†ΠΈΠΈ?

Π”ΠΎΠΌΠ΅Π½ β€” это всС значСния, ΠΊΠΎΡ‚ΠΎΡ€Ρ‹Π΅ Ρ€Π°Π·Ρ€Π΅ΡˆΠ΅Π½ΠΎ ΠΏΡ€ΠΈΠ½ΠΈΠΌΠ°Ρ‚ΡŒ x . ЕдинствСнная ΠΏΡ€ΠΎΠ±Π»Π΅ΠΌΠ°, с ΠΊΠΎΡ‚ΠΎΡ€ΠΎΠΉ я столкнулся с этой Ρ„ΡƒΠ½ΠΊΡ†ΠΈΠ΅ΠΉ, Π·Π°ΠΊΠ»ΡŽΡ‡Π°Π΅Ρ‚ΡΡ Π² Ρ‚ΠΎΠΌ, Ρ‡Ρ‚ΠΎ ΠΌΠ½Π΅ Π½ΡƒΠΆΠ½ΠΎ Π±Ρ‹Ρ‚ΡŒ остороТным, Ρ‡Ρ‚ΠΎΠ±Ρ‹ Π½Π΅ Π΄Π΅Π»ΠΈΡ‚ΡŒ Π½Π° ноль. Π’Π°ΠΊΠΈΠΌ ΠΎΠ±Ρ€Π°Π·ΠΎΠΌ, СдинствСнныС значСния, ΠΊΠΎΡ‚ΠΎΡ€Ρ‹Π΅ x Π½Π΅ ΠΌΠΎΠ³ΡƒΡ‚ ΠΏΡ€ΠΈΠ½ΠΈΠΌΠ°Ρ‚ΡŒ, это Ρ‚Π΅, ΠΊΠΎΡ‚ΠΎΡ€Ρ‹Π΅ Π²Ρ‹Π·Π²Π°Π»ΠΈ Π±Ρ‹ Π΄Π΅Π»Π΅Π½ΠΈΠ΅ Π½Π° ноль. ΠŸΠΎΡΡ‚ΠΎΠΌΡƒ я ΠΏΡ€ΠΈΡ€Π°Π²Π½ΡΡŽ Π·Π½Π°ΠΌΠ΅Π½Π°Ρ‚Π΅Π»ΡŒ ΠΊ Π½ΡƒΠ»ΡŽ ΠΈ Ρ€Π΅ΡˆΡƒ; ΠΌΠΎΠΉ Π΄ΠΎΠΌΠ΅Π½ Π±ΡƒΠ΄Π΅Ρ‚ всСм ΠΎΡΡ‚Π°Π»ΡŒΠ½Ρ‹ΠΌ.

x 2 βˆ’ Ρ… — 2 = 0

( Ρ… — 2)( Ρ… + 1) = 0

Ρ… = 2 ΠΈΠ»ΠΈ Ρ… = -1

0. Π½Π΅ Ρ€Π°Π²Π½ΠΎ -1 ΠΈΠ»ΠΈ 2″.

Π”ΠΈΠ°ΠΏΠ°Π·ΠΎΠ½ Π½Π΅ΠΌΠ½ΠΎΠ³ΠΎ слоТнСС, поэтому Π΅Π³ΠΎ ΠΌΠΎΠ³ΡƒΡ‚ ΠΈ Π½Π΅ ΠΏΠΎΠΏΡ€ΠΎΡΠΈΡ‚ΡŒ. Π’ ΠΎΠ±Ρ‰Π΅ΠΌ, ΠΎΠ΄Π½Π°ΠΊΠΎ, ΠΎΠ½ΠΈ захотят, Ρ‡Ρ‚ΠΎΠ±Ρ‹ Π²Ρ‹ построили Π³Ρ€Π°Ρ„ΠΈΠΊ Ρ„ΡƒΠ½ΠΊΡ†ΠΈΠΈ ΠΈ нашли Π΄ΠΈΠ°ΠΏΠ°Π·ΠΎΠ½ ΠΏΠΎ ΠΊΠ°Ρ€Ρ‚ΠΈΠ½ΠΊΠ΅. Π’ Π΄Π°Π½Π½ΠΎΠΌ случаС:

Как Π²ΠΈΠ΄Π½ΠΎ ΠΈΠ· ΠΌΠΎΠ΅Π³ΠΎ рисунка, Π³Ρ€Π°Ρ„ΠΈΠΊ «ΠΏΠΎΠΊΡ€Ρ‹Π²Π°Π΅Ρ‚» всС ΠΈ -значСния; Ρ‚ΠΎ Π΅ΡΡ‚ΡŒ Π³Ρ€Π°Ρ„ΠΈΠΊ Π±ΡƒΠ΄Π΅Ρ‚ ΠΈΠ΄Ρ‚ΠΈ Ρ‚Π°ΠΊ Π½ΠΈΠ·ΠΊΠΎ, ΠΊΠ°ΠΊ ΠΌΠ½Π΅ нравится, ΠΈ Ρ‚Π°ΠΊΠΆΠ΅ Π±ΡƒΠ΄Π΅Ρ‚ ΠΈΠ΄Ρ‚ΠΈ Ρ‚Π°ΠΊ высоко, ΠΊΠ°ΠΊ ΠΌΠ½Π΅ нравится. Для любой Ρ‚ΠΎΡ‡ΠΊΠΈ Π½Π° y -ось, Π½Π΅Π²Π°ΠΆΠ½ΠΎ высоко Π²Π²Π΅Ρ€Ρ… ΠΈΠ»ΠΈ Π½ΠΈΠ·ΠΊΠΎ Π²Π½ΠΈΠ·, я ΠΌΠΎΠ³Ρƒ ΠΏΠΎΠΉΡ‚ΠΈ ΠΎΡ‚ этой Ρ‚ΠΎΡ‡ΠΊΠΈ Π»ΠΈΠ±ΠΎ Π²ΠΏΡ€Π°Π²ΠΎ, Π»ΠΈΠ±ΠΎ Π²Π»Π΅Π²ΠΎ ΠΈ, Π² ΠΊΠΎΠ½Ρ†Π΅ ΠΊΠΎΠ½Ρ†ΠΎΠ², пСрСсСчу Π³Ρ€Π°Ρ„ΠΈΠΊ. ΠŸΠΎΡΠΊΠΎΠ»ΡŒΠΊΡƒ Π³Ρ€Π°Ρ„ΠΈΠΊ Π² ΠΊΠΎΠ½Π΅Ρ‡Π½ΠΎΠΌ ΠΈΡ‚ΠΎΠ³Π΅ ΠΏΠΎΠΊΡ€ΠΎΠ΅Ρ‚ всС Π²ΠΎΠ·ΠΌΠΎΠΆΠ½Ρ‹Π΅ значСния y , Ρ‚ΠΎΠ³Π΄Π°:

Π΄ΠΈΠ°ΠΏΠ°Π·ΠΎΠ½ — это «всС Π΄Π΅ΠΉΡΡ‚Π²ΠΈΡ‚Π΅Π»ΡŒΠ½Ρ‹Π΅ числа».

Π”ΠΎΠΌΠ΅Π½ β€” это всС значСния, ΠΊΠΎΡ‚ΠΎΡ€Ρ‹Π΅ ΠΌΠΎΠΆΠ΅Ρ‚ ΠΏΡ€ΠΈΠ½ΠΈΠΌΠ°Ρ‚ΡŒ x . ЕдинствСнная ΠΏΡ€ΠΎΠ±Π»Π΅ΠΌΠ°, с ΠΊΠΎΡ‚ΠΎΡ€ΠΎΠΉ я столкнулся с этой Ρ„ΡƒΠ½ΠΊΡ†ΠΈΠ΅ΠΉ, Π·Π°ΠΊΠ»ΡŽΡ‡Π°Π΅Ρ‚ΡΡ Π² Ρ‚ΠΎΠΌ, Ρ‡Ρ‚ΠΎ Ρƒ мСня Π½Π΅ ΠΌΠΎΠΆΠ΅Ρ‚ Π±Ρ‹Ρ‚ΡŒ ΠΎΡ‚Ρ€ΠΈΡ†Π°Ρ‚Π΅Π»ΡŒΠ½ΠΎΠ³ΠΎ значСния Π²Π½ΡƒΡ‚Ρ€ΠΈ ΠΊΠ²Π°Π΄Ρ€Π°Ρ‚Π½ΠΎΠ³ΠΎ корня. Π’Π°ΠΊ Ρ‡Ρ‚ΠΎ я ΡƒΡΡ‚Π°Π½ΠΎΠ²Π»ΡŽ внутрСнности большС ΠΈΠ»ΠΈ Ρ€Π°Π²Π½Ρ‹Π΅ Π½ΡƒΠ»ΡŽ ΠΈ Ρ€Π΅ΡˆΡƒ. Π’ Ρ€Π΅Π·ΡƒΠ»ΡŒΡ‚Π°Ρ‚Π΅ Π±ΡƒΠ΄Π΅Ρ‚ ΠΌΠΎΠΉ Π΄ΠΎΠΌΠ΅Π½:

βˆ’2 x + 3 β‰₯ 0
βˆ’2 x β‰₯ βˆ’3
2 x ≀ 3
x ≀ 3/2 = 1,5

Π’ΠΎΠ³Π΄Π° Π΄ΠΎΠΌΠ΅Π½ «всС x ≀ 3/2Β».

Для Π΄ΠΈΠ°ΠΏΠ°Π·ΠΎΠ½Π° трСбуСтся Π³Ρ€Π°Ρ„ΠΈΠΊ. МнС Π½ΡƒΠΆΠ½ΠΎ Π±Ρ‹Ρ‚ΡŒ остороТным ΠΏΡ€ΠΈ графичСском ΠΎΡ‚ΠΎΠ±Ρ€Π°ΠΆΠ΅Π½ΠΈΠΈ Ρ€Π°Π΄ΠΈΠΊΠ°Π»ΠΎΠ²:

Π“Ρ€Π°Ρ„ΠΈΠΊ начинаСтся с y = 0 ΠΈ ΠΈΠ΄Π΅Ρ‚ Π²Π½ΠΈΠ· (Π½Π°ΠΏΡ€Π°Π²Π»ΡΡΡΡŒ Π²Π»Π΅Π²ΠΎ) ΠΎΡ‚Ρ‚ΡƒΠ΄Π°. Π₯отя Π³Ρ€Π°Ρ„ΠΈΠΊ ΠΈΠ΄Π΅Ρ‚ Π²Π½ΠΈΠ· ΠΎΡ‡Π΅Π½ΡŒ ΠΌΠ΅Π΄Π»Π΅Π½Π½ΠΎ, я знаю, Ρ‡Ρ‚ΠΎ, Π² ΠΊΠΎΠ½Ρ†Π΅ ΠΊΠΎΠ½Ρ†ΠΎΠ², я ΠΌΠΎΠ³Ρƒ ΠΎΠΏΡƒΡΡ‚ΠΈΡ‚ΡŒΡΡ Ρ‚Π°ΠΊ Π½ΠΈΠ·ΠΊΠΎ, ΠΊΠ°ΠΊ Π·Π°Ρ…ΠΎΡ‡Ρƒ (Π²Ρ‹Π±Ρ€Π°Π² x , Ρ‡Ρ‚ΠΎ достаточно ΠΌΠ½ΠΎΠ³ΠΎ). ΠšΡ€ΠΎΠΌΠ΅ Ρ‚ΠΎΠ³ΠΎ, ΠΈΠ· ΠΌΠΎΠ΅Π³ΠΎ ΠΎΠΏΡ‹Ρ‚Π° построСния Π³Ρ€Π°Ρ„ΠΈΠΊΠΎΠ² я знаю, Ρ‡Ρ‚ΠΎ Π³Ρ€Π°Ρ„ΠΈΠΊ Π½ΠΈΠΊΠΎΠ³Π΄Π° Π½Π΅ Π½Π°Ρ‡Π½Π΅Ρ‚ Π²ΠΎΡΡΡ‚Π°Π½Π°Π²Π»ΠΈΠ²Π°Ρ‚ΡŒΡΡ. Π’ΠΎΠ³Π΄Π°:

Π΄ΠΈΠ°ΠΏΠ°Π·ΠΎΠ½ «Π²ΡΠ΅ y ≀ 0″.

y = βˆ’ x 4 + 4

Π­Ρ‚ΠΎ ΠΎΠ±Ρ‹Ρ‡Π½Ρ‹ΠΉ ΠΌΠ½ΠΎΠ³ΠΎΡ‡Π»Π΅Π½. Π—Π΄Π΅ΡΡŒ Π½Π΅Ρ‚ Π·Π½Π°ΠΌΠ΅Π½Π°Ρ‚Π΅Π»Π΅ΠΉ (поэтому Π½Π΅Ρ‚ ΠΏΡ€ΠΎΠ±Π»Π΅ΠΌ с Π΄Π΅Π»Π΅Π½ΠΈΠ΅ΠΌ Π½Π° ноль) ΠΈ Ρ€Π°Π΄ΠΈΠΊΠ°Π»ΠΎΠ² (поэтому Π½Π΅Ρ‚ ΠΏΡ€ΠΎΠ±Π»Π΅ΠΌ с ΠΈΠ·Π²Π»Π΅Ρ‡Π΅Π½ΠΈΠ΅ΠΌ ΠΊΠ²Π°Π΄Ρ€Π°Ρ‚Π½ΠΎΠ³ΠΎ корня ΠΈΠ· ΠΎΡ‚Ρ€ΠΈΡ†Π°Ρ‚Π΅Π»ΡŒΠ½ΠΎΠ³ΠΎ). Π‘ ΠΌΠ½ΠΎΠ³ΠΎΡ‡Π»Π΅Π½ΠΎΠΌ ΠΏΡ€ΠΎΠ±Π»Π΅ΠΌ Π½Π΅Ρ‚. НСт Π·Π½Π°Ρ‡Π΅Π½ΠΈΠΉ, ΠΊΠΎΡ‚ΠΎΡ€Ρ‹Π΅ я Π½Π΅ ΠΌΠΎΠ³Ρƒ ΠΈΡΠΏΠΎΠ»ΡŒΠ·ΠΎΠ²Π°Ρ‚ΡŒ для Ρ… . Когда Ρƒ мСня Π΅ΡΡ‚ΡŒ ΠΌΠ½ΠΎΠ³ΠΎΡ‡Π»Π΅Π½, ΠΎΡ‚Π²Π΅Ρ‚ для Π΄ΠΎΠΌΠ΅Π½Π° всСгда :

домСн «всС x ».

Π”ΠΈΠ°ΠΏΠ°Π·ΠΎΠ½ Π±ΡƒΠ΄Π΅Ρ‚ Π²Π°Ρ€ΡŒΠΈΡ€ΠΎΠ²Π°Ρ‚ΡŒΡΡ ΠΎΡ‚ ΠΌΠ½ΠΎΠ³ΠΎΡ‡Π»Π΅Π½Π° ΠΊ ΠΌΠ½ΠΎΠ³ΠΎΡ‡Π»Π΅Π½Ρƒ, ΠΈ ΠΎΠ½ΠΈ, вСроятно, Π΄Π°ΠΆΠ΅ Π½Π΅ спросят, Π½ΠΎ ΠΊΠΎΠ³Π΄Π° ΠΎΠ½ΠΈ это ΡΠ΄Π΅Π»Π°ΡŽΡ‚, я ΡΠΌΠΎΡ‚Ρ€ΡŽ Π½Π° ΠΊΠ°Ρ€Ρ‚ΠΈΠ½ΠΊΡƒ:

Π“Ρ€Π°Ρ„ΠΈΠΊ ΠΈΠ΄Π΅Ρ‚ Ρ‚ΠΎΠ»ΡŒΠΊΠΎ Π΄ΠΎ y = 4, Π½ΠΎ это Π±ΡƒΠ΄Π΅Ρ‚ Ρ‚Π°ΠΊ Π½ΠΈΠ·ΠΊΠΎ, ΠΊΠ°ΠΊ я Ρ…ΠΎΡ‡Ρƒ. Π’ΠΎΠ³Π΄Π°:

Π”ΠΈΠ°ΠΏΠ°Π·ΠΎΠ½ «Π²ΡΠ΅ y ≀ 4″.

Β 


URL: https://www.purplemath.com/modules/fcns2.htm

Π’Ρ‹ ΠΌΠΎΠΆΠ΅Ρ‚Π΅ ΠΈΡΠΏΠΎΠ»ΡŒΠ·ΠΎΠ²Π°Ρ‚ΡŒ Π²ΠΈΠ΄ΠΆΠ΅Ρ‚ Mathway Π½ΠΈΠΆΠ΅, Ρ‡Ρ‚ΠΎΠ±Ρ‹ ΠΏΠΎΠΏΡ€Π°ΠΊΡ‚ΠΈΠΊΠΎΠ²Π°Ρ‚ΡŒΡΡ Π² поискС Π΄ΠΎΠΌΠ΅Π½ΠΎΠ² ΠΈ Π΄ΠΈΠ°ΠΏΠ°Π·ΠΎΠ½ΠΎΠ² Ρ„ΡƒΠ½ΠΊΡ†ΠΈΠΉ.. ΠŸΠΎΠΏΡ€ΠΎΠ±ΡƒΠΉΡ‚Π΅ Π²Π²Π΅Π΄Π΅Π½Π½ΠΎΠ΅ ΡƒΠΏΡ€Π°ΠΆΠ½Π΅Π½ΠΈΠ΅ ΠΈΠ»ΠΈ Π²Π²Π΅Π΄ΠΈΡ‚Π΅ своС собствСнноС ΡƒΠΏΡ€Π°ΠΆΠ½Π΅Π½ΠΈΠ΅. Π—Π°Ρ‚Π΅ΠΌ Π½Π°ΠΆΠΌΠΈΡ‚Π΅ ΠΊΠ½ΠΎΠΏΠΊΡƒ ΠΈ Π²Ρ‹Π±Π΅Ρ€ΠΈΡ‚Π΅ «Найти Π΄ΠΎΠΌΠ΅Π½ ΠΈ Π΄ΠΈΠ°ΠΏΠ°Π·ΠΎΠ½Β», Ρ‡Ρ‚ΠΎΠ±Ρ‹ ΡΡ€Π°Π²Π½ΠΈΡ‚ΡŒ свой ΠΎΡ‚Π²Π΅Ρ‚ с ΠΎΡ‚Π²Π΅Ρ‚ΠΎΠΌ Mathway.

ΠŸΠΎΠΆΠ°Π»ΡƒΠΉΡΡ‚Π°, ΠΏΡ€ΠΈΠΌΠΈΡ‚Π΅ ΠΊΡƒΠΊΠΈ-Ρ„Π°ΠΉΠ»Ρ‹ настроСк, Ρ‡Ρ‚ΠΎΠ±Ρ‹ Π²ΠΊΠ»ΡŽΡ‡ΠΈΡ‚ΡŒ этот Π²ΠΈΠ΄ΠΆΠ΅Ρ‚.

(НаТмитС «НаТмитС, Ρ‡Ρ‚ΠΎΠ±Ρ‹ ΠΏΡ€ΠΎΡΠΌΠΎΡ‚Ρ€Π΅Ρ‚ΡŒ шаги», Ρ‡Ρ‚ΠΎΠ±Ρ‹ ΠΏΠ΅Ρ€Π΅ΠΉΡ‚ΠΈ нСпосрСдствСнно Π½Π° сайт Mathway для ΠΏΠ»Π°Ρ‚Π½ΠΎΠ³ΠΎ обновлСния.)

0002

Напомним, Ρ‡Ρ‚ΠΎ Π΄ΠΎΠΌΠ΅Π½ Ρ„ΡƒΠ½ΠΊΡ†ΠΈΠΈ прСдставляСт собой Π½Π°Π±ΠΎΡ€ Π²Ρ…ΠΎΠ΄Π½Ρ‹Ρ… ΠΈΠ»ΠΈ Икс -значСния, для ΠΊΠΎΡ‚ΠΎΡ€Ρ‹Ρ… ΠΎΠΏΡ€Π΅Π΄Π΅Π»Π΅Π½Π° функция, Π° Π΄ΠΈΠ°ΠΏΠ°Π·ΠΎΠ½ это Π½Π°Π±ΠΎΡ€ всСх Π²Ρ‹Ρ…ΠΎΠ΄Π½Ρ‹Ρ… ΠΈΠ»ΠΈ Ρƒ -значСния, ΠΊΠΎΡ‚ΠΎΡ€Ρ‹Π΅ ΠΏΡ€ΠΈΠ½ΠΈΠΌΠ°Π΅Ρ‚ функция.

ΠŸΡ€ΠΎΡΡ‚Π°Ρ ΡΠΊΡΠΏΠΎΠ½Π΅Π½Ρ†ΠΈΠ°Π»ΡŒΠ½Π°Ρ функция, Π½Π°ΠΏΡ€ΠΈΠΌΠ΅Ρ€ Ρ„ ( Икс ) Π·Π½Π°ΠΊ Ρ€Π°Π²Π½ΠΎ 2 Икс ΠΈΠΌΠ΅Π΅Ρ‚ своСй ΠΎΠ±Π»Π°ΡΡ‚ΡŒΡŽ всю Ρ€Π΅Π°Π»ΡŒΠ½ΡƒΡŽ линию. Но Π΅Π³ΠΎ Π΄ΠΈΠ°ΠΏΠ°Π·ΠΎΠ½ Ρ‚ΠΎΠ»ΡŒΠΊΠΎ ΠΏΠΎΠ»ΠΎΠΆΠΈΡ‚Π΅Π»ΡŒΠ½Ρ‹ΠΉ вСщСствСнныС числа, Ρƒ > 0 : Ρ„ ( Икс ) Π½ΠΈΠΊΠΎΠ³Π΄Π° Π½Π΅ ΠΏΡ€ΠΈΠ½ΠΈΠΌΠ°Π΅Ρ‚ ΠΎΡ‚Ρ€ΠΈΡ†Π°Ρ‚Π΅Π»ΡŒΠ½ΠΎΠ΅ Π·Π½Π°Ρ‡Π΅Π½ΠΈΠ΅. ΠšΡ€ΠΎΠΌΠ΅ Ρ‚ΠΎΠ³ΠΎ, ΠΎΠ½ Π½ΠΈΠΊΠΎΠ³Π΄Π° Π½Π΅ достигаСт 0 , хотя асимптотичСски приблиТаСтся ΠΊ Икс ΠΈΠ΄Π΅Ρ‚ ΠΊ βˆ’ ∞ .

Если ΠΌΡ‹ Π·Π°ΠΌΠ΅Π½ΠΈΠΌ Икс с βˆ’ Икс Ρ‡Ρ‚ΠΎΠ±Ρ‹ ΠΏΠΎΠ»ΡƒΡ‡ΠΈΡ‚ΡŒ ΡƒΡ€Π°Π²Π½Π΅Π½ΠΈΠ΅ Π³Ρ€Π°ΠΌΠΌ ( Икс ) Π·Π½Π°ΠΊ Ρ€Π°Π²Π½ΠΎ 2 βˆ’ Икс , Π³Ρ€Π°Ρ„ΠΈΠΊ отраТаСтся Π²ΠΎΠΊΡ€ΡƒΠ³ Ρƒ -axis, Π½ΠΎ Π΄ΠΎΠΌΠ΅Π½ ΠΈ Π΄ΠΈΠ°ΠΏΠ°Π·ΠΎΠ½ Π½Π΅ ΠΌΠ΅Π½ΡΡŽΡ‚ΡΡ:

Если ΠΌΡ‹ поставим ΠΏΠ΅Ρ€Π΅Π΄ собой Π·Π½Π°ΠΊ минус, Ρ‡Ρ‚ΠΎΠ±Ρ‹ ΠΏΠΎΠ»ΡƒΡ‡ΠΈΡ‚ΡŒ ΡƒΡ€Π°Π²Π½Π΅Π½ΠΈΠ΅ час ( Икс ) Π·Π½Π°ΠΊ Ρ€Π°Π²Π½ΠΎ βˆ’ 2 Икс , Π³Ρ€Π°Ρ„ΠΈΠΊ отраТаСтся Π²ΠΎΠΊΡ€ΡƒΠ³ Икс -ось. Π£ нас всС Π΅Ρ‰Π΅ Π΅ΡΡ‚ΡŒ цСлая Π΄Π΅ΠΉΡΡ‚Π²ΠΈΡ‚Π΅Π»ΡŒΠ½Π°Ρ линия Π² качСствС нашСго Π΄ΠΎΠΌΠ΅Π½Π°, Π½ΠΎ Π΄ΠΈΠ°ΠΏΠ°Π·ΠΎΠ½ Ρ‚Π΅ΠΏΠ΅Ρ€ΡŒ состоит ΠΈΠ· ΠΎΡ‚Ρ€ΠΈΡ†Π°Ρ‚Π΅Π»ΡŒΠ½Ρ‹Ρ… чисСл, Ρƒ < 0 .

Π’Π΅ΠΏΠ΅Ρ€ΡŒ рассмотрим Ρ„ΡƒΠ½ΠΊΡ†ΠΈΡŽ Ρ„ ( Икс ) Π·Π½Π°ΠΊ Ρ€Π°Π²Π½ΠΎ ( βˆ’ 2 ) Икс . Когда Икс Π·Π½Π°ΠΊ Ρ€Π°Π²Π½ΠΎ 1 2 , Ρƒ Π΄ΠΎΠ»ΠΆΠ½ΠΎ Π±Ρ‹Ρ‚ΡŒ комплСксным числом, Ρ‚Π°ΠΊ Ρ‡Ρ‚ΠΎ всС услоТняСтся. Для этого ΡƒΡ€ΠΎΠΊΠ° Π½Π°ΠΌ потрСбуСтся, Ρ‡Ρ‚ΠΎΠ±Ρ‹ наши базисы Π±Ρ‹Π»ΠΈ ΠΏΠΎΠ»ΠΎΠΆΠΈΡ‚Π΅Π»ΡŒΠ½Ρ‹ΠΌΠΈ Π½Π° Π΄Π°Π½Π½Ρ‹ΠΉ ΠΌΠΎΠΌΠ΅Π½Ρ‚, Ρ‡Ρ‚ΠΎΠ±Ρ‹ ΠΌΡ‹ ΠΌΠΎΠ³Π»ΠΈ ΠΎΡΡ‚Π°Π²Π°Ρ‚ΡŒΡΡ Π² Ρ€Π΅Π°Π»ΡŒΠ½ΠΎΠΌ ΠΌΠΈΡ€Π΅.

Π’ ΠΎΠ±Ρ‰Π΅ΠΌ случаС Π³Ρ€Π°Ρ„ΠΈΠΊ основной ΡΠΊΡΠΏΠΎΠ½Π΅Π½Ρ†ΠΈΠ°Π»ΡŒΠ½ΠΎΠΉ Ρ„ΡƒΠ½ΠΊΡ†ΠΈΠΈ Ρƒ Π·Π½Π°ΠΊ Ρ€Π°Π²Π½ΠΎ Π° Икс ΠΊΠ°ΠΏΠ»ΠΈ ΠΎΡ‚ ∞ ΠΊ 0 ΠΊΠΎΠ³Π΄Π° 0 < Π° < 1 Π² качСствС Икс Π²Π°Ρ€ΡŒΠΈΡ€ΡƒΠ΅Ρ‚ΡΡ ΠΎΡ‚ βˆ’ ∞ ΠΊ ∞ ΠΈ поднимаСтся ΠΈΠ· 0 ΠΊ ∞ ΠΊΠΎΠ³Π΄Π° Π° > 1 .

Π­ΠΊΡΠΏΠΎΠ½Π΅Π½Ρ†ΠΈΠ°Π»ΡŒΠ½Π°Ρ функция Ρƒ Π·Π½Π°ΠΊ Ρ€Π°Π²Π½ΠΎ Π° Икс , ΠΌΠΎΠΆΠ½ΠΎ ΡΠ΄Π²ΠΈΠ½ΡƒΡ‚ΡŒ ΠΊ Π΅Π΄ΠΈΠ½ΠΈΡ†Ρ‹ ΠΏΠΎ Π²Π΅Ρ€Ρ‚ΠΈΠΊΠ°Π»ΠΈ ΠΈ час Π΅Π΄ΠΈΠ½ΠΈΡ†Ρ‹ ΠΏΠΎ Π³ΠΎΡ€ΠΈΠ·ΠΎΠ½Ρ‚Π°Π»ΠΈ с ΡƒΡ€Π°Π²Π½Π΅Π½ΠΈΠ΅ΠΌ Ρƒ Π·Π½Π°ΠΊ Ρ€Π°Π²Π½ΠΎ Π° ( Икс + час ) + ΠΊ . Π’ΠΎΠ³Π΄Π° ΠΎΠ±Π»Π°ΡΡ‚ΡŒ опрСдСлСния Ρ„ΡƒΠ½ΠΊΡ†ΠΈΠΈ остаСтся Π½Π΅ΠΈΠ·ΠΌΠ΅Π½Π½ΠΎΠΉ, Π° Π΄ΠΈΠ°ΠΏΠ°Π·ΠΎΠ½ становится { Ρƒ Π΅ ℝ | Ρƒ > час } .

ΠŸΡ€ΠΈΠΌΠ΅Ρ€ 1:

НайдитС ΠΎΠ±Π»Π°ΡΡ‚ΡŒ опрСдСлСния ΠΈ Π΄ΠΈΠ°ΠΏΠ°Π·ΠΎΠ½ Ρ„ΡƒΠ½ΠΊΡ†ΠΈΠΈ Ρƒ Π·Π½Π°ΠΊ Ρ€Π°Π²Π½ΠΎ 3 Икс + 2 .

Π“Ρ€Π°Ρ„ΠΈΠΊ Ρ„ΡƒΠ½ΠΊΡ†ΠΈΠΈ Π½Π° ΠΊΠΎΠΎΡ€Π΄ΠΈΠ½Π°Ρ‚Π½ΠΎΠΉ плоскости.

Π“Ρ€Π°Ρ„ΠΈΠΊ Π½Π΅ Ρ‡Ρ‚ΠΎ ΠΈΠ½ΠΎΠ΅, ΠΊΠ°ΠΊ Π³Ρ€Π°Ρ„ΠΈΠΊ Ρƒ Π·Π½Π°ΠΊ Ρ€Π°Π²Π½ΠΎ 3 Икс ΠΏΠ΅Ρ€Π΅Π²Π΅Π΄Π΅Π½ΠΎ 2 Π΅Π΄ΠΈΠ½ΠΈΡ†Ρ‹ Π²Π»Π΅Π²ΠΎ.

Ѐункция ΠΎΠΏΡ€Π΅Π΄Π΅Π»Π΅Π½Π° для всСх Π΄Π΅ΠΉΡΡ‚Π²ΠΈΡ‚Π΅Π»ΡŒΠ½Ρ‹Ρ… чисСл. Π˜Ρ‚Π°ΠΊ, ΠΎΠ±Π»Π°ΡΡ‚ΡŒΡŽ опрСдСлСния Ρ„ΡƒΠ½ΠΊΡ†ΠΈΠΈ являСтся мноТСство Π΄Π΅ΠΉΡΡ‚Π²ΠΈΡ‚Π΅Π»ΡŒΠ½Ρ‹Ρ… чисСл.

Π’ качСствС Икс ΠΊΠ°ΠΊ ΠΏΡ€Π°Π²ΠΈΠ»ΠΎ ∞ , Π·Π½Π°Ρ‡Π΅Π½ΠΈΠ΅ Ρ„ΡƒΠ½ΠΊΡ†ΠΈΠΈ Ρ‚Π°ΠΊΠΆΠ΅ стрСмится ΠΊ ∞ ΠΈ Π² качСствС Икс ΠΊΠ°ΠΊ ΠΏΡ€Π°Π²ΠΈΠ»ΠΎ βˆ’ ∞ , функция приблиТаСтся ΠΊ Икс -ось, Π½ΠΎ Π½ΠΈΠΊΠΎΠ³Π΄Π° Π½Π΅ касаСтся Π΅Π΅.

Π‘Π»Π΅Π΄ΠΎΠ²Π°Ρ‚Π΅Π»ΡŒΠ½ΠΎ, Π΄ΠΈΠ°ΠΏΠ°Π·ΠΎΠ½ Ρ„ΡƒΠ½ΠΊΡ†ΠΈΠΈ прСдставляСт собой Π½Π°Π±ΠΎΡ€ Π΄Π΅ΠΉΡΡ‚Π²ΠΈΡ‚Π΅Π»ΡŒΠ½Ρ‹Ρ… ΠΏΠΎΠ»ΠΎΠΆΠΈΡ‚Π΅Π»ΡŒΠ½Ρ‹Ρ… чисСл ΠΈΠ»ΠΈ { Икс Π΅ ℝ | Икс > 0 } .

ΠŸΡ€ΠΈΠΌΠ΅Ρ€ 2:

НайдитС ΠΎΠ±Π»Π°ΡΡ‚ΡŒ опрСдСлСния ΠΈ Π΄ΠΈΠ°ΠΏΠ°Π·ΠΎΠ½ Ρ„ΡƒΠ½ΠΊΡ†ΠΈΠΈ Ρƒ Π·Π½Π°ΠΊ Ρ€Π°Π²Π½ΠΎ ( 1 4 ) 2 Икс .

Π“Ρ€Π°Ρ„ΠΈΠΊ Ρ„ΡƒΠ½ΠΊΡ†ΠΈΠΈ Π½Π° ΠΊΠΎΠΎΡ€Π΄ΠΈΠ½Π°Ρ‚Π½ΠΎΠΉ плоскости.

Π“Ρ€Π°Ρ„ΠΈΠΊ Π½Π΅ Ρ‡Ρ‚ΠΎ ΠΈΠ½ΠΎΠ΅, ΠΊΠ°ΠΊ Π³Ρ€Π°Ρ„ΠΈΠΊ Ρƒ Π·Π½Π°ΠΊ Ρ€Π°Π²Π½ΠΎ ( 1 4 ) Икс сТатый Π² Ρ€Π°Π·Ρ‹ 2 .

Ѐункция ΠΎΠΏΡ€Π΅Π΄Π΅Π»Π΅Π½Π° для всСх Π΄Π΅ΠΉΡΡ‚Π²ΠΈΡ‚Π΅Π»ΡŒΠ½Ρ‹Ρ… чисСл. Π˜Ρ‚Π°ΠΊ, ΠΎΠ±Π»Π°ΡΡ‚ΡŒΡŽ опрСдСлСния Ρ„ΡƒΠ½ΠΊΡ†ΠΈΠΈ являСтся мноТСство Π΄Π΅ΠΉΡΡ‚Π²ΠΈΡ‚Π΅Π»ΡŒΠ½Ρ‹Ρ… чисСл.

Π’ качСствС Икс ΠΊΠ°ΠΊ ΠΏΡ€Π°Π²ΠΈΠ»ΠΎ ∞ , Π·Π½Π°Ρ‡Π΅Π½ΠΈΠ΅ Ρ„ΡƒΠ½ΠΊΡ†ΠΈΠΈ стрСмится ΠΊ Π½ΡƒΠ»ΡŽ, Π° Π³Ρ€Π°Ρ„ΠΈΠΊ приблиТаСтся Икс -ось, Π½ΠΎ Π½ΠΈΠΊΠΎΠ³Π΄Π° Π½Π΅ касаСтся Π΅Π΅.

Π’ качСствС Икс ΠΊΠ°ΠΊ ΠΏΡ€Π°Π²ΠΈΠ»ΠΎ βˆ’ ∞ , функция Ρ‚Π°ΠΊΠΆΠ΅ стрСмится ΠΊ ∞ .

Π‘Π»Π΅Π΄ΠΎΠ²Π°Ρ‚Π΅Π»ΡŒΠ½ΠΎ, Π΄ΠΈΠ°ΠΏΠ°Π·ΠΎΠ½ Ρ„ΡƒΠ½ΠΊΡ†ΠΈΠΈ прСдставляСт собой Π½Π°Π±ΠΎΡ€ Π΄Π΅ΠΉΡΡ‚Π²ΠΈΡ‚Π΅Π»ΡŒΠ½Ρ‹Ρ… ΠΏΠΎΠ»ΠΎΠΆΠΈΡ‚Π΅Π»ΡŒΠ½Ρ‹Ρ… чисСл ΠΈΠ»ΠΈ { Ρƒ Π΅ ℝ | Ρƒ > 0 } .

ΠžΠ±Ρ€Π°Ρ‚Π½Π°Ρ ΡΠΊΡΠΏΠΎΠ½Π΅Π½Ρ†ΠΈΠ°Π»ΡŒΠ½Π°Ρ функция являСтся логарифмичСской Ρ„ΡƒΠ½ΠΊΡ†ΠΈΠ΅ΠΉ.

ΠŸΡ€ΠΎΡΡ‚ΠΎΠΉ логарифмичСская функция Ρƒ Π·Π½Π°ΠΊ Ρ€Π°Π²Π½ΠΎ ΠΆΡƒΡ€Π½Π°Π» 2 Икс ΠΊΡƒΠ΄Π° Икс > 0 эквивалСнтна Ρ„ΡƒΠ½ΠΊΡ†ΠΈΠΈ Икс Π·Π½Π°ΠΊ Ρ€Π°Π²Π½ΠΎ 2 Ρƒ . Π’ΠΎ Π΅ΡΡ‚ΡŒ, Ρƒ Π·Π½Π°ΠΊ Ρ€Π°Π²Π½ΠΎ ΠΆΡƒΡ€Π½Π°Π» 2 Икс являСтся ΠΎΠ±Ρ€Π°Ρ‚Π½ΠΎΠΉ Ρ„ΡƒΠ½ΠΊΡ†ΠΈΠ΅ΠΉ Ρƒ Π·Π½Π°ΠΊ Ρ€Π°Π²Π½ΠΎ 2 Икс .

Ѐункция Ρƒ Π·Π½Π°ΠΊ Ρ€Π°Π²Π½ΠΎ ΠΆΡƒΡ€Π½Π°Π» 2 Икс ΠΈΠΌΠ΅Π΅Ρ‚ ΠΎΠ±Π»Π°ΡΡ‚ΡŒ опрСдСлСния мноТСства ΠΏΠΎΠ»ΠΎΠΆΠΈΡ‚Π΅Π»ΡŒΠ½Ρ‹Ρ… Π΄Π΅ΠΉΡΡ‚Π²ΠΈΡ‚Π΅Π»ΡŒΠ½Ρ‹Ρ… чисСл ΠΈ Π΄ΠΈΠ°ΠΏΠ°Π·ΠΎΠ½ мноТСства Π΄Π΅ΠΉΡΡ‚Π²ΠΈΡ‚Π΅Π»ΡŒΠ½Ρ‹Ρ… чисСл.

ΠŸΠΎΠΌΠ½ΠΈΡ‚Π΅, Ρ‡Ρ‚ΠΎ, ΠΏΠΎΡΠΊΠΎΠ»ΡŒΠΊΡƒ логарифмичСская функция являСтся ΠΎΠ±Ρ€Π°Ρ‚Π½ΠΎΠΉ ΡΠΊΡΠΏΠΎΠ½Π΅Π½Ρ†ΠΈΠ°Π»ΡŒΠ½ΠΎΠΉ Ρ„ΡƒΠ½ΠΊΡ†ΠΈΠΈ, ΠΎΠ±Π»Π°ΡΡ‚ΡŒΡŽ опрСдСлСния логарифмичСской Ρ„ΡƒΠ½ΠΊΡ†ΠΈΠΈ являСтся Π΄ΠΈΠ°ΠΏΠ°Π·ΠΎΠ½ ΡΠΊΡΠΏΠΎΠ½Π΅Π½Ρ†ΠΈΠ°Π»ΡŒΠ½ΠΎΠΉ Ρ„ΡƒΠ½ΠΊΡ†ΠΈΠΈ, ΠΈ Π½Π°ΠΎΠ±ΠΎΡ€ΠΎΡ‚.

Π’ ΠΎΠ±Ρ‰Π΅ΠΌ случаС функция Ρƒ Π·Π½Π°ΠΊ Ρ€Π°Π²Π½ΠΎ ΠΆΡƒΡ€Π½Π°Π» Π± Икс ΠΊΡƒΠ΄Π° Π± , Икс > 0 Π° Ρ‚Π°ΠΊΠΆΠ΅ Π± β‰  1 являСтся Π½Π΅ΠΏΡ€Π΅Ρ€Ρ‹Π²Π½ΠΎΠΉ ΠΈ ΠΎΠ΄Π½ΠΎΠ·Π½Π°Ρ‡Π½ΠΎΠΉ Ρ„ΡƒΠ½ΠΊΡ†ΠΈΠ΅ΠΉ. ΠžΠ±Ρ€Π°Ρ‚ΠΈΡ‚Π΅ Π²Π½ΠΈΠΌΠ°Π½ΠΈΠ΅, Ρ‡Ρ‚ΠΎ логарифмичСская функция Π½Π΅ ΠΎΠΏΡ€Π΅Π΄Π΅Π»Π΅Π½ΠΎ для ΠΎΡ‚Ρ€ΠΈΡ†Π°Ρ‚Π΅Π»ΡŒΠ½Ρ‹Ρ… чисСл ΠΈΠ»ΠΈ для нуля. Π“Ρ€Π°Ρ„ΠΈΠΊ Ρ„ΡƒΠ½ΠΊΡ†ΠΈΠΈ приблиТаСтся ΠΊ Ρƒ -ось ΠΊΠ°ΠΊ Икс ΠΊΠ°ΠΊ ΠΏΡ€Π°Π²ΠΈΠ»ΠΎ ∞ , Π½ΠΎ Π½ΠΈΠΊΠΎΠ³Π΄Π° Π½Π΅ касаСтся Π΅Π³ΠΎ.

Π‘Π»Π΅Π΄ΠΎΠ²Π°Ρ‚Π΅Π»ΡŒΠ½ΠΎ, ΠΎΠ±Π»Π°ΡΡ‚ΡŒ опрСдСлСния логарифмичСской Ρ„ΡƒΠ½ΠΊΡ†ΠΈΠΈ Ρƒ Π·Π½Π°ΠΊ Ρ€Π°Π²Π½ΠΎ ΠΆΡƒΡ€Π½Π°Π» Π± Икс β€” мноТСство ΠΏΠΎΠ»ΠΎΠΆΠΈΡ‚Π΅Π»ΡŒΠ½Ρ‹Ρ… Π΄Π΅ΠΉΡΡ‚Π²ΠΈΡ‚Π΅Π»ΡŒΠ½Ρ‹Ρ… чисСл, Π° Π΄ΠΈΠ°ΠΏΠ°Π·ΠΎΠ½ β€” мноТСство Π΄Π΅ΠΉΡΡ‚Π²ΠΈΡ‚Π΅Π»ΡŒΠ½Ρ‹Ρ… чисСл.

Ѐункция Π²ΠΎΠ·Π½ΠΈΠΊΠ°Π΅Ρ‚ ΠΈΠ· βˆ’ ∞ ΠΊ ∞ Π² качСствС Икс увСличиваСтся, Ссли Π± > 1 ΠΈ ΠΏΠ°Π΄Π°Π΅Ρ‚ с ∞ ΠΊ βˆ’ ∞ Π² качСствС Икс увСличиваСтся, Ссли 0 < Π± < 1 .

ЛогарифмичСская функция, Ρƒ Π·Π½Π°ΠΊ Ρ€Π°Π²Π½ΠΎ ΠΆΡƒΡ€Π½Π°Π» Π± Икс , ΠΌΠΎΠΆΠ½ΠΎ ΡΠ΄Π²ΠΈΠ½ΡƒΡ‚ΡŒ ΠΊ Π΅Π΄ΠΈΠ½ΠΈΡ†Ρ‹ ΠΏΠΎ Π²Π΅Ρ€Ρ‚ΠΈΠΊΠ°Π»ΠΈ ΠΈ час Π΅Π΄ΠΈΠ½ΠΈΡ†Ρ‹ ΠΏΠΎ Π³ΠΎΡ€ΠΈΠ·ΠΎΠ½Ρ‚Π°Π»ΠΈ с ΡƒΡ€Π°Π²Π½Π΅Π½ΠΈΠ΅ΠΌ Ρƒ Π·Π½Π°ΠΊ Ρ€Π°Π²Π½ΠΎ ΠΆΡƒΡ€Π½Π°Π» Π± ( Икс + час ) + ΠΊ . Π’ΠΎΠ³Π΄Π° ΠΎΠ±Π»Π°ΡΡ‚ΡŒ опрСдСлСния Ρ„ΡƒΠ½ΠΊΡ†ΠΈΠΈ становится { Икс Π΅ ℝ | Икс > βˆ’ час } . Однако ассортимСнт остаСтся ΠΏΡ€Π΅ΠΆΠ½ΠΈΠΌ.

ΠŸΡ€ΠΈΠΌΠ΅Ρ€ 3:

НайдитС ΠΎΠ±Π»Π°ΡΡ‚ΡŒ опрСдСлСния ΠΈ Π΄ΠΈΠ°ΠΏΠ°Π·ΠΎΠ½ Ρ„ΡƒΠ½ΠΊΡ†ΠΈΠΈ Ρƒ Π·Π½Π°ΠΊ Ρ€Π°Π²Π½ΠΎ ΠΆΡƒΡ€Π½Π°Π» ( Икс ) βˆ’ 3 .

НачСртитС Ρ„ΡƒΠ½ΠΊΡ†ΠΈΡŽ Π½Π° ΠΊΠΎΠΎΡ€Π΄ΠΈΠ½Π°Ρ‚Π½ΠΎΠΉ плоскости. ΠŸΠΎΠΌΠ½ΠΈΡ‚Π΅, Ρ‡Ρ‚ΠΎ Ссли основаниС Π½Π΅ ΠΏΠΎΠΊΠ°Π·Π°Π½ΠΎ, Ρ‚ΠΎ ΠΏΠΎΠ΄ основаниСм понимаСтся 10 .

Π“Ρ€Π°Ρ„ΠΈΠΊ Π½Π΅ Ρ‡Ρ‚ΠΎ ΠΈΠ½ΠΎΠ΅, ΠΊΠ°ΠΊ Π³Ρ€Π°Ρ„ΠΈΠΊ Ρƒ Π·Π½Π°ΠΊ Ρ€Π°Π²Π½ΠΎ ΠΆΡƒΡ€Π½Π°Π» ( Икс ) ΠΏΠ΅Ρ€Π΅Π²Π΅Π΄Π΅Π½ΠΎ 3 Π΅Π΄ΠΈΠ½ΠΈΡ† Π²Π½ΠΈΠ·.

Ѐункция ΠΎΠΏΡ€Π΅Π΄Π΅Π»Π΅Π½Π° Ρ‚ΠΎΠ»ΡŒΠΊΠΎ для ΠΏΠΎΠ»ΠΎΠΆΠΈΡ‚Π΅Π»ΡŒΠ½Ρ‹Ρ… Π΄Π΅ΠΉΡΡ‚Π²ΠΈΡ‚Π΅Π»ΡŒΠ½Ρ‹Ρ… чисСл. Π˜Ρ‚Π°ΠΊ, ΠΎΠ±Π»Π°ΡΡ‚ΡŒ опрСдСлСния Ρ„ΡƒΠ½ΠΊΡ†ΠΈΠΈ β€” это мноТСство ΠΏΠΎΠ»ΠΎΠΆΠΈΡ‚Π΅Π»ΡŒΠ½Ρ‹Ρ… Π΄Π΅ΠΉΡΡ‚Π²ΠΈΡ‚Π΅Π»ΡŒΠ½Ρ‹Ρ… чисСл ΠΈΠ»ΠΈ { Икс Π΅ ℝ | Икс > 0 } .

Ѐункция ΠΏΡ€ΠΈΠ½ΠΈΠΌΠ°Π΅Ρ‚ всС Π΄Π΅ΠΉΡΡ‚Π²ΠΈΡ‚Π΅Π»ΡŒΠ½Ρ‹Π΅ значСния ΠΈΠ· βˆ’ ∞ ΠΊ ∞ .

Π’Π°ΠΊΠΈΠΌ ΠΎΠ±Ρ€Π°Π·ΠΎΠΌ, Π΄ΠΈΠ°ΠΏΠ°Π·ΠΎΠ½ Ρ„ΡƒΠ½ΠΊΡ†ΠΈΠΈ прСдставляСт собой мноТСство Π΄Π΅ΠΉΡΡ‚Π²ΠΈΡ‚Π΅Π»ΡŒΠ½Ρ‹Ρ… чисСл.

Π³. ΠŸΡ€ΠΈΠΌΠ΅Ρ€ 4:

НайдитС ΠΎΠ±Π»Π°ΡΡ‚ΡŒ опрСдСлСния ΠΈ Π΄ΠΈΠ°ΠΏΠ°Π·ΠΎΠ½ Ρ„ΡƒΠ½ΠΊΡ†ΠΈΠΈ Ρƒ Π·Π½Π°ΠΊ Ρ€Π°Π²Π½ΠΎ ΠΆΡƒΡ€Π½Π°Π» 3 ( Икс βˆ’ 2 ) + 4 .

Π“Ρ€Π°Ρ„ΠΈΠΊ Ρ„ΡƒΠ½ΠΊΡ†ΠΈΠΈ Π½Π° ΠΊΠΎΠΎΡ€Π΄ΠΈΠ½Π°Ρ‚Π½ΠΎΠΉ плоскости.

Π“Ρ€Π°Ρ„ΠΈΠΊ Π½Π΅ Ρ‡Ρ‚ΠΎ ΠΈΠ½ΠΎΠ΅, ΠΊΠ°ΠΊ Π³Ρ€Π°Ρ„ΠΈΠΊ Ρƒ Π·Π½Π°ΠΊ Ρ€Π°Π²Π½ΠΎ ΠΆΡƒΡ€Π½Π°Π» 3 ( Икс ) ΠΏΠ΅Ρ€Π΅Π²Π΅Π΄Π΅Π½ΠΎ 2 Π΅Π΄ΠΈΠ½ΠΈΡ†Ρ‹ Π²ΠΏΡ€Π°Π²ΠΎ ΠΈ 4 Π΅Π΄ΠΈΠ½ΠΈΡ† Π²Π²Π΅Ρ€Ρ….

Π’ качСствС Икс ΠΊΠ°ΠΊ ΠΏΡ€Π°Π²ΠΈΠ»ΠΎ 2 , функция приблиТаСтся ΠΊ прямой Икс Π·Π½Π°ΠΊ Ρ€Π°Π²Π½ΠΎ 2 Π½ΠΎ Π½ΠΈΠΊΠΎΠ³Π΄Π° Π½Π΅ прикасаСтся ΠΊ Π½Π΅ΠΌΡƒ. Π’ качСствС Икс ΠΊΠ°ΠΊ ΠΏΡ€Π°Π²ΠΈΠ»ΠΎ ∞ Π·Π½Π°Ρ‡Π΅Π½ΠΈΠ΅ Ρ„ΡƒΠ½ΠΊΡ†ΠΈΠΈ Ρ‚Π°ΠΊΠΆΠ΅ стрСмится ΠΊ ∞ . Π’ΠΎ Π΅ΡΡ‚ΡŒ функция ΠΎΠΏΡ€Π΅Π΄Π΅Π»Π΅Π½Π° для Π΄Π΅ΠΉΡΡ‚Π²ΠΈΡ‚Π΅Π»ΡŒΠ½Ρ‹Ρ… чисСл, Π±ΠΎΠ»ΡŒΡˆΠΈΡ…, Ρ‡Π΅ΠΌ 2 . Π˜Ρ‚Π°ΠΊ, ΠΎΠ±Π»Π°ΡΡ‚ΡŒ опрСдСлСния Ρ„ΡƒΠ½ΠΊΡ†ΠΈΠΈ β€” это мноТСство ΠΏΠΎΠ»ΠΎΠΆΠΈΡ‚Π΅Π»ΡŒΠ½Ρ‹Ρ… Π΄Π΅ΠΉΡΡ‚Π²ΠΈΡ‚Π΅Π»ΡŒΠ½Ρ‹Ρ… чисСл ΠΈΠ»ΠΈ { Икс Π΅ ℝ | Икс > 2 } .

Ѐункция ΠΏΡ€ΠΈΠ½ΠΈΠΌΠ°Π΅Ρ‚ всС Π΄Π΅ΠΉΡΡ‚Π²ΠΈΡ‚Π΅Π»ΡŒΠ½Ρ‹Π΅ значСния ΠΈΠ· βˆ’ ∞ ΠΊ ∞ .

Π’Π°ΠΊΠΈΠΌ ΠΎΠ±Ρ€Π°Π·ΠΎΠΌ, Π΄ΠΈΠ°ΠΏΠ°Π·ΠΎΠ½ Ρ„ΡƒΠ½ΠΊΡ†ΠΈΠΈ прСдставляСт собой мноТСство Π΄Π΅ΠΉΡΡ‚Π²ΠΈΡ‚Π΅Π»ΡŒΠ½Ρ‹Ρ… чисСл.

Π”ΠΎΠΌΠ΅Π½ ΠΈ Π΄ΠΈΠ°ΠΏΠ°Π·ΠΎΠ½ β€” ΠΈΠ· Π³Ρ€Π°Ρ„ΠΈΠΊΠ°

Π€ΡƒΠ½ΠΊΡ†ΠΈΠΈ Π² ΠΌΠ°Ρ‚Π΅ΠΌΠ°Ρ‚ΠΈΠΊΠ΅ ΠΌΠΎΠΆΠ½ΠΎ ΡΡ€Π°Π²Π½ΠΈΡ‚ΡŒ с Ρ€Π°Π±ΠΎΡ‚ΠΎΠΉ Π°Π²Ρ‚ΠΎΠΌΠ°Ρ‚Π° ΠΏΠΎ ΠΏΡ€ΠΎΠ΄Π°ΠΆΠ΅ Π³Π°Π·ΠΈΡ€ΠΎΠ²Π°Π½Π½Ρ‹Ρ… Π½Π°ΠΏΠΈΡ‚ΠΊΠΎΠ². Когда Π²Ρ‹ Π²ΠΊΠ»Π°Π΄Ρ‹Π²Π°Π΅Ρ‚Π΅ ΠΎΠΏΡ€Π΅Π΄Π΅Π»Π΅Π½Π½ΡƒΡŽ сумму Π΄Π΅Π½Π΅Π³, Π²Ρ‹ ΠΌΠΎΠΆΠ΅Ρ‚Π΅ Π²Ρ‹Π±Ρ€Π°Ρ‚ΡŒ Ρ€Π°Π·Π½Ρ‹Π΅ Ρ‚ΠΈΠΏΡ‹ Π³Π°Π·ΠΈΡ€ΠΎΠ²Π°Π½Π½Ρ‹Ρ… Π½Π°ΠΏΠΈΡ‚ΠΊΠΎΠ². Π’ΠΎΡ‡Π½ΠΎ Ρ‚Π°ΠΊ ΠΆΠ΅ для Ρ„ΡƒΠ½ΠΊΡ†ΠΈΠΉ ΠΌΡ‹ Π²Π²ΠΎΠ΄ΠΈΠΌ Ρ€Π°Π·Π½Ρ‹Π΅ числа ΠΈ Π² Ρ€Π΅Π·ΡƒΠ»ΡŒΡ‚Π°Ρ‚Π΅ ΠΏΠΎΠ»ΡƒΡ‡Π°Π΅ΠΌ Π½ΠΎΠ²Ρ‹Π΅ числа. Π”ΠΎΠΌΠ΅Π½ ΠΈ Π΄ΠΈΠ°ΠΏΠ°Π·ΠΎΠ½ ΡΠ²Π»ΡΡŽΡ‚ΡΡ основными аспСктами Ρ„ΡƒΠ½ΠΊΡ†ΠΈΠΉ. Π’Ρ‹ ΠΌΠΎΠΆΠ΅Ρ‚Π΅ ΠΈΡΠΏΠΎΠ»ΡŒΠ·ΠΎΠ²Π°Ρ‚ΡŒ Ρ‡Π΅Ρ‚Π²Π΅Ρ€Ρ‚ΠΈ ΠΈ ΠΎΠ΄Π½ΠΎΠ΄ΠΎΠ»Π»Π°Ρ€ΠΎΠ²Ρ‹Π΅ ΠΊΡƒΠΏΡŽΡ€Ρ‹, Ρ‡Ρ‚ΠΎΠ±Ρ‹ ΠΊΡƒΠΏΠΈΡ‚ΡŒ ΡΠΎΠ΄ΠΎΠ²ΡƒΡŽ. Машина Π½Π΅ даст Π²Π°ΠΌ Π½ΠΈΠΊΠ°ΠΊΠΎΠ³ΠΎ вкуса Π³Π°Π·ΠΈΡ€ΠΎΠ²ΠΊΠΈ, Ссли Π²Ρ‹ Π²Π²Π΅Π΄Π΅Ρ‚Π΅ ΠΏΠ΅Π½Π½ΠΈ. Π‘Π»Π΅Π΄ΠΎΠ²Π°Ρ‚Π΅Π»ΡŒΠ½ΠΎ, Π΄ΠΎΠΌΠ΅Π½ прСдставляСт Π²Ρ…ΠΎΠ΄Π½Ρ‹Π΅ Π΄Π°Π½Π½Ρ‹Π΅, ΠΊΠΎΡ‚ΠΎΡ€Ρ‹Π΅ ΠΌΡ‹ ΠΌΠΎΠΆΠ΅ΠΌ здСсь ΠΈΠΌΠ΅Ρ‚ΡŒ, Ρ‚ΠΎ Π΅ΡΡ‚ΡŒ ΠΌΠΎΠ½Π΅Ρ‚Ρ‹ Π² Ρ‡Π΅Ρ‚Π²Π΅Ρ€Ρ‚Π°ΠΊΠ΅ ΠΈ ΠΎΠ΄Π½ΠΎΠ΄ΠΎΠ»Π»Π°Ρ€ΠΎΠ²Ρ‹Π΅ ΠΊΡƒΠΏΡŽΡ€Ρ‹. НСзависимо ΠΎΡ‚ Ρ‚ΠΎΠ³ΠΎ, ΠΊΠ°ΠΊΡƒΡŽ сумму Π²Ρ‹ Π·Π°ΠΏΠ»Π°Ρ‚ΠΈΡ‚Π΅, Π²Ρ‹ Π½Π΅ ΠΏΠΎΠ»ΡƒΡ‡ΠΈΡ‚Π΅ Ρ‡ΠΈΠ·Π±ΡƒΡ€Π³Π΅Ρ€ ΠΈΠ· Π°Π²Ρ‚ΠΎΠΌΠ°Ρ‚Π° с Π³Π°Π·ΠΈΡ€ΠΎΠ²ΠΊΠΎΠΉ. Π’Π°ΠΊΠΈΠΌ ΠΎΠ±Ρ€Π°Π·ΠΎΠΌ, Π΄ΠΈΠ°ΠΏΠ°Π·ΠΎΠ½ β€” это Π²ΠΎΠ·ΠΌΠΎΠΆΠ½Ρ‹Π΅ Π²Ρ‹Ρ…ΠΎΠ΄Ρ‹, ΠΊΠΎΡ‚ΠΎΡ€Ρ‹Π΅ ΠΌΡ‹ ΠΌΠΎΠΆΠ΅ΠΌ здСсь ΠΏΠΎΠ»ΡƒΡ‡ΠΈΡ‚ΡŒ, Ρ‚ΠΎ Π΅ΡΡ‚ΡŒ вкус Π³Π°Π·ΠΈΡ€ΠΎΠ²Π°Π½Π½Ρ‹Ρ… Π½Π°ΠΏΠΈΡ‚ΠΊΠΎΠ² Π² машинС. Π”Π°Π²Π°ΠΉΡ‚Π΅ научимся Π½Π°Ρ…ΠΎΠ΄ΠΈΡ‚ΡŒ ΠΎΠ±Π»Π°ΡΡ‚ΡŒ опрСдСлСния ΠΈ ΠΎΠ±Π»Π°ΡΡ‚ΡŒ Π·Π½Π°Ρ‡Π΅Π½ΠΈΠΉ Π·Π°Π΄Π°Π½Π½ΠΎΠΉ Ρ„ΡƒΠ½ΠΊΡ†ΠΈΠΈ, Π° Ρ‚Π°ΠΊΠΆΠ΅ ΠΎΡ‚ΠΎΠ±Ρ€Π°ΠΆΠ°Ρ‚ΡŒ ΠΈΡ… Π½Π° Π³Ρ€Π°Ρ„ΠΈΠΊΠ΅.

1. Π§Ρ‚ΠΎ Ρ‚Π°ΠΊΠΎΠ΅ Π΄ΠΎΠΌΠ΅Π½ ΠΈ Π΄ΠΈΠ°ΠΏΠ°Π·ΠΎΠ½?
2. Π”ΠΎΠΌΠ΅Π½ ΠΈ ΠΎΠ±Π»Π°ΡΡ‚ΡŒ дСйствия
3. Π”ΠΎΠΌΠ΅Π½ Ρ„ΡƒΠ½ΠΊΡ†ΠΈΠΈ
4. Π”ΠΈΠ°ΠΏΠ°Π·ΠΎΠ½ Ρ„ΡƒΠ½ΠΊΡ†ΠΈΠΈ
5. Как Ρ€Π°ΡΡΡ‡ΠΈΡ‚Π°Ρ‚ΡŒ Π΄ΠΎΠΌΠ΅Π½ ΠΈ Π΄ΠΈΠ°ΠΏΠ°Π·ΠΎΠ½?
6. Π”ΠΎΠΌΠ΅Π½ ΠΈ Π΄ΠΈΠ°ΠΏΠ°Π·ΠΎΠ½ ΡΠΊΡΠΏΠΎΠ½Π΅Π½Ρ†ΠΈΠ°Π»ΡŒΠ½Ρ‹Ρ… Ρ„ΡƒΠ½ΠΊΡ†ΠΈΠΉ
7. ΠžΠ±Π»Π°ΡΡ‚ΡŒ опрСдСлСния ΠΈ Π΄ΠΈΠ°ΠΏΠ°Π·ΠΎΠ½ тригономСтричСских Ρ„ΡƒΠ½ΠΊΡ†ΠΈΠΉ
8. Π”ΠΎΠΌΠ΅Π½ ΠΈ Π΄ΠΈΠ°ΠΏΠ°Π·ΠΎΠ½ Ρ„ΡƒΠ½ΠΊΡ†ΠΈΠΈ Π°Π±ΡΠΎΠ»ΡŽΡ‚Π½ΠΎΠ³ΠΎ значСния
9. Π“Ρ€Π°Ρ„ΠΈΠΊΠΈ области ΠΈ Π΄ΠΈΠ°ΠΏΠ°Π·ΠΎΠ½Π°
10. Часто Π·Π°Π΄Π°Π²Π°Π΅ΠΌΡ‹Π΅ вопросы ΠΎ Π΄ΠΎΠΌΠ΅Π½Π΅ ΠΈ Π΄ΠΈΠ°ΠΏΠ°Π·ΠΎΠ½Π΅

Π§Ρ‚ΠΎ Ρ‚Π°ΠΊΠΎΠ΅ Π΄ΠΎΠΌΠ΅Π½ ΠΈ Π΄ΠΈΠ°ΠΏΠ°Π·ΠΎΠ½?

Π”ΠΎΠΌΠ΅Π½ ΠΈ Π΄ΠΈΠ°ΠΏΠ°Π·ΠΎΠ½ ΠΎΠΏΡ€Π΅Π΄Π΅Π»Π΅Π½Ρ‹ для ΠΎΡ‚Π½ΠΎΡˆΠ΅Π½ΠΈΡ ΠΈ ΠΏΡ€Π΅Π΄ΡΡ‚Π°Π²Π»ΡΡŽΡ‚ собой Π½Π°Π±ΠΎΡ€Ρ‹ всСх ΠΊΠΎΠΎΡ€Π΄ΠΈΠ½Π°Ρ‚ x ΠΈ всСх ΠΊΠΎΠΎΡ€Π΄ΠΈΠ½Π°Ρ‚ y упорядочСнных ΠΏΠ°Ρ€ соотвСтствСнно. НапримСр, Ссли ΡΠΎΠΎΡ‚Π½ΠΎΡˆΠ΅Π½ΠΈΠ΅ R = {(1, 2), (2, 2), (3, 3), (4, 3)}, Ρ‚ΠΎ:

  • Π”ΠΎΠΌΠ΅Π½ = Π½Π°Π±ΠΎΡ€ всСх ΠΊΠΎΠΎΡ€Π΄ΠΈΠ½Π°Ρ‚ x = {1, 2, 3, 4}
  • Π”ΠΈΠ°ΠΏΠ°Π·ΠΎΠ½ = Π½Π°Π±ΠΎΡ€ всСх ΠΊΠΎΠΎΡ€Π΄ΠΈΠ½Π°Ρ‚ y = {2, 3}

ΠœΡ‹ ΠΌΠΎΠΆΠ΅ΠΌ Π²ΠΈΠ·ΡƒΠ°Π»ΠΈΠ·ΠΈΡ€ΠΎΠ²Π°Ρ‚ΡŒ это здСсь:

Π”ΠΎΠΌΠ΅Π½ ΠΈ Π΄ΠΈΠ°ΠΏΠ°Π·ΠΎΠ½ Ρ„ΡƒΠ½ΠΊΡ†ΠΈΠΈ

Π΄ΠΎΠΌΠ΅Π½ ΠΈ Π΄ΠΈΠ°ΠΏΠ°Π·ΠΎΠ½ Ρ„ΡƒΠ½ΠΊΡ†ΠΈΠΈ ΡΠ²Π»ΡΡŽΡ‚ΡΡ ΠΊΠΎΠΌΠΏΠΎΠ½Π΅Π½Ρ‚Π°ΠΌΠΈ Ρ„ΡƒΠ½ΠΊΡ†ΠΈΠΈ. Π”ΠΎΠΌΠ΅Π½ β€” это Π½Π°Π±ΠΎΡ€ всСх Π²Ρ…ΠΎΠ΄Π½Ρ‹Ρ… Π·Π½Π°Ρ‡Π΅Π½ΠΈΠΉ Ρ„ΡƒΠ½ΠΊΡ†ΠΈΠΈ, Π° Π΄ΠΈΠ°ΠΏΠ°Π·ΠΎΠ½ β€” это Π²ΠΎΠ·ΠΌΠΎΠΆΠ½Ρ‹ΠΉ Ρ€Π΅Π·ΡƒΠ»ΡŒΡ‚Π°Ρ‚, Π·Π°Π΄Π°Π½Π½Ρ‹ΠΉ Ρ„ΡƒΠ½ΠΊΡ†ΠΈΠ΅ΠΉ. Π”ΠΎΠΌΠ΅Π½ β†’ Ѐункция β†’ Π”ΠΈΠ°ΠΏΠ°Π·ΠΎΠ½. Если сущСствуСт функция f: A β†’ B такая, Ρ‡Ρ‚ΠΎ ΠΊΠ°ΠΆΠ΄Ρ‹ΠΉ элСмСнт A отобраТаСтся Π² элСмСнты B, Ρ‚ΠΎ A являСтся Π΄ΠΎΠΌΠ΅Π½ΠΎΠΌ, Π° B являСтся со-ΠΎΠ±Π»Π°ΡΡ‚ΡŒΡŽ. ΠžΠ±Ρ€Π°Π· элСмСнта ‘a’ ΠΏΡ€ΠΈ ΠΎΡ‚Π½ΠΎΡˆΠ΅Π½ΠΈΠΈ R задаСтся ΠΊΠ°ΠΊ ‘b’, Π³Π΄Π΅ (a,b) ∈ R. ΠžΠ±Π»Π°ΡΡ‚ΡŒΡŽ Π·Π½Π°Ρ‡Π΅Π½ΠΈΠΉ Ρ„ΡƒΠ½ΠΊΡ†ΠΈΠΈ являСтся мноТСство ΠΈΠ·ΠΎΠ±Ρ€Π°ΠΆΠ΅Π½ΠΈΠΉ. ΠžΠ±Π»Π°ΡΡ‚ΡŒ опрСдСлСния ΠΈ ΠΎΠ±Π»Π°ΡΡ‚ΡŒ Π·Π½Π°Ρ‡Π΅Π½ΠΈΠΉ Ρ„ΡƒΠ½ΠΊΡ†ΠΈΠΈ Π² ΠΎΠ±Ρ‰Π΅ΠΌ случаС ΠΎΠ±ΠΎΠ·Π½Π°Ρ‡Π°ΡŽΡ‚ΡΡ ΡΠ»Π΅Π΄ΡƒΡŽΡ‰ΠΈΠΌ ΠΎΠ±Ρ€Π°Π·ΠΎΠΌ: ΠΎΠ±Π»Π°ΡΡ‚ΡŒ опрСдСлСния (f) = {x ∈ R} ΠΈ ΠΎΠ±Π»Π°ΡΡ‚ΡŒ Π·Π½Π°Ρ‡Π΅Π½ΠΈΠΉ (f) = {f (x) : x ∈ ΠΎΠ±Π»Π°ΡΡ‚ΡŒ Π·Π½Π°Ρ‡Π΅Π½ΠΈΠΉ (f)}

ΠžΠ±Π»Π°ΡΡ‚ΡŒ опрСдСлСния ΠΈ ΠΎΠ±Π»Π°ΡΡ‚ΡŒ Π·Π½Π°Ρ‡Π΅Π½ΠΈΠΉ этой Ρ„ΡƒΠ½ΠΊΡ†ΠΈΠΈ f(x) = 2x Π·Π°Π΄Π°ΡŽΡ‚ΡΡ ΠΊΠ°ΠΊ ΠΎΠ±Π»Π°ΡΡ‚ΡŒ опрСдСлСния D = {x ∈ N } , ΠΎΠ±Π»Π°ΡΡ‚ΡŒ Π·Π½Π°Ρ‡Π΅Π½ΠΈΠΉ R = {(y): y = 2x}

Π”ΠΎΠΌΠ΅Π½ Ρ„ΡƒΠ½ΠΊΡ†ΠΈΠΈ

Π”ΠΎΠΌΠ΅Π½ Ρ„ΡƒΠ½ΠΊΡ†ΠΈΠΈ относится ΠΊΠΎ «всСм значСниям», ΠΊΠΎΡ‚ΠΎΡ€Ρ‹Π΅ входят Π² Ρ„ΡƒΠ½ΠΊΡ†ΠΈΡŽ. ΠžΠ±Π»Π°ΡΡ‚ΡŒ опрСдСлСния Ρ„ΡƒΠ½ΠΊΡ†ΠΈΠΈ β€” это Π½Π°Π±ΠΎΡ€ всСх Π²ΠΎΠ·ΠΌΠΎΠΆΠ½Ρ‹Ρ… Π²Ρ…ΠΎΠ΄Π½Ρ‹Ρ… Π΄Π°Π½Π½Ρ‹Ρ… для Ρ„ΡƒΠ½ΠΊΡ†ΠΈΠΈ. Рассмотрим этот ящик ΠΊΠ°ΠΊ Ρ„ΡƒΠ½ΠΊΡ†ΠΈΡŽ f(x) = 2x . ΠŸΡ€ΠΈ Π²Π²ΠΎΠ΄Π΅ Π·Π½Π°Ρ‡Π΅Π½ΠΈΠΉ x = {1,2,3,4,…} Π΄ΠΎΠΌΠ΅Π½ прСдставляСт собой просто Π½Π°Π±ΠΎΡ€ Π½Π°Ρ‚ΡƒΡ€Π°Π»ΡŒΠ½Ρ‹Ρ… чисСл, Π° Π²Ρ‹Ρ…ΠΎΠ΄Π½Ρ‹Π΅ значСния Π½Π°Π·Ρ‹Π²Π°ΡŽΡ‚ΡΡ Π΄ΠΈΠ°ΠΏΠ°Π·ΠΎΠ½ΠΎΠΌ. Но Π² ΠΎΠ±Ρ‰Π΅ΠΌ случаС f(x) = 2x ΠΎΠΏΡ€Π΅Π΄Π΅Π»Π΅Π½ΠΎ для всСх Π΄Π΅ΠΉΡΡ‚Π²ΠΈΡ‚Π΅Π»ΡŒΠ½Ρ‹Ρ… Π·Π½Π°Ρ‡Π΅Π½ΠΈΠΉ x, ΠΈ, ΡΠ»Π΅Π΄ΠΎΠ²Π°Ρ‚Π΅Π»ΡŒΠ½ΠΎ, Π΅Π³ΠΎ ΠΎΠ±Π»Π°ΡΡ‚ΡŒΡŽ опрСдСлСния являСтся мноТСство всСх Π΄Π΅ΠΉΡΡ‚Π²ΠΈΡ‚Π΅Π»ΡŒΠ½Ρ‹Ρ… чисСл, ΠΊΠΎΡ‚ΠΎΡ€ΠΎΠ΅ обозначаСтся (-∞, ∞). Π’ΠΎΡ‚ ΠΎΠ±Ρ‰ΠΈΠ΅ Ρ„ΠΎΡ€ΠΌΡƒΠ»Ρ‹, ΠΈΡΠΏΠΎΠ»ΡŒΠ·ΡƒΠ΅ΠΌΡ‹Π΅ для нахоТдСния области опрСдСлСния Ρ€Π°Π·Π»ΠΈΡ‡Π½Ρ‹Ρ… Ρ‚ΠΈΠΏΠΎΠ² Ρ„ΡƒΠ½ΠΊΡ†ΠΈΠΉ. Π—Π΄Π΅ΡΡŒ R β€” мноТСство всСх Π΄Π΅ΠΉΡΡ‚Π²ΠΈΡ‚Π΅Π»ΡŒΠ½Ρ‹Ρ… чисСл.

  • ΠžΠ±Π»Π°ΡΡ‚ΡŒ опрСдСлСния любой полиномиальной (Π»ΠΈΠ½Π΅ΠΉΠ½ΠΎΠΉ, ΠΊΠ²Π°Π΄Ρ€Π°Ρ‚ΠΈΡ‡Π½ΠΎΠΉ, кубичСской ΠΈ Ρ‚. Π΄.) Ρ„ΡƒΠ½ΠΊΡ†ΠΈΠΈ Ρ€Π°Π²Π½Π° R.
  • ΠžΠ±Π»Π°ΡΡ‚ΡŒ опрСдСлСния Ρ„ΡƒΠ½ΠΊΡ†ΠΈΠΈ извлСчСния ΠΊΠ²Π°Π΄Ρ€Π°Ρ‚Π½ΠΎΠ³ΠΎ корня √x Ρ€Π°Π²Π½Π° xβ‰₯0.
  • ΠžΠ±Π»Π°ΡΡ‚ΡŒ опрСдСлСния ΡΠΊΡΠΏΠΎΠ½Π΅Π½Ρ†ΠΈΠ°Π»ΡŒΠ½ΠΎΠΉ Ρ„ΡƒΠ½ΠΊΡ†ΠΈΠΈ R.
  • ΠžΠ±Π»Π°ΡΡ‚ΡŒ опрСдСлСния логарифмичСской Ρ„ΡƒΠ½ΠΊΡ†ΠΈΠΈ x>0.
  • Π§Ρ‚ΠΎΠ±Ρ‹ Π½Π°ΠΉΡ‚ΠΈ ΠΎΠ±Π»Π°ΡΡ‚ΡŒ опрСдСлСния Ρ€Π°Ρ†ΠΈΠΎΠ½Π°Π»ΡŒΠ½ΠΎΠΉ Ρ„ΡƒΠ½ΠΊΡ†ΠΈΠΈ y = f(x), установитС Π·Π½Π°ΠΌΠ΅Π½Π°Ρ‚Π΅Π»ΡŒ β‰  0.

Π”ΠΈΠ°ΠΏΠ°Π·ΠΎΠ½ Ρ„ΡƒΠ½ΠΊΡ†ΠΈΠΈ

Π”ΠΈΠ°ΠΏΠ°Π·ΠΎΠ½ Ρ„ΡƒΠ½ΠΊΡ†ΠΈΠΈ β€” это Π½Π°Π±ΠΎΡ€ всСх Π΅Π΅ Π²Ρ‹Ρ…ΠΎΠ΄ΠΎΠ². ΠŸΡ€ΠΈΠΌΠ΅Ρ€. Рассмотрим Ρ„ΡƒΠ½ΠΊΡ†ΠΈΡŽ f: A β†’ B, Π³Π΄Π΅ f(x) = 2x ΠΈ ΠΊΠ°ΠΆΠ΄ΠΎΠ΅ ΠΈΠ· A ΠΈ B = {мноТСство Π½Π°Ρ‚ΡƒΡ€Π°Π»ΡŒΠ½Ρ‹Ρ… чисСл}. Π—Π΄Π΅ΡΡŒ ΠΌΡ‹ Π³ΠΎΠ²ΠΎΡ€ΠΈΠΌ, Ρ‡Ρ‚ΠΎ А β€” Π΄ΠΎΠΌΠ΅Π½, Π° Π’ β€” содомСн. Π—Π°Ρ‚Π΅ΠΌ Π²Ρ‹Ρ…ΠΎΠ΄ этой Ρ„ΡƒΠ½ΠΊΡ†ΠΈΠΈ становится Π΄ΠΈΠ°ΠΏΠ°Π·ΠΎΠ½ΠΎΠΌ. Π”ΠΈΠ°ΠΏΠ°Π·ΠΎΠ½ = {мноТСство Ρ‡Π΅Ρ‚Π½Ρ‹Ρ… Π½Π°Ρ‚ΡƒΡ€Π°Π»ΡŒΠ½Ρ‹Ρ… чисСл}. Π­Π»Π΅ΠΌΠ΅Π½Ρ‚Ρ‹ Π΄ΠΎΠΌΠ΅Π½Π° Π½Π°Π·Ρ‹Π²Π°ΡŽΡ‚ΡΡ ΠΏΡ€ΠΎΠΎΠ±Ρ€Π°Π·Π°ΠΌΠΈ, Π° ΠΎΡ‚ΠΎΠ±Ρ€Π°ΠΆΠ°Π΅ΠΌΡ‹Π΅ элСмСнты содомСна Π½Π°Π·Ρ‹Π²Π°ΡŽΡ‚ΡΡ изобраТСниями. Π—Π΄Π΅ΡΡŒ ΠΎΠ±Π»Π°ΡΡ‚ΡŒΡŽ Π·Π½Π°Ρ‡Π΅Π½ΠΈΠΉ Ρ„ΡƒΠ½ΠΊΡ†ΠΈΠΈ f являСтся мноТСство всСх ΠΈΠ·ΠΎΠ±Ρ€Π°ΠΆΠ΅Π½ΠΈΠΉ элСмСнтов области (ΠΈΠ»ΠΈ) мноТСство всСх Π²Ρ‹Ρ…ΠΎΠ΄ΠΎΠ² Ρ„ΡƒΠ½ΠΊΡ†ΠΈΠΈ. Π’ ΡΠ»Π΅Π΄ΡƒΡŽΡ‰ΠΈΡ… Ρ€Π°Π·Π΄Π΅Π»Π°Ρ… ΠΌΡ‹ ΡƒΠ²ΠΈΠ΄ΠΈΠΌ, ΠΊΠ°ΠΊ Π½Π°ΠΉΡ‚ΠΈ Π΄ΠΈΠ°ΠΏΠ°Π·ΠΎΠ½ Ρ€Π°Π·Π»ΠΈΡ‡Π½Ρ‹Ρ… Ρ‚ΠΈΠΏΠΎΠ² Ρ„ΡƒΠ½ΠΊΡ†ΠΈΠΉ. Π’ΠΎΡ‚ ΠΎΠ±Ρ‰ΠΈΠ΅ Ρ„ΠΎΡ€ΠΌΡƒΠ»Ρ‹, ΠΈΡΠΏΠΎΠ»ΡŒΠ·ΡƒΠ΅ΠΌΡ‹Π΅ для нахоТдСния Π΄ΠΈΠ°ΠΏΠ°Π·ΠΎΠ½Π° Ρ€Π°Π·Π»ΠΈΡ‡Π½Ρ‹Ρ… Ρ‚ΠΈΠΏΠΎΠ² Ρ„ΡƒΠ½ΠΊΡ†ΠΈΠΉ. ΠžΠ±Ρ€Π°Ρ‚ΠΈΡ‚Π΅ Π²Π½ΠΈΠΌΠ°Π½ΠΈΠ΅, Ρ‡Ρ‚ΠΎ здСсь R β€” это Π½Π°Π±ΠΎΡ€ всСх Π΄Π΅ΠΉΡΡ‚Π²ΠΈΡ‚Π΅Π»ΡŒΠ½Ρ‹Ρ… чисСл.

  • Π”ΠΈΠ°ΠΏΠ°Π·ΠΎΠ½ Π»ΠΈΠ½Π΅ΠΉΠ½ΠΎΠΉ Ρ„ΡƒΠ½ΠΊΡ†ΠΈΠΈ R.
  • Π”ΠΈΠ°ΠΏΠ°Π·ΠΎΠ½ ΠΊΠ²Π°Π΄Ρ€Π°Ρ‚ΠΈΡ‡Π½ΠΎΠΉ Ρ„ΡƒΠ½ΠΊΡ†ΠΈΠΈ y = a(x-h) 2 + k Ρ€Π°Π²Π΅Π½:
    yβ‰₯k, Ссли a>0 ΠΈ
    y≀k, Ссли a<0
  • Π”ΠΈΠ°ΠΏΠ°Π·ΠΎΠ½ Ρ„ΡƒΠ½ΠΊΡ†ΠΈΠΈ извлСчСния ΠΊΠ²Π°Π΄Ρ€Π°Ρ‚Π½ΠΎΠ³ΠΎ корня: yβ‰₯0.
  • Π”ΠΈΠ°ΠΏΠ°Π·ΠΎΠ½ ΡΠΊΡΠΏΠΎΠ½Π΅Π½Ρ†ΠΈΠ°Π»ΡŒΠ½ΠΎΠΉ Ρ„ΡƒΠ½ΠΊΡ†ΠΈΠΈ: y>0.
  • Π”ΠΈΠ°ΠΏΠ°Π·ΠΎΠ½ логарифмичСской Ρ„ΡƒΠ½ΠΊΡ†ΠΈΠΈ R.
  • Π§Ρ‚ΠΎΠ±Ρ‹ Π½Π°ΠΉΡ‚ΠΈ Π΄ΠΈΠ°ΠΏΠ°Π·ΠΎΠ½ Ρ€Π°Ρ†ΠΈΠΎΠ½Π°Π»ΡŒΠ½ΠΎΠΉ Ρ„ΡƒΠ½ΠΊΡ†ΠΈΠΈ y = f(x), Ρ€Π΅ΡˆΠΈΡ‚Π΅ Π΅Π΅ ΠΎΡ‚Π½ΠΎΡΠΈΡ‚Π΅Π»ΡŒΠ½ΠΎ x ΠΈ установитС Π·Π½Π°ΠΌΠ΅Π½Π°Ρ‚Π΅Π»ΡŒ β‰  0.

Как Ρ€Π°ΡΡΡ‡ΠΈΡ‚Π°Ρ‚ΡŒ Π΄ΠΎΠΌΠ΅Π½ ΠΈ Π΄ΠΈΠ°ΠΏΠ°Π·ΠΎΠ½?

ΠŸΡ€Π΅Π΄ΠΏΠΎΠ»ΠΎΠΆΠΈΠΌ, Ρ‡Ρ‚ΠΎ X = {1, 2, 3, 4, 5}, f: X β†’ Y, Π³Π΄Π΅ R = {(x,y) : y = x+1}.

Π”ΠΎΠΌΠ΅Π½ = Π²Ρ…ΠΎΠ΄Π½Ρ‹Π΅ значСния. Π’Π°ΠΊΠΈΠΌ ΠΎΠ±Ρ€Π°Π·ΠΎΠΌ, Π”ΠΎΠΌΠ΅Π½ = X = {1, 2, 3, 4, 5}

Π”ΠΈΠ°ΠΏΠ°Π·ΠΎΠ½ = Π²Ρ‹Ρ…ΠΎΠ΄Π½Ρ‹Π΅ значСния Ρ„ΡƒΠ½ΠΊΡ†ΠΈΠΈ = {2, 3, 4, 5, 6}

и со-домСн = Y = {2 , 3, 4, 5, 6}

Π”Π°Π²Π°ΠΉΡ‚Π΅ разбСрСмся Π² ΠΏΡ€Π΅Π΄ΠΌΠ΅Ρ‚Π½ΠΎΠΉ области ΠΈ Π΄ΠΈΠ°ΠΏΠ°Π·ΠΎΠ½Π΅ Π½Π΅ΠΊΠΎΡ‚ΠΎΡ€Ρ‹Ρ… ΡΠΏΠ΅Ρ†ΠΈΠ°Π»ΡŒΠ½Ρ‹Ρ… Ρ„ΡƒΠ½ΠΊΡ†ΠΈΠΉ, принимая Π²ΠΎ Π²Π½ΠΈΠΌΠ°Π½ΠΈΠ΅ Ρ€Π°Π·Π»ΠΈΡ‡Π½Ρ‹Π΅ Ρ‚ΠΈΠΏΡ‹ Ρ„ΡƒΠ½ΠΊΡ†ΠΈΠΉ.

ΠžΠ±Π»Π°ΡΡ‚ΡŒ ΠΈ Π΄ΠΈΠ°ΠΏΠ°Π·ΠΎΠ½ ΡΠΊΡΠΏΠΎΠ½Π΅Π½Ρ†ΠΈΠ°Π»ΡŒΠ½Ρ‹Ρ… Ρ„ΡƒΠ½ΠΊΡ†ΠΈΠΉ

Ѐункция y = a x , a β‰₯ 0 ΠΎΠΏΡ€Π΅Π΄Π΅Π»Π΅Π½Π° для всСх Π΄Π΅ΠΉΡΡ‚Π²ΠΈΡ‚Π΅Π»ΡŒΠ½Ρ‹Ρ… чисСл. Π‘Π»Π΅Π΄ΠΎΠ²Π°Ρ‚Π΅Π»ΡŒΠ½ΠΎ, ΠΎΠ±Π»Π°ΡΡ‚ΡŒΡŽ опрСдСлСния ΡΠΊΡΠΏΠΎΠ½Π΅Π½Ρ†ΠΈΠ°Π»ΡŒΠ½ΠΎΠΉ Ρ„ΡƒΠ½ΠΊΡ†ΠΈΠΈ являСтся вся вСщСствСнная прямая. Π­ΠΊΡΠΏΠΎΠ½Π΅Π½Ρ†ΠΈΠ°Π»ΡŒΠ½Π°Ρ функция всСгда Π΄Π°Π΅Ρ‚ ΠΏΠΎΠ»ΠΎΠΆΠΈΡ‚Π΅Π»ΡŒΠ½ΠΎΠ΅ Π·Π½Π°Ρ‡Π΅Π½ΠΈΠ΅. Π’Π°ΠΊΠΈΠΌ ΠΎΠ±Ρ€Π°Π·ΠΎΠΌ, Π΄ΠΈΠ°ΠΏΠ°Π·ΠΎΠ½ ΡΠΊΡΠΏΠΎΠ½Π΅Π½Ρ†ΠΈΠ°Π»ΡŒΠ½ΠΎΠΉ Ρ„ΡƒΠ½ΠΊΡ†ΠΈΠΈ ΠΈΠΌΠ΅Π΅Ρ‚ Π²ΠΈΠ΄ y= |ax+b| y ∈ R , {y > 0}. Π”ΠΎΠΌΠ΅Π½ = R, Π”ΠΈΠ°ΠΏΠ°Π·ΠΎΠ½ = (0, ∞)

ΠŸΡ€ΠΈΠΌΠ΅Ρ€: ΠŸΠΎΡΠΌΠΎΡ‚Ρ€ΠΈΡ‚Π΅ Π½Π° Π³Ρ€Π°Ρ„ΠΈΠΊ этой Ρ„ΡƒΠ½ΠΊΡ†ΠΈΠΈ f: 2 x

ΠžΠ±Ρ€Π°Ρ‚ΠΈΡ‚Π΅ Π²Π½ΠΈΠΌΠ°Π½ΠΈΠ΅, Ρ‡Ρ‚ΠΎ Π·Π½Π°Ρ‡Π΅Π½ΠΈΠ΅ Ρ„ΡƒΠ½ΠΊΡ†ΠΈΠΈ Π±Π»ΠΈΠΆΠ΅ ΠΊ 0, ΠΏΠΎΡΠΊΠΎΠ»ΡŒΠΊΡƒ x стрСмится ΠΊ ∞, Π½ΠΎ это Π½ΠΈΠΊΠΎΠ³Π΄Π° Π½Π΅ достигнСт значСния 0. ΠžΠ±Π»Π°ΡΡ‚ΡŒ опрСдСлСния ΠΈ Π΄ΠΈΠ°ΠΏΠ°Π·ΠΎΠ½ ΡΠΊΡΠΏΠΎΠ½Π΅Π½Ρ†ΠΈΠ°Π»ΡŒΠ½ΠΎΠΉ Ρ„ΡƒΠ½ΠΊΡ†ΠΈΠΈ Π·Π°Π΄Π°ΡŽΡ‚ΡΡ ΡΠ»Π΅Π΄ΡƒΡŽΡ‰ΠΈΠΌ ΠΎΠ±Ρ€Π°Π·ΠΎΠΌ:

  • Π”ΠΎΠΌΠ΅Π½: Π”ΠΎΠΌΠ΅Π½ Ρ„ΡƒΠ½ΠΊΡ†ΠΈΠΈ β€” мноТСство R.
  • Π”ΠΈΠ°ΠΏΠ°Π·ΠΎΠ½: Π­ΠΊΡΠΏΠΎΠ½Π΅Π½Ρ†ΠΈΠ°Π»ΡŒΠ½Π°Ρ функция всСгда ΠΏΡ€ΠΈΠ²ΠΎΠ΄ΠΈΡ‚ ΠΊ ΠΏΠΎΠ»ΠΎΠΆΠΈΡ‚Π΅Π»ΡŒΠ½Ρ‹ΠΌ Π΄Π΅ΠΉΡΡ‚Π²ΠΈΡ‚Π΅Π»ΡŒΠ½Ρ‹ΠΌ значСниям.

ΠžΠ±Π»Π°ΡΡ‚ΡŒ опрСдСлСния ΠΈ ΠΎΠ±Π»Π°ΡΡ‚ΡŒ Π·Π½Π°Ρ‡Π΅Π½ΠΈΠΉ тригономСтричСских Ρ„ΡƒΠ½ΠΊΡ†ΠΈΠΉ

ΠŸΠΎΡΠΌΠΎΡ‚Ρ€ΠΈΡ‚Π΅ Π½Π° Π³Ρ€Π°Ρ„ΠΈΠΊ Ρ„ΡƒΠ½ΠΊΡ†ΠΈΠΈ синуса ΠΈ косинуса. ΠžΠ±Ρ€Π°Ρ‚ΠΈΡ‚Π΅ Π²Π½ΠΈΠΌΠ°Π½ΠΈΠ΅, Ρ‡Ρ‚ΠΎ Π·Π½Π°Ρ‡Π΅Π½ΠΈΠ΅ Ρ„ΡƒΠ½ΠΊΡ†ΠΈΠΉ колСблСтся ΠΌΠ΅ΠΆΠ΄Ρƒ -1 ΠΈ 1 ΠΈ ΠΎΠΏΡ€Π΅Π΄Π΅Π»Π΅Π½ΠΎ для всСх Π΄Π΅ΠΉΡΡ‚Π²ΠΈΡ‚Π΅Π»ΡŒΠ½Ρ‹Ρ… чисСл.

Π’Π°ΠΊΠΈΠΌ ΠΎΠ±Ρ€Π°Π·ΠΎΠΌ, для ΠΊΠ°ΠΆΠ΄ΠΎΠΉ ΠΈΠ· Ρ„ΡƒΠ½ΠΊΡ†ΠΈΠΉ синуса ΠΈ косинуса:

  • Π”ΠΎΠΌΠ΅Π½: Π”ΠΎΠΌΠ΅Π½ Ρ„ΡƒΠ½ΠΊΡ†ΠΈΠΉ β€” мноТСство R.
  • Π”ΠΈΠ°ΠΏΠ°Π·ΠΎΠ½: Π”ΠΈΠ°ΠΏΠ°Π·ΠΎΠ½ Ρ„ΡƒΠ½ΠΊΡ†ΠΈΠΉ [-1, 1]

ΠžΠ±Π»Π°ΡΡ‚ΡŒ опрСдСлСния ΠΈ Π΄ΠΈΠ°ΠΏΠ°Π·ΠΎΠ½ всСх тригономСтричСских Ρ„ΡƒΠ½ΠΊΡ†ΠΈΠΉ ΠΏΠΎΠΊΠ°Π·Π°Π½Ρ‹ Π½ΠΈΠΆΠ΅:

ВригономСтричСскиС Ρ„ΡƒΠ½ΠΊΡ†ΠΈΠΈ Π”ΠΎΠΌΠ΅Π½ Π”ΠΈΠ°ΠΏΠ°Π·ΠΎΠ½
Sinθ (-∞, + ∞) [-1, +1]
Cosθ (-∞ +∞) [-1, +1]
Π’Π°Π½ΞΈ Π  — (2n + 1)Ο€/2 (-∞, +∞)
ДСтская ΠΊΡ€ΠΎΠ²Π°Ρ‚ΠΊΠ° Π  — Π½ΠΎΠΌΠ΅Ρ€ (-∞, +∞)
сСк ΞΈ Π  — (2n + 1)Ο€/2 (-∞, -1] U [+1, +∞)
CosecΞΈ Π  — Π½ΠΎΠΌΠ΅Ρ€ (-∞, -1] U [+1, +∞)

Π”ΠΎΠΌΠ΅Π½ ΠΈ Π΄ΠΈΠ°ΠΏΠ°Π·ΠΎΠ½ Ρ„ΡƒΠ½ΠΊΡ†ΠΈΠΈ Π°Π±ΡΠΎΠ»ΡŽΡ‚Π½ΠΎΠ³ΠΎ значСния

Ѐункция y=|ax+b| ΠΎΠΏΡ€Π΅Π΄Π΅Π»Π΅Π½ΠΎ для всСх Π΄Π΅ΠΉΡΡ‚Π²ΠΈΡ‚Π΅Π»ΡŒΠ½Ρ‹Ρ… чисСл. Π˜Ρ‚Π°ΠΊ, ΠΎΠ±Π»Π°ΡΡ‚ΡŒ опрСдСлСния Ρ„ΡƒΠ½ΠΊΡ†ΠΈΠΈ Π°Π±ΡΠΎΠ»ΡŽΡ‚Π½ΠΎΠ³ΠΎ значСния β€” это мноТСство всСх Π΄Π΅ΠΉΡΡ‚Π²ΠΈΡ‚Π΅Π»ΡŒΠ½Ρ‹Ρ… чисСл. ΠΠ±ΡΠΎΠ»ΡŽΡ‚Π½ΠΎΠ΅ Π·Π½Π°Ρ‡Π΅Π½ΠΈΠ΅ числа всСгда Π΄Π°Π΅Ρ‚ Π½Π΅ΠΎΡ‚Ρ€ΠΈΡ†Π°Ρ‚Π΅Π»ΡŒΠ½ΠΎΠ΅ Π·Π½Π°Ρ‡Π΅Π½ΠΈΠ΅. Π’Π°ΠΊΠΈΠΌ ΠΎΠ±Ρ€Π°Π·ΠΎΠΌ, Π΄ΠΈΠ°ΠΏΠ°Π·ΠΎΠ½ Ρ„ΡƒΠ½ΠΊΡ†ΠΈΠΈ Π°Π±ΡΠΎΠ»ΡŽΡ‚Π½ΠΎΠ³ΠΎ значСния Π²ΠΈΠ΄Π° y= |ax+b| y ∈ R | y β‰₯ 0. ΠžΠ±Π»Π°ΡΡ‚ΡŒ опрСдСлСния ΠΈ Π΄ΠΈΠ°ΠΏΠ°Π·ΠΎΠ½ Ρ„ΡƒΠ½ΠΊΡ†ΠΈΠΈ Π°Π±ΡΠΎΠ»ΡŽΡ‚Π½ΠΎΠ³ΠΎ значСния Π·Π°Π΄Π°ΡŽΡ‚ΡΡ ΡΠ»Π΅Π΄ΡƒΡŽΡ‰ΠΈΠΌ ΠΎΠ±Ρ€Π°Π·ΠΎΠΌ

  • Π”ΠΎΠΌΠ΅Π½ = R
  • Π”ΠΈΠ°ΠΏΠ°Π·ΠΎΠ½ = [0, ∞)

ΠŸΡ€ΠΈΠΌΠ΅Ρ€: |6-x|

  • Π”ΠΎΠΌΠ΅Π½: Π”ΠΎΠΌΠ΅Π½ Ρ„ΡƒΠ½ΠΊΡ†ΠΈΠΈ β€” мноТСство R.
  • Π”ΠΈΠ°ΠΏΠ°Π·ΠΎΠ½: ΠœΡ‹ ΡƒΠΆΠ΅ Π·Π½Π°Π΅ΠΌ, Ρ‡Ρ‚ΠΎ функция Π°Π±ΡΠΎΠ»ΡŽΡ‚Π½ΠΎΠ³ΠΎ значСния всСгда Π΄Π°Π΅Ρ‚ Π½Π΅ΠΎΡ‚Ρ€ΠΈΡ†Π°Ρ‚Π΅Π»ΡŒΠ½ΠΎΠ΅ Π·Π½Π°Ρ‡Π΅Π½ΠΈΠ΅. Ρ‚. Π΅. |6-Ρ…| β‰₯ 0 для всСх Ρ….

ΠžΠ±Π»Π°ΡΡ‚ΡŒ опрСдСлСния ΠΈ ΠΎΠ±Π»Π°ΡΡ‚ΡŒ Π·Π½Π°Ρ‡Π΅Π½ΠΈΠΉ Ρ„ΡƒΠ½ΠΊΡ†ΠΈΠΈ ΠΊΠ²Π°Π΄Ρ€Π°Ρ‚Π½ΠΎΠ³ΠΎ корня

Ѐункция y= √(ax+b) ΠΎΠΏΡ€Π΅Π΄Π΅Π»Π΅Π½Π° Ρ‚ΠΎΠ»ΡŒΠΊΠΎ для x β‰₯ -b/a

Π˜Ρ‚Π°ΠΊ, ΠΎΠ±Π»Π°ΡΡ‚ΡŒ опрСдСлСния Ρ„ΡƒΠ½ΠΊΡ†ΠΈΠΈ извлСчСния ΠΊΠ²Π°Π΄Ρ€Π°Ρ‚Π½ΠΎΠ³ΠΎ корня β€” это мноТСство всСх Π΄Π΅ΠΉΡΡ‚Π²ΠΈΡ‚Π΅Π»ΡŒΠ½Ρ‹Ρ… чисСл, Π±ΠΎΠ»ΡŒΡˆΠΈΡ… ΠΈΠ»ΠΈ Ρ€Π°Π²Π½Ρ‹Ρ… β€”b/a. ΠœΡ‹ Π·Π½Π°Π΅ΠΌ, Ρ‡Ρ‚ΠΎ ΠΊΠ²Π°Π΄Ρ€Π°Ρ‚Π½Ρ‹ΠΉ ΠΊΠΎΡ€Π΅Π½ΡŒ всСгда Π΄Π°Π΅Ρ‚ Π½Π΅ΠΎΡ‚Ρ€ΠΈΡ†Π°Ρ‚Π΅Π»ΡŒΠ½ΠΎΠ΅ Π·Π½Π°Ρ‡Π΅Π½ΠΈΠ΅. Π’Π°ΠΊΠΈΠΌ ΠΎΠ±Ρ€Π°Π·ΠΎΠΌ, ΠΎΠ±Π»Π°ΡΡ‚ΡŒΡŽ дСйствия Ρ„ΡƒΠ½ΠΊΡ†ΠΈΠΈ ΠΊΠ²Π°Π΄Ρ€Π°Ρ‚Π½ΠΎΠ³ΠΎ корня являСтся мноТСство всСх Π½Π΅ΠΎΡ‚Ρ€ΠΈΡ†Π°Ρ‚Π΅Π»ΡŒΠ½Ρ‹Ρ… Π΄Π΅ΠΉΡΡ‚Π²ΠΈΡ‚Π΅Π»ΡŒΠ½Ρ‹Ρ… чисСл. ΠžΠ±Π»Π°ΡΡ‚ΡŒ опрСдСлСния ΠΈ Π΄ΠΈΠ°ΠΏΠ°Π·ΠΎΠ½ Ρ„ΡƒΠ½ΠΊΡ†ΠΈΠΈ ΠΊΠ²Π°Π΄Ρ€Π°Ρ‚Π½ΠΎΠ³ΠΎ корня Π·Π°Π΄Π°ΡŽΡ‚ΡΡ ΡΠ»Π΅Π΄ΡƒΡŽΡ‰ΠΈΠΌ ΠΎΠ±Ρ€Π°Π·ΠΎΠΌ: ΠžΠ±Π»Π°ΡΡ‚ΡŒ = [-b/a,∞), Π”ΠΈΠ°ΠΏΠ°Π·ΠΎΠ½ = [0,∞)

ΠŸΡ€ΠΈΠΌΠ΅Ρ€: y= 2- √(-3x+2)

Π”ΠΎΠΌΠ΅Π½: Ѐункция извлСчСния ΠΊΠ²Π°Π΄Ρ€Π°Ρ‚Π½ΠΎΠ³ΠΎ корня опрСдСляСтся Ρ‚ΠΎΠ»ΡŒΠΊΠΎ Ρ‚ΠΎΠ³Π΄Π°, ΠΊΠΎΠ³Π΄Π° Π·Π½Π°Ρ‡Π΅Π½ΠΈΠ΅ Π²Π½ΡƒΡ‚Ρ€ΠΈ Π½Π΅Π΅ являСтся Π½Π΅ΠΎΡ‚Ρ€ΠΈΡ†Π°Ρ‚Π΅Π»ΡŒΠ½Ρ‹ΠΌ числом. Π˜Ρ‚Π°ΠΊ, для Π΄ΠΎΠΌΠ΅Π½Π°

-3x+2 β‰₯ 0

-3x β‰₯ -2

x ≀ 2/3

Π”ΠΈΠ°ΠΏΠ°Π·ΠΎΠ½: ΠœΡ‹ ΡƒΠΆΠ΅ Π·Π½Π°Π΅ΠΌ, Ρ‡Ρ‚ΠΎ функция ΠΊΠ²Π°Π΄Ρ€Π°Ρ‚Π½ΠΎΠ³ΠΎ корня всСгда Π΄Π°Π΅Ρ‚ Π½Π΅ΠΎΡ‚Ρ€ΠΈΡ†Π°Ρ‚Π΅Π»ΡŒΠ½ΠΎΠ΅ Π·Π½Π°Ρ‡Π΅Π½ΠΈΠ΅.

√(-3x+2)β‰₯ 0

Π£ΠΌΠ½ΠΎΠΆΠ΅Π½ΠΈΠ΅ -1 с ΠΎΠ±Π΅ΠΈΡ… сторон

-√(-3x+2) ≀ 0

Π”ΠΎΠ±Π°Π²Π»Π΅Π½ΠΈΠ΅ 2 с ΠΎΠ±Π΅ΠΈΡ… сторон

2-√(-3x+2)≀ 2

лСт≀ 2

Π“Ρ€Π°Ρ„ΠΈΠΊΠΈ области ΠΈ Π΄ΠΈΠ°ΠΏΠ°Π·ΠΎΠ½Π°

Π”Ρ€ΡƒΠ³ΠΎΠΉ способ опрСдСлСния области ΠΈ Π΄ΠΈΠ°ΠΏΠ°Π·ΠΎΠ½Π° Ρ„ΡƒΠ½ΠΊΡ†ΠΈΠΉ β€” использованиС Π³Ρ€Π°Ρ„ΠΈΠΊΠΎΠ². Π”ΠΎΠΌΠ΅Π½ относится ΠΊ Π½Π°Π±ΠΎΡ€Ρƒ Π²ΠΎΠ·ΠΌΠΎΠΆΠ½Ρ‹Ρ… Π²Ρ…ΠΎΠ΄Π½Ρ‹Ρ… Π·Π½Π°Ρ‡Π΅Π½ΠΈΠΉ. Π”ΠΎΠΌΠ΅Π½ Π³Ρ€Π°Ρ„ΠΈΠΊΠ° состоит ΠΈΠ· всСх Π²Ρ…ΠΎΠ΄Π½Ρ‹Ρ… Π·Π½Π°Ρ‡Π΅Π½ΠΈΠΉ, ΠΏΠΎΠΊΠ°Π·Π°Π½Π½Ρ‹Ρ… Π½Π° оси X. Π”ΠΈΠ°ΠΏΠ°Π·ΠΎΠ½ β€” это Π½Π°Π±ΠΎΡ€ Π²ΠΎΠ·ΠΌΠΎΠΆΠ½Ρ‹Ρ… Π²Ρ‹Ρ…ΠΎΠ΄Π½Ρ‹Ρ… Π·Π½Π°Ρ‡Π΅Π½ΠΈΠΉ, ΠΏΠΎΠΊΠ°Π·Π°Π½Π½Ρ‹Ρ… Π½Π° оси Y. Π‘Π°ΠΌΡ‹ΠΉ простой способ Π½Π°ΠΉΡ‚ΠΈ Π΄ΠΈΠ°ΠΏΠ°Π·ΠΎΠ½ Ρ„ΡƒΠ½ΠΊΡ†ΠΈΠΈ состоит Π² построСнии Π³Ρ€Π°Ρ„ΠΈΠΊΠ° ΠΈ поискС Π·Π½Π°Ρ‡Π΅Π½ΠΈΠΉ y, ΠΎΡ…Π²Π°Ρ‚Ρ‹Π²Π°Π΅ΠΌΡ‹Ρ… Π³Ρ€Π°Ρ„ΠΈΠΊΠΎΠΌ. Π§Ρ‚ΠΎΠ±Ρ‹ Π½Π°ΠΉΡ‚ΠΈ Π΄ΠΈΠ°ΠΏΠ°Π·ΠΎΠ½ ΠΊΠ²Π°Π΄Ρ€Π°Ρ‚ΠΈΡ‡Π½ΠΎΠΉ Ρ„ΡƒΠ½ΠΊΡ†ΠΈΠΈ, достаточно ΠΏΠΎΡΠΌΠΎΡ‚Ρ€Π΅Ρ‚ΡŒ, ΠΈΠΌΠ΅Π΅Ρ‚ Π»ΠΈ ΠΎΠ½Π° максимальноС ΠΈΠ»ΠΈ минимальноС Π·Π½Π°Ρ‡Π΅Π½ΠΈΠ΅. МаксимальноС/минимальноС Π·Π½Π°Ρ‡Π΅Π½ΠΈΠ΅ ΠΊΠ²Π°Π΄Ρ€Π°Ρ‚ΠΈΡ‡Π½ΠΎΠΉ Ρ„ΡƒΠ½ΠΊΡ†ΠΈΠΈ β€” это ΠΊΠΎΠΎΡ€Π΄ΠΈΠ½Π°Ρ‚Π° y Π΅Π΅ Π²Π΅Ρ€ΡˆΠΈΠ½Ρ‹. Π§Ρ‚ΠΎΠ±Ρ‹ Π½Π°ΠΉΡ‚ΠΈ ΠΎΠ±Π»Π°ΡΡ‚ΡŒ опрСдСлСния Ρ€Π°Ρ†ΠΈΠΎΠ½Π°Π»ΡŒΠ½ΠΎΠΉ Ρ„ΡƒΠ½ΠΊΡ†ΠΈΠΈ, установитС Π·Π½Π°ΠΌΠ΅Π½Π°Ρ‚Π΅Π»ΡŒ Ρ€Π°Π²Π½Ρ‹ΠΌ 0 ΠΈ Π½Π°ΠΉΠ΄ΠΈΡ‚Π΅ ΠΏΠ΅Ρ€Π΅ΠΌΠ΅Π½Π½ΡƒΡŽ. Π”ΠΎΠΌΠ΅Π½ обозначаСтся всСми значСниями слСва Π½Π°ΠΏΡ€Π°Π²ΠΎ ΠΏΠΎ оси x, Π° Π΄ΠΈΠ°ΠΏΠ°Π·ΠΎΠ½ задаСтся Ρ€Π°Π·ΠΌΠ°Ρ…ΠΎΠΌ Π³Ρ€Π°Ρ„ΠΈΠΊΠ° свСрху Π²Π½ΠΈΠ·.

Π”ΠΎΠΌΠ΅Π½ ΠΈ Π΄ΠΈΠ°ΠΏΠ°Π·ΠΎΠ½ ΠΏΠΎ Π³Ρ€Π°Ρ„ΠΈΠΊΡƒ

ΠžΡ‡Π΅Π½ΡŒ Π»Π΅Π³ΠΊΠΎ Π½Π°ΠΉΡ‚ΠΈ Π΄ΠΎΠΌΠ΅Π½ ΠΈ Π΄ΠΈΠ°ΠΏΠ°Π·ΠΎΠ½ Ρ„ΡƒΠ½ΠΊΡ†ΠΈΠΈ, Ссли Π·Π°Π΄Π°Π½/извСстСн Π΅Π΅ Π³Ρ€Π°Ρ„ΠΈΠΊ. Набор Π·Π½Π°Ρ‡Π΅Π½ΠΈΠΉ x, ΠΏΠΎΠΊΡ€Ρ‹Π²Π°Π΅ΠΌΡ‹Ρ… Π³Ρ€Π°Ρ„ΠΈΠΊΠΎΠΌ, Π΄Π°Π΅Ρ‚ Π΄ΠΎΠΌΠ΅Π½, Π° Π½Π°Π±ΠΎΡ€ Π·Π½Π°Ρ‡Π΅Π½ΠΈΠΉ y, ΠΏΠΎΠΊΡ€Ρ‹Π²Π°Π΅ΠΌΡ‹Ρ… Π³Ρ€Π°Ρ„ΠΈΠΊΠΎΠΌ, Π΄Π°Π΅Ρ‚ Π΄ΠΈΠ°ΠΏΠ°Π·ΠΎΠ½. Но ΠΎΠ±Ρ€Π°Ρ‚ΠΈΡ‚Π΅ Π²Π½ΠΈΠΌΠ°Π½ΠΈΠ΅ Π½Π° ΡΠ»Π΅Π΄ΡƒΡŽΡ‰ΠΈΠ΅ Π²Π΅Ρ‰ΠΈ, записывая Π΄ΠΎΠΌΠ΅Π½ ΠΈ Π΄ΠΈΠ°ΠΏΠ°Π·ΠΎΠ½ Π½Π° Π³Ρ€Π°Ρ„ΠΈΠΊΠ΅.

  • ΠŸΡ€ΠΎΠ²Π΅Ρ€ΡŒΡ‚Π΅, ΠΏΡ€ΠΎΡ…ΠΎΠ΄ΠΈΡ‚ Π»ΠΈ Π³Ρ€Π°Ρ„ΠΈΠΊ тСст Π²Π΅Ρ€Ρ‚ΠΈΠΊΠ°Π»ΡŒΠ½ΠΎΠΉ Π»ΠΈΠ½ΠΈΠΈ. Π’ ΠΏΡ€ΠΎΡ‚ΠΈΠ²Π½ΠΎΠΌ случаС это Π½Π΅ функция, ΠΈ ΠΌΡ‹ ΠΎΠ±Ρ‹Ρ‡Π½ΠΎ Π½Π΅ опрСдСляСм ΠΎΠ±Π»Π°ΡΡ‚ΡŒ ΠΈ Π΄ΠΈΠ°ΠΏΠ°Π·ΠΎΠ½ для Ρ‚Π°ΠΊΠΈΡ… ΠΊΡ€ΠΈΠ²Ρ‹Ρ….
  • Если Π½Π° Π³Ρ€Π°Ρ„ΠΈΠΊΠ΅ Π΅ΡΡ‚ΡŒ какая-Ρ‚ΠΎ Π΄Ρ‹Ρ€Π°, Ρ‚ΠΎ Π΅Π΅ ΠΊΠΎΠΎΡ€Π΄ΠΈΠ½Π°Ρ‚Ρ‹ Π½Π΅ Π΄ΠΎΠ»ΠΆΠ½Ρ‹ Π±Ρ‹Ρ‚ΡŒ Π² Π΄ΠΎΠΌΠ΅Π½Π΅ ΠΈ Π΄ΠΈΠ°ΠΏΠ°Π·ΠΎΠ½Π΅.
  • Если Π΅ΡΡ‚ΡŒ Π²Π΅Ρ€Ρ‚ΠΈΠΊΠ°Π»ΡŒΠ½Π°Ρ асимптота, Ρ‚ΠΎ ΡΠΎΠΎΡ‚Π²Π΅Ρ‚ΡΡ‚Π²ΡƒΡŽΡ‰Π΅Π³ΠΎ значСния x Π½Π΅ Π΄ΠΎΠ»ΠΆΠ½ΠΎ Π±Ρ‹Ρ‚ΡŒ Π² области.
  • Если Π΅ΡΡ‚ΡŒ Π³ΠΎΡ€ΠΈΠ·ΠΎΠ½Ρ‚Π°Π»ΡŒΠ½Π°Ρ асимптота, Ρ‚ΠΎ ΡΠΎΠΎΡ‚Π²Π΅Ρ‚ΡΡ‚Π²ΡƒΡŽΡ‰Π΅Π΅ Π·Π½Π°Ρ‡Π΅Π½ΠΈΠ΅ x Π½Π΅ Π΄ΠΎΠ»ΠΆΠ½ΠΎ Π±Ρ‹Ρ‚ΡŒ Π² Π΄ΠΈΠ°ΠΏΠ°Π·ΠΎΠ½Π΅.
  • Если Π³Ρ€Π°Ρ„ Ρ€Π°Π·Π±ΠΈΡ‚ Π½Π° части, Ρ‚ΠΎ ΠΌΡ‹ ΠΏΠΎΠ»ΡƒΡ‡Π°Π΅ΠΌ нСсколько Π½Π°Π±ΠΎΡ€ΠΎΠ²/ΠΈΠ½Ρ‚Π΅Ρ€Π²Π°Π»ΠΎΠ² Π² Π΄ΠΎΠΌΠ΅Π½Π΅ ΠΈ Π΄ΠΈΠ°ΠΏΠ°Π·ΠΎΠ½Π΅ ΠΈ объСдиняСм всС Ρ‚Π°ΠΊΠΈΠ΅ Π½Π°Π±ΠΎΡ€Ρ‹/ΠΈΠ½Ρ‚Π΅Ρ€Π²Π°Π»Ρ‹ символом «объСдинСния» (βˆͺ).
  • Если Π½Π° ΠΊΠΎΠ½Ρ†Π΅ ΠΊΡ€ΠΈΠ²ΠΎΠΉ Π΅ΡΡ‚ΡŒ стрСлка, Ρ‚ΠΎ это ΠΎΠ·Π½Π°Ρ‡Π°Π΅Ρ‚, Ρ‡Ρ‚ΠΎ кривая Π΄ΠΎΠ»ΠΆΠ½Π° бСсконСчно ΠΏΡ€ΠΎΠ΄ΠΎΠ»ΠΆΠ°Ρ‚ΡŒΡΡ Π² этом ΠΊΠΎΠ½ΠΊΡ€Π΅Ρ‚Π½ΠΎΠΌ Π½Π°ΠΏΡ€Π°Π²Π»Π΅Π½ΠΈΠΈ.

Π’ΠΎΡ‚ ΠΏΡ€ΠΈΠΌΠ΅Ρ€ Π³Ρ€Π°Ρ„ΠΈΠΊΠ°, ΠΈ ΠΌΡ‹ Π½Π°ΠΉΠ΄Π΅ΠΌ Π΄ΠΎΠΌΠ΅Π½ ΠΈ Π΄ΠΈΠ°ΠΏΠ°Π·ΠΎΠ½ ΠΈΠ· Π³Ρ€Π°Ρ„ΠΈΠΊΠ°.

На ΠΏΡ€ΠΈΠ²Π΅Π΄Π΅Π½Π½ΠΎΠΌ Π²Ρ‹ΡˆΠ΅ Π³Ρ€Π°Ρ„ΠΈΠΊΠ΅:

  • ВсС значСния x ΠΎΡ‚ -∞ Π΄ΠΎ ∞ ΠΏΠΎΠΊΡ€Ρ‹Π²Π°ΡŽΡ‚ΡΡ Π³Ρ€Π°Ρ„ΠΈΠΊΠΎΠΌ (ΠΈΠ·-Π·Π° стрСлок Π΄Π²Π΅ ΠΊΡ€ΠΈΠ²Ρ‹Π΅ ΠΏΡ€ΠΎΠ΄ΠΎΠ»ΠΆΠ°ΡŽΡ‚ΡΡ бСсконСчно Π² Π·Π°Π΄Π°Π½Π½Ρ‹Ρ… направлСниях). Π‘Π»Π΅Π΄ΠΎΠ²Π°Ρ‚Π΅Π»ΡŒΠ½ΠΎ, ΠΎΠ±Π»Π°ΡΡ‚ΡŒ опрСдСлСния = (-∞, ∞).
  • ВсС значСния y, большиС ΠΈΠ»ΠΈ Ρ€Π°Π²Π½Ρ‹Π΅ ΠΈΠ»ΠΈ Ρ€Π°Π²Π½Ρ‹Π΅ 0, ΠΏΠΎΠΊΡ€Ρ‹Π²Π°ΡŽΡ‚ΡΡ Π³Ρ€Π°Ρ„ΠΈΠΊΠΎΠΌ (см., Ρ‡Ρ‚ΠΎ Π½Π΅Ρ‚ части ΠΊΡ€ΠΈΠ²ΠΎΠΉ, которая находится Π½ΠΈΠΆΠ΅ оси y). Π‘Π»Π΅Π΄ΠΎΠ²Π°Ρ‚Π΅Π»ΡŒΠ½ΠΎ, Π΄ΠΈΠ°ΠΏΠ°Π·ΠΎΠ½ = [0, ∞).

Π’Π°ΠΆΠ½Ρ‹Π΅ примСчания ΠΎΡ‚Π½ΠΎΡΠΈΡ‚Π΅Π»ΡŒΠ½ΠΎ Π΄ΠΎΠΌΠ΅Π½Π° ΠΈ Π΄ΠΈΠ°ΠΏΠ°Π·ΠΎΠ½Π°:

  • Π”ΠΎΠΌΠ΅Π½ ΠΈ Π΄ΠΈΠ°ΠΏΠ°Π·ΠΎΠ½ Ρ„ΡƒΠ½ΠΊΡ†ΠΈΠΈ β€” это Π½Π°Π±ΠΎΡ€ всСх Π²ΠΎΠ·ΠΌΠΎΠΆΠ½Ρ‹Ρ… Π²Ρ…ΠΎΠ΄ΠΎΠ² ΠΈ Π²Ρ‹Ρ…ΠΎΠ΄ΠΎΠ² Ρ„ΡƒΠ½ΠΊΡ†ΠΈΠΈ соотвСтствСнно.
  • ΠžΠ±Π»Π°ΡΡ‚ΡŒ опрСдСлСния ΠΈ Π΄ΠΈΠ°ΠΏΠ°Π·ΠΎΠ½ Ρ„ΡƒΠ½ΠΊΡ†ΠΈΠΈ y = f(x) Π·Π°Π΄Π°ΡŽΡ‚ΡΡ ΠΊΠ°ΠΊ domain= {x ,x∈R }, range= {f(x), x∈Domain}.
  • ΠžΠ±Π»Π°ΡΡ‚ΡŒ опрСдСлСния ΠΈ ΠΎΠ±Π»Π°ΡΡ‚ΡŒ Π·Π½Π°Ρ‡Π΅Π½ΠΈΠΉ любой Ρ„ΡƒΠ½ΠΊΡ†ΠΈΠΈ ΠΌΠΎΠΆΠ½ΠΎ Π½Π°ΠΉΡ‚ΠΈ алгСбраичСски ΠΈΠ»ΠΈ графичСски.

β˜› БвязанныС Ρ‚Π΅ΠΌΡ‹:

  • ГрафичСскиС Ρ„ΡƒΠ½ΠΊΡ†ΠΈΠΈ
  • ΠšΡƒΠ±ΠΈΡ‡Π΅ΡΠΊΠΈΠ΅ Ρ„ΡƒΠ½ΠΊΡ†ΠΈΠΈ
  • ΠžΠ±Ρ€Π°Ρ‚Π½Ρ‹Π΅ тригономСтричСскиС Ρ„ΡƒΠ½ΠΊΡ†ΠΈΠΈ

Часто Π·Π°Π΄Π°Π²Π°Π΅ΠΌΡ‹Π΅ вопросы ΠΎ Π΄ΠΎΠΌΠ΅Π½Π΅ ΠΈ Π΄ΠΈΠ°ΠΏΠ°Π·ΠΎΠ½Π΅

Π§Ρ‚ΠΎ Ρ‚Π°ΠΊΠΎΠ΅ Π΄ΠΎΠΌΠ΅Π½ ΠΈ Π΄ΠΈΠ°ΠΏΠ°Π·ΠΎΠ½ Ρ„ΡƒΠ½ΠΊΡ†ΠΈΠΈ?

Π”ΠΎΠΌΠ΅Π½ ΠΈ Π΄ΠΈΠ°ΠΏΠ°Π·ΠΎΠ½ Ρ„ΡƒΠ½ΠΊΡ†ΠΈΠΈ ΠΏΡ€Π΅Π΄ΡΡ‚Π°Π²Π»ΡΡŽΡ‚ собой Π½Π°Π±ΠΎΡ€ всСх Π²Ρ…ΠΎΠ΄ΠΎΠ² ΠΈ Π²Ρ‹Ρ…ΠΎΠ΄ΠΎΠ², ΠΊΠΎΡ‚ΠΎΡ€Ρ‹Π΅ функция ΠΌΠΎΠΆΠ΅Ρ‚ Π΄Π°Ρ‚ΡŒ соотвСтствСнно. Π”ΠΎΠΌΠ΅Π½ ΠΈ Π΄ΠΈΠ°ΠΏΠ°Π·ΠΎΠ½ ΡΠ²Π»ΡΡŽΡ‚ΡΡ Π²Π°ΠΆΠ½Ρ‹ΠΌΠΈ аспСктами Ρ„ΡƒΠ½ΠΊΡ†ΠΈΠΈ. Π”ΠΎΠΌΠ΅Π½ ΠΏΡ€ΠΈΠ½ΠΈΠΌΠ°Π΅Ρ‚ всС Π²ΠΎΠ·ΠΌΠΎΠΆΠ½Ρ‹Π΅ Π²Ρ…ΠΎΠ΄Π½Ρ‹Π΅ значСния ΠΈΠ· Π½Π°Π±ΠΎΡ€Π° Π΄Π΅ΠΉΡΡ‚Π²ΠΈΡ‚Π΅Π»ΡŒΠ½Ρ‹Ρ… чисСл, Π° Π΄ΠΈΠ°ΠΏΠ°Π·ΠΎΠ½ ΠΏΡ€ΠΈΠ½ΠΈΠΌΠ°Π΅Ρ‚ всС Π²Ρ‹Ρ…ΠΎΠ΄Π½Ρ‹Π΅ значСния Ρ„ΡƒΠ½ΠΊΡ†ΠΈΠΈ.

Как Π·Π°ΠΏΠΈΡΠ°Ρ‚ΡŒ Π΄ΠΎΠΌΠ΅Π½ ΠΈ Π΄ΠΈΠ°ΠΏΠ°Π·ΠΎΠ½?

ΠœΡ‹ пишСм Π΄ΠΎΠΌΠ΅Π½ ΠΈ Π΄ΠΈΠ°ΠΏΠ°Π·ΠΎΠ½ Ρ„ΡƒΠ½ΠΊΡ†ΠΈΠΈ ΠΊΠ°ΠΊ Π½Π°Π±ΠΎΡ€ всСх Π²Ρ…ΠΎΠ΄Π½Ρ‹Ρ… Π΄Π°Π½Π½Ρ‹Ρ…, ΠΊΠΎΡ‚ΠΎΡ€Ρ‹Π΅ функция ΠΌΠΎΠΆΠ΅Ρ‚ ΠΏΡ€ΠΈΠ½ΠΈΠΌΠ°Ρ‚ΡŒ, ΠΈ Π²Ρ‹Ρ…ΠΎΠ΄Π½Ρ‹Ρ… Π΄Π°Π½Π½Ρ‹Ρ… Ρ„ΡƒΠ½ΠΊΡ†ΠΈΠΉ соотвСтствСнно. Π”ΠΎΠΌΠ΅Π½ ΠΈ Π΄ΠΈΠ°ΠΏΠ°Π·ΠΎΠ½ Π·Π°ΠΏΠΈΡΡ‹Π²Π°ΡŽΡ‚ΡΡ ΠΎΡ‚ ΠΌΠ΅Π½ΡŒΡˆΠΈΡ… Π·Π½Π°Ρ‡Π΅Π½ΠΈΠΉ ΠΊ большим значСниям. Π”ΠΎΠΌΠ΅Π½ записываСтся слСва Π½Π°ΠΏΡ€Π°Π²ΠΎ, Π° Π΄ΠΈΠ°ΠΏΠ°Π·ΠΎΠ½ записываСтся свСрху Π²Π½ΠΈΠ· Π³Ρ€Π°Ρ„ΠΈΠΊΠ°.

Π§Ρ‚ΠΎ Ρ‚Π°ΠΊΠΎΠ΅ СстСствСнный Π΄ΠΎΠΌΠ΅Π½ ΠΈ Π΄ΠΈΠ°ΠΏΠ°Π·ΠΎΠ½ Ρ„ΡƒΠ½ΠΊΡ†ΠΈΠΈ?

ЕстСствСнная ΠΎΠ±Π»Π°ΡΡ‚ΡŒ опрСдСлСния ΠΈ ΠΎΠ±Π»Π°ΡΡ‚ΡŒ Π·Π½Π°Ρ‡Π΅Π½ΠΈΠΉ Ρ„ΡƒΠ½ΠΊΡ†ΠΈΠΈ β€” это всС Π²ΠΎΠ·ΠΌΠΎΠΆΠ½Ρ‹Π΅ Π²Ρ…ΠΎΠ΄Π½Ρ‹Π΅ ΠΈ Π²Ρ‹Ρ…ΠΎΠ΄Π½Ρ‹Π΅ значСния Ρ„ΡƒΠ½ΠΊΡ†ΠΈΠΈ соотвСтствСнно. Π”ΠΎΠΌΠ΅Π½ (f) = {x∈R} ΠΈ Π΄ΠΈΠ°ΠΏΠ°Π·ΠΎΠ½ (f) = {f (x): x ∈ domain (f)}.

Π§Ρ‚ΠΎ Ρ‚Π°ΠΊΠΎΠ΅ ΠΎΠ±Π»Π°ΡΡ‚ΡŒ опрСдСлСния ΠΈ Π΄ΠΈΠ°ΠΏΠ°Π·ΠΎΠ½ постоянной Ρ„ΡƒΠ½ΠΊΡ†ΠΈΠΈ?

ΠŸΡƒΡΡ‚ΡŒ постоянная функция Ρ€Π°Π²Π½Π° f(x)=k. ΠžΠ±Π»Π°ΡΡ‚ΡŒ опрСдСлСния постоянной Ρ„ΡƒΠ½ΠΊΡ†ΠΈΠΈ задаСтся R, Ρ‚ΠΎ Π΅ΡΡ‚ΡŒ мноТСством Π΄Π΅ΠΉΡΡ‚Π²ΠΈΡ‚Π΅Π»ΡŒΠ½Ρ‹Ρ… чисСл. Π”ΠΈΠ°ΠΏΠ°Π·ΠΎΠ½ постоянной Ρ„ΡƒΠ½ΠΊΡ†ΠΈΠΈ задаСтся одноэлСмСнтным Π½Π°Π±ΠΎΡ€ΠΎΠΌ {k}. Π”ΠΎΠΌΠ΅Π½ ΠΈ Π΄ΠΈΠ°ΠΏΠ°Π·ΠΎΠ½ постоянной Ρ„ΡƒΠ½ΠΊΡ†ΠΈΠΈ Π·Π°Π΄Π°ΡŽΡ‚ΡΡ ΠΊΠ°ΠΊ domain = x∈R ΠΈ range = {k}, Ρ‡Ρ‚ΠΎ являСтся одноэлСмСнтным Π½Π°Π±ΠΎΡ€ΠΎΠΌ.

Как Π½Π°ΠΉΡ‚ΠΈ ΠΎΠ±Π»Π°ΡΡ‚ΡŒ опрСдСлСния Ρ€Π°Ρ†ΠΈΠΎΠ½Π°Π»ΡŒΠ½ΠΎΠΉ Ρ„ΡƒΠ½ΠΊΡ†ΠΈΠΈ?

Π§Ρ‚ΠΎΠ±Ρ‹ Π½Π°ΠΉΡ‚ΠΈ ΠΎΠ±Π»Π°ΡΡ‚ΡŒ опрСдСлСния Ρ€Π°Ρ†ΠΈΠΎΠ½Π°Π»ΡŒΠ½ΠΎΠΉ Ρ„ΡƒΠ½ΠΊΡ†ΠΈΠΈ, ΠΌΡ‹ просто устанавливаСм Π·Π½Π°ΠΌΠ΅Π½Π°Ρ‚Π΅Π»ΡŒ Π½Π΅ Ρ€Π°Π²Π½Ρ‹ΠΌ Π½ΡƒΠ»ΡŽ. НапримСр, Ρ‡Ρ‚ΠΎΠ±Ρ‹ Π½Π°ΠΉΡ‚ΠΈ ΠΎΠ±Π»Π°ΡΡ‚ΡŒ опрСдСлСния f(x) = 2/(x-3), ΠΌΡ‹ устанавливаСм x-3 β‰  0, Ρ€Π΅ΡˆΠ°Ρ это, ΠΌΡ‹ ΠΏΠΎΠ»ΡƒΡ‡Π°Π΅ΠΌ xβ‰ 3. Π’Π°ΠΊΠΈΠΌ ΠΎΠ±Ρ€Π°Π·ΠΎΠΌ, ΠΎΠ±Π»Π°ΡΡ‚ΡŒΡŽ опрСдСлСния являСтся мноТСство всСх Ρ€Π°Ρ†ΠΈΠΎΠ½Π°Π»ΡŒΠ½Ρ‹Ρ… чисСл, ΠΊΡ€ΠΎΠΌΠ΅ 3. Π’ ΠΈΠ½Ρ‚Π΅Ρ€Π²Π°Π»ΡŒΠ½ΠΎΠΉ записи это ΠΌΠΎΠΆΠ½ΠΎ Π·Π°ΠΏΠΈΡΠ°Ρ‚ΡŒ ΠΊΠ°ΠΊ (-∞, 3) U (3, ∞).

Как Π½Π°ΠΉΡ‚ΠΈ Π΄ΠΈΠ°ΠΏΠ°Π·ΠΎΠ½ Ρ€Π°Ρ†ΠΈΠΎΠ½Π°Π»ΡŒΠ½ΠΎΠΉ Ρ„ΡƒΠ½ΠΊΡ†ΠΈΠΈ?

Π§Ρ‚ΠΎΠ±Ρ‹ Π½Π°ΠΉΡ‚ΠΈ Π΄ΠΈΠ°ΠΏΠ°Π·ΠΎΠ½ Ρ€Π°Ρ†ΠΈΠΎΠ½Π°Π»ΡŒΠ½ΠΎΠΉ Ρ„ΡƒΠ½ΠΊΡ†ΠΈΠΈ, ΠΌΡ‹ просто Ρ€Π΅ΡˆΠ°Π΅ΠΌ ΡƒΡ€Π°Π²Π½Π΅Π½ΠΈΠ΅ для x ΠΈ примСняСм ΡƒΡΡ‚Π°Π½ΠΎΠ²ΠΈΡ‚ΡŒ Π·Π½Π°ΠΌΠ΅Π½Π°Ρ‚Π΅Π»ΡŒ Π½Π΅ Ρ€Π°Π²Π½Ρ‹ΠΌ Π½ΡƒΠ»ΡŽ. НапримСр, Ρ‡Ρ‚ΠΎΠ±Ρ‹ Π½Π°ΠΉΡ‚ΠΈ Π΄ΠΈΠ°ΠΏΠ°Π·ΠΎΠ½ y=2/(x-3), сначала Ρ€Π΅ΡˆΠΈΡ‚Π΅ Π΅Π³ΠΎ для x. Π’ΠΎΠ³Π΄Π° ΠΌΡ‹ ΠΏΠΎΠ»ΡƒΡ‡Π°Π΅ΠΌ x-3 = 2/y ΠΈ ΠΎΡ‚ΡΡŽΠ΄Π° x = (2/y) + 3. Π’ΠΎΠ³Π΄Π° Π΅Π³ΠΎ Π΄ΠΈΠ°ΠΏΠ°Π·ΠΎΠ½ Ρ€Π°Π²Π΅Π½ yβ‰ 0 (ΠΈΠ»ΠΈ) Π² ΠΈΠ½Ρ‚Π΅Ρ€Π²Π°Π»ΡŒΠ½ΠΎΠΉ записи, (-∞, 0) U (0, ∞ ).

ΠšΠ°ΠΊΠΎΠ²Ρ‹ ΠΏΡ€Π°Π²ΠΈΠ»Π° опрСдСлСния области опрСдСлСния Ρ„ΡƒΠ½ΠΊΡ†ΠΈΠΈ?

Π’ΠΎΡ‚ нСсколько ΠΎΠ±Ρ‰ΠΈΡ… ΠΏΡ€Π°Π²ΠΈΠ», ΠΈΡΠΏΠΎΠ»ΡŒΠ·ΡƒΠ΅ΠΌΡ‹Ρ… для опрСдСлСния Π΄ΠΎΠΌΠ΅Π½Π° Ρ€Π°Π·Π»ΠΈΡ‡Π½Ρ‹Ρ… Ρ‚ΠΈΠΏΠΎΠ² Ρ„ΡƒΠ½ΠΊΡ†ΠΈΠΉ:

  • f(x) = ΠΌΠ½ΠΎΠ³ΠΎΡ‡Π»Π΅Π½, ΠΎΠ±Π»Π°ΡΡ‚ΡŒΡŽ опрСдСлСния являСтся мноТСство всСх Π΄Π΅ΠΉΡΡ‚Π²ΠΈΡ‚Π΅Π»ΡŒΠ½Ρ‹Ρ… чисСл.
  • f(x) = 1/x, Π΄ΠΎΠΌΠ΅Π½, Ссли мноТСство всСх Π΄Π΅ΠΉΡΡ‚Π²ΠΈΡ‚Π΅Π»ΡŒΠ½Ρ‹Ρ… чисСл, ΠΊΡ€ΠΎΠΌΠ΅ xβ‰ 0.
  • f(x) = √x, Π΄ΠΎΠΌΠ΅Π½, Ссли мноТСство всСх Π΄Π΅ΠΉΡΡ‚Π²ΠΈΡ‚Π΅Π»ΡŒΠ½Ρ‹Ρ… чисСл, Ρ‚Π°ΠΊΠΈΡ… Ρ‡Ρ‚ΠΎ x β‰₯ 0.
  • f(x) = ln x, ΠΎΠ±Π»Π°ΡΡ‚ΡŒΡŽ опрСдСлСния являСтся мноТСство всСх Π΄Π΅ΠΉΡΡ‚Π²ΠΈΡ‚Π΅Π»ΡŒΠ½Ρ‹Ρ… чисСл, для ΠΊΠΎΡ‚ΠΎΡ€Ρ‹Ρ… x > 0.

Как алгСбраичСски Π½Π°ΠΉΡ‚ΠΈ ΠΎΠ±Π»Π°ΡΡ‚ΡŒ опрСдСлСния ΠΈ Π΄ΠΈΠ°ΠΏΠ°Π·ΠΎΠ½ Ρ„ΡƒΠ½ΠΊΡ†ΠΈΠΉ?

ΠŸΡƒΡΡ‚ΡŒ функция Ρ€Π°Π²Π½Π° y=f(x). НайдСм ΠΎΠ±Π»Π°ΡΡ‚ΡŒ опрСдСлСния ΠΈ ΠΎΠ±Π»Π°ΡΡ‚ΡŒ Π·Π½Π°Ρ‡Π΅Π½ΠΈΠΉ этой Ρ„ΡƒΠ½ΠΊΡ†ΠΈΠΈ алгСбраичСски.

Π§Ρ‚ΠΎΠ±Ρ‹ Π²Ρ‹Ρ‡ΠΈΡΠ»ΠΈΡ‚ΡŒ ΠΎΠ±Π»Π°ΡΡ‚ΡŒ опрСдСлСния Ρ„ΡƒΠ½ΠΊΡ†ΠΈΠΈ, ΠΌΡ‹ просто Ρ€Π΅ΡˆΠ°Π΅ΠΌ ΡƒΡ€Π°Π²Π½Π΅Π½ΠΈΠ΅ для опрСдСлСния Π·Π½Π°Ρ‡Π΅Π½ΠΈΠΉ нСзависимой ΠΏΠ΅Ρ€Π΅ΠΌΠ΅Π½Π½ΠΎΠΉ x. Π§Ρ‚ΠΎΠ±Ρ‹ Π²Ρ‹Ρ‡ΠΈΡΠ»ΠΈΡ‚ΡŒ Π΄ΠΈΠ°ΠΏΠ°Π·ΠΎΠ½ Ρ„ΡƒΠ½ΠΊΡ†ΠΈΠΈ, ΠΌΡ‹ просто Π²Ρ‹Ρ€Π°ΠΆΠ°Π΅ΠΌ x ΠΊΠ°ΠΊ x = g (y), Π° Π·Π°Ρ‚Π΅ΠΌ Π½Π°Ρ…ΠΎΠ΄ΠΈΠΌ ΠΎΠ±Π»Π°ΡΡ‚ΡŒ опрСдСлСния g (y).

Как Π½Π°ΠΉΡ‚ΠΈ ΠΎΠ±Π»Π°ΡΡ‚ΡŒ опрСдСлСния ΠΈ ΠΎΠ±Π»Π°ΡΡ‚ΡŒ Π·Π½Π°Ρ‡Π΅Π½ΠΈΠΉ уравнСния?

Π§Ρ‚ΠΎΠ±Ρ‹ Π½Π°ΠΉΡ‚ΠΈ Π΄ΠΎΠΌΠ΅Π½ ΠΈ Π΄ΠΈΠ°ΠΏΠ°Π·ΠΎΠ½, ΠΌΡ‹ просто Ρ€Π΅ΡˆΠ°Π΅ΠΌ ΡƒΡ€Π°Π²Π½Π΅Π½ΠΈΠ΅ y = f(x), Ρ‡Ρ‚ΠΎΠ±Ρ‹ ΠΎΠΏΡ€Π΅Π΄Π΅Π»ΠΈΡ‚ΡŒ значСния нСзависимой ΠΏΠ΅Ρ€Π΅ΠΌΠ΅Π½Π½ΠΎΠΉ x ΠΈ ΠΏΠΎΠ»ΡƒΡ‡ΠΈΡ‚ΡŒ Π΄ΠΎΠΌΠ΅Π½. Π§Ρ‚ΠΎΠ±Ρ‹ Π²Ρ‹Ρ‡ΠΈΡΠ»ΠΈΡ‚ΡŒ Π΄ΠΈΠ°ΠΏΠ°Π·ΠΎΠ½ Ρ„ΡƒΠ½ΠΊΡ†ΠΈΠΈ, ΠΌΡ‹ просто Π²Ρ‹Ρ€Π°ΠΆΠ°Π΅ΠΌ x ΠΊΠ°ΠΊ x = g (y), Π° Π·Π°Ρ‚Π΅ΠΌ Π½Π°Ρ…ΠΎΠ΄ΠΈΠΌ ΠΎΠ±Π»Π°ΡΡ‚ΡŒ опрСдСлСния g (y).

Как Ρ€Π°ΡΡΡ‡ΠΈΡ‚Π°Ρ‚ΡŒ Π΄ΠΎΠΌΠ΅Π½ ΠΈ Π΄ΠΈΠ°ΠΏΠ°Π·ΠΎΠ½ ΠΏΠΎ Π³Ρ€Π°Ρ„ΠΈΠΊΡƒ Ρ„ΡƒΠ½ΠΊΡ†ΠΈΠΈ?

Набор всСх ΠΊΠΎΠΎΡ€Π΄ΠΈΠ½Π°Ρ‚ x всСх Ρ‚ΠΎΡ‡Π΅ΠΊ ΠΊΡ€ΠΈΠ²ΠΎΠΉ Π΄Π°Π΅Ρ‚ Π΄ΠΎΠΌΠ΅Π½, Π° Π½Π°Π±ΠΎΡ€ всСх ΠΊΠΎΠΎΡ€Π΄ΠΈΠ½Π°Ρ‚ y всСх Ρ‚ΠΎΡ‡Π΅ΠΊ ΠΊΡ€ΠΈΠ²ΠΎΠΉ Π΄Π°Π΅Ρ‚ Π΄ΠΈΠ°ΠΏΠ°Π·ΠΎΠ½. ΠšΠ°ΠΆΠ΄Ρ‹ΠΉ ΠΈΠ· Π΄ΠΎΠΌΠ΅Π½ΠΎΠ² ΠΈ Π΄ΠΈΠ°ΠΏΠ°Π·ΠΎΠ½ΠΎΠ² ΠΌΠΎΠΆΠ΅Ρ‚ Π±Ρ‹Ρ‚ΡŒ записан ΠΊΠ°ΠΊ Π½Π°Π±ΠΎΡ€ ΠΈΠ»ΠΈ ΠΈΠ½Ρ‚Π΅Ρ€Π²Π°Π».

Π’ Ρ‡Π΅ΠΌ Ρ€Π°Π·Π½ΠΈΡ†Π° ΠΌΠ΅ΠΆΠ΄Ρƒ Π΄ΠΎΠΌΠ΅Π½ΠΎΠΌ ΠΈ Π΄ΠΈΠ°ΠΏΠ°Π·ΠΎΠ½ΠΎΠΌ Ρ„ΡƒΠ½ΠΊΡ†ΠΈΠΈ?

Π”ΠΎΠΌΠ΅Π½ ΠΈ Π΄ΠΈΠ°ΠΏΠ°Π·ΠΎΠ½ Ρ„ΡƒΠ½ΠΊΡ†ΠΈΠΈ ΡΠ²Π»ΡΡŽΡ‚ΡΡ ΠΊΠΎΠΌΠΏΠΎΠ½Π΅Π½Ρ‚Π°ΠΌΠΈ Ρ„ΡƒΠ½ΠΊΡ†ΠΈΠΈ. ΠžΠ±Π»Π°ΡΡ‚ΡŒ опрСдСлСния Ρ„ΡƒΠ½ΠΊΡ†ΠΈΠΈ β€” это Π½Π°Π±ΠΎΡ€ всСх Π²ΠΎΠ·ΠΌΠΎΠΆΠ½Ρ‹Ρ… Π²Ρ…ΠΎΠ΄Π½Ρ‹Ρ… Π΄Π°Π½Π½Ρ‹Ρ… для Ρ„ΡƒΠ½ΠΊΡ†ΠΈΠΈ, Ρ‚ΠΎΠ³Π΄Π° ΠΊΠ°ΠΊ Π΄ΠΈΠ°ΠΏΠ°Π·ΠΎΠ½ Ρ„ΡƒΠ½ΠΊΡ†ΠΈΠΈ β€” это Π½Π°Π±ΠΎΡ€ всСх Π²Ρ‹Ρ…ΠΎΠ΄Π½Ρ‹Ρ… Π΄Π°Π½Π½Ρ‹Ρ…, ΠΊΠΎΡ‚ΠΎΡ€Ρ‹Π΅ ΠΌΠΎΠΆΠ΅Ρ‚ Π΄Π°Ρ‚ΡŒ функция.

Π§Ρ‚ΠΎ Ρ‚Π°ΠΊΠΎΠ΅ Π΄ΠΎΠΌΠ΅Π½ ΠΈ Π΄ΠΈΠ°ΠΏΠ°Π·ΠΎΠ½ ΠΎΡ‚Π½ΠΎΡˆΠ΅Π½ΠΈΡ?

Π΄ΠΎΠΌΠ΅Π½ ΠΈ Π΄ΠΈΠ°ΠΏΠ°Π·ΠΎΠ½ ΠΎΡ‚Π½ΠΎΡˆΠ΅Π½ΠΈΡ находятся ΡΠ»Π΅Π΄ΡƒΡŽΡ‰ΠΈΠΌ ΠΎΠ±Ρ€Π°Π·ΠΎΠΌ. ΠŸΡƒΡΡ‚ΡŒ R β€” ΠΎΡ‚Π½ΠΎΡˆΠ΅Π½ΠΈΠ΅ нСпустого мноТСства A ΠΊ нСпустому мноТСству B. ΠžΠ±Π»Π°ΡΡ‚ΡŒ опрСдСлСния ΠΈ Π΄ΠΈΠ°ΠΏΠ°Π·ΠΎΠ½ ΠΎΡ‚Π½ΠΎΡˆΠ΅Π½ΠΈΡ β€” это мноТСство ΠΏΠ΅Ρ€Π²Ρ‹Ρ… элСмСнтов ΠΈ Π²Ρ‚ΠΎΡ€Ρ‹Ρ… элСмСнтов соотвСтствСнно Π² упорядочСнных ΠΏΠ°Ρ€Π°Ρ… Π² ΠΎΡ‚Π½ΠΎΡˆΠ΅Π½ΠΈΠΈ R, Π½Π°Π·Ρ‹Π²Π°Π΅ΠΌΠΎΠ΅ Π΄ΠΎΠΌΠ΅Π½ΠΎΠΌ.

Π§Ρ‚ΠΎ Ρ‚Π°ΠΊΠΎΠ΅ Π΄ΠΎΠΌΠ΅Π½ ΠΈ Π΄ΠΈΠ°ΠΏΠ°Π·ΠΎΠ½ составных Ρ„ΡƒΠ½ΠΊΡ†ΠΈΠΉ?

ΠŸΡƒΡΡ‚ΡŒ составная функция Ρ€Π°Π²Π½Π° \(h=f \circ g\). ΠžΠ±Π»Π°ΡΡ‚ΡŒ опрСдСлСния ΠΈ Π΄ΠΈΠ°ΠΏΠ°Π·ΠΎΠ½ Π·Π½Π°Ρ‡Π΅Π½ΠΈΠΉ h ΠΎΠΏΡ€Π΅Π΄Π΅Π»ΡΡŽΡ‚ΡΡ ΡΠ»Π΅Π΄ΡƒΡŽΡ‰ΠΈΠΌ ΠΎΠ±Ρ€Π°Π·ΠΎΠΌ. ΠžΠ±Π»Π°ΡΡ‚ΡŒ опрСдСлСния h Π»ΠΈΠ±ΠΎ совпадаСт с ΠΎΠ±Π»Π°ΡΡ‚ΡŒΡŽ опрСдСлСния f, Π»ΠΈΠ±ΠΎ Π»Π΅ΠΆΠΈΡ‚ Π² ΠΏΡ€Π΅Π΄Π΅Π»Π°Ρ… области опрСдСлСния f. Π”ΠΈΠ°ΠΏΠ°Π·ΠΎΠ½ h Π΄ΠΎΠ»ΠΆΠ΅Π½ Π»Π΅ΠΆΠ°Ρ‚ΡŒ Π² Π΄ΠΈΠ°ΠΏΠ°Π·ΠΎΠ½Π΅ g. ΠŸΡƒΡΡ‚ΡŒ f(x) = x 2 ΠΈ g(x) = x+ 3. ΠœΡ‹ Π·Π½Π°Π΅ΠΌ, Ρ‡Ρ‚ΠΎ f: X β†’ Y ΠΈ g: Y β†’ Z. Π—Π°Ρ‚Π΅ΠΌ Ρ‚ΡƒΠΌΠ°Π½: X β†’ Z. f(g(x)) = (x+3) 2 . Π’Π°ΠΊΠΈΠΌ ΠΎΠ±Ρ€Π°Π·ΠΎΠΌ, Π΄ΠΎΠΌΠ΅Π½ ΠΈ Π΄ΠΈΠ°ΠΏΠ°Π·ΠΎΠ½: domain= {ВсС элСмСнты мноТСства X}, range= {всС элСмСнты мноТСства Z}

Π§Ρ‚ΠΎ Ρ‚Π°ΠΊΠΎΠ΅ Π΄ΠΎΠΌΠ΅Π½ ΠΈ Π΄ΠΈΠ°ΠΏΠ°Π·ΠΎΠ½ ΠΊΠ²Π°Π΄Ρ€Π°Ρ‚ΠΈΡ‡Π½ΠΎΠΉ Ρ„ΡƒΠ½ΠΊΡ†ΠΈΠΈ?

ΠžΠ±Π»Π°ΡΡ‚ΡŒ опрСдСлСния ΠΈ ΠΎΠ±Π»Π°ΡΡ‚ΡŒ Π·Π½Π°Ρ‡Π΅Π½ΠΈΠΉ ΠΊΠ²Π°Π΄Ρ€Π°Ρ‚ΠΈΡ‡Π½ΠΎΠΉ Ρ„ΡƒΠ½ΠΊΡ†ΠΈΠΈ y=a(x-h) 2 +k ΠΎΠΏΡ€Π΅Π΄Π΅Π»ΡΡŽΡ‚ Ρ…Π°Ρ€Π°ΠΊΡ‚Π΅Ρ€ ΠΏΠ°Ρ€Π°Π±ΠΎΠ»Ρ‹: Π½Π°ΠΏΡ€Π°Π²Π»Π΅Π½Π° ​​ли ΠΎΠ½Π° Π²Π²Π΅Ρ€Ρ… ΠΈΠ»ΠΈ Π²Π½ΠΈΠ·, Π½Π°ΠΏΡ€Π°Π²Π»Π΅Π½Π° ​​ли ΠΎΠ½Π° Π²Π»Π΅Π²ΠΎ ΠΈΠ»ΠΈ Π²ΠΏΡ€Π°Π²ΠΎ.

  • y β‰₯ k, Ссли функция ΠΈΠΌΠ΅Π΅Ρ‚ минимальноС Π·Π½Π°Ρ‡Π΅Π½ΠΈΠ΅, Ρ‚ΠΎ Π΅ΡΡ‚ΡŒ ΠΊΠΎΠ³Π΄Π° a>0(ΠΏΠ°Ρ€Π°Π±ΠΎΠ»Π° раскрываСтся)
  • y ≀ k, Ссли функция ΠΈΠΌΠ΅Π΅Ρ‚ максимальноС Π·Π½Π°Ρ‡Π΅Π½ΠΈΠ΅, Ρ‚ΠΎ Π΅ΡΡ‚ΡŒ ΠΊΠΎΠ³Π΄Π° a<0(ΠΏΠ°Ρ€Π°Π±ΠΎΠ»Π° раскрываСтся Π²Π½ΠΈΠ·)

Π—Π°ΠΏΠΈΡΡŒ области опрСдСлСния ΠΈ Π΄ΠΈΠ°ΠΏΠ°Π·ΠΎΠ½Π° ΠΏΠΎ ΡƒΡ€Π°Π²Π½Π΅Π½ΠΈΡŽ

Π Π΅Π·ΡƒΠ»ΡŒΡ‚Π°Ρ‚Ρ‹ обучСния

  • НахоТдСниС области опрСдСлСния Ρ„ΡƒΠ½ΠΊΡ†ΠΈΠΈ, Π·Π°Π΄Π°Π½Π½ΠΎΠΉ ΡƒΡ€Π°Π²Π½Π΅Π½ΠΈΠ΅ΠΌ.
  • Π—Π°ΠΏΠΈΡˆΠΈΡ‚Π΅ Π΄ΠΎΠΌΠ΅Π½ ΠΈ Π΄ΠΈΠ°ΠΏΠ°Π·ΠΎΠ½, ΠΈΡΠΏΠΎΠ»ΡŒΠ·ΡƒΡ стандартныС обозначСния.

Π’ Ρ€Π°Π·Π΄Π΅Π»Π΅ Π€ΡƒΠ½ΠΊΡ†ΠΈΠΈ ΠΈ обозначСния Ρ„ΡƒΠ½ΠΊΡ†ΠΈΠΉ ΠΌΡ‹ познакомились с понятиями Π΄ΠΎΠΌΠ΅Π½Π° ΠΈ Π΄ΠΈΠ°ΠΏΠ°Π·ΠΎΠ½Π° . Π’ этом Ρ€Π°Π·Π΄Π΅Π»Π΅ ΠΌΡ‹ попрактикуСмся Π² ΠΎΠΏΡ€Π΅Π΄Π΅Π»Π΅Π½ΠΈΠΈ Π΄ΠΎΠΌΠ΅Π½ΠΎΠ² ΠΈ Π΄ΠΈΠ°ΠΏΠ°Π·ΠΎΠ½ΠΎΠ² для ΠΊΠΎΠ½ΠΊΡ€Π΅Ρ‚Π½Ρ‹Ρ… Ρ„ΡƒΠ½ΠΊΡ†ΠΈΠΉ. Π˜ΠΌΠ΅ΠΉΡ‚Π΅ Π² Π²ΠΈΠ΄Ρƒ, Ρ‡Ρ‚ΠΎ ΠΏΡ€ΠΈ ΠΎΠΏΡ€Π΅Π΄Π΅Π»Π΅Π½ΠΈΠΈ Π΄ΠΎΠΌΠ΅Π½ΠΎΠ² ΠΈ Π΄ΠΈΠ°ΠΏΠ°Π·ΠΎΠ½ΠΎΠ² Π½Π°ΠΌ Π½Π΅ΠΎΠ±Ρ…ΠΎΠ΄ΠΈΠΌΠΎ ΡƒΡ‡ΠΈΡ‚Ρ‹Π²Π°Ρ‚ΡŒ, Ρ‡Ρ‚ΠΎ физичСски Π²ΠΎΠ·ΠΌΠΎΠΆΠ½ΠΎ ΠΈΠ»ΠΈ Π·Π½Π°Ρ‡ΠΈΠΌΠΎ Π² Ρ€Π΅Π°Π»ΡŒΠ½Ρ‹Ρ… ΠΏΡ€ΠΈΠΌΠ΅Ρ€Π°Ρ…, Ρ‚Π°ΠΊΠΈΡ… ΠΊΠ°ΠΊ ΠΏΡ€ΠΎΠ΄Π°ΠΆΠΈ Π±ΠΈΠ»Π΅Ρ‚ΠΎΠ² ΠΈ Π³ΠΎΠ΄ Π² ΠΏΡ€ΠΈΠ²Π΅Π΄Π΅Π½Π½ΠΎΠΌ Π²Ρ‹ΡˆΠ΅ ΠΏΡ€ΠΈΠΌΠ΅Ρ€Π΅ с Ρ„ΠΈΠ»ΡŒΠΌΠΎΠΌ уТасов. ΠœΡ‹ Ρ‚Π°ΠΊΠΆΠ΅ Π΄ΠΎΠ»ΠΆΠ½Ρ‹ Ρ€Π°ΡΡΠΌΠΎΡ‚Ρ€Π΅Ρ‚ΡŒ, Ρ‡Ρ‚ΠΎ матСматичСски Ρ€Π°Π·Ρ€Π΅ΡˆΠ΅Π½ΠΎ. НапримСр, ΠΌΡ‹ Π½Π΅ ΠΌΠΎΠΆΠ΅ΠΌ Π²ΠΊΠ»ΡŽΡ‡Π°Ρ‚ΡŒ ΠΊΠ°ΠΊΠΎΠ΅-Π»ΠΈΠ±ΠΎ Π²Ρ…ΠΎΠ΄Π½ΠΎΠ΅ Π·Π½Π°Ρ‡Π΅Π½ΠΈΠ΅, ΠΊΠΎΡ‚ΠΎΡ€ΠΎΠ΅ ΠΏΡ€ΠΈΠ²ΠΎΠ΄ΠΈΡ‚ ΠΊ ΠΈΠ·Π²Π»Π΅Ρ‡Π΅Π½ΠΈΡŽ Ρ‡Π΅Ρ‚Π½ΠΎΠ³ΠΎ корня ΠΈΠ· ΠΎΡ‚Ρ€ΠΈΡ†Π°Ρ‚Π΅Π»ΡŒΠ½ΠΎΠ³ΠΎ числа, Ссли Π΄ΠΎΠΌΠ΅Π½ ΠΈ Π΄ΠΈΠ°ΠΏΠ°Π·ΠΎΠ½ состоят ΠΈΠ· Π΄Π΅ΠΉΡΡ‚Π²ΠΈΡ‚Π΅Π»ΡŒΠ½Ρ‹Ρ… чисСл. Или Π² Ρ„ΡƒΠ½ΠΊΡ†ΠΈΠΈ, Π²Ρ‹Ρ€Π°ΠΆΠ΅Π½Π½ΠΎΠΉ Π² Π²ΠΈΠ΄Π΅ Ρ„ΠΎΡ€ΠΌΡƒΠ»Ρ‹, ΠΌΡ‹ Π½Π΅ ΠΌΠΎΠΆΠ΅ΠΌ Π²ΠΊΠ»ΡŽΡ‡ΠΈΡ‚ΡŒ ΠΊΠ°ΠΊΠΎΠ΅-Π»ΠΈΠ±ΠΎ Π²Ρ…ΠΎΠ΄Π½ΠΎΠ΅ Π·Π½Π°Ρ‡Π΅Π½ΠΈΠ΅ Π² ΠΎΠ±Π»Π°ΡΡ‚ΡŒ опрСдСлСния, которая ΠΏΡ€ΠΈΠ²Π΅Π»Π° Π±Ρ‹ ΠΊ дСлСнию Π½Π° 0,9.1811

ΠœΡ‹ ΠΌΠΎΠΆΠ΅ΠΌ ΠΏΡ€Π΅Π΄ΡΡ‚Π°Π²ΠΈΡ‚ΡŒ Π΄ΠΎΠΌΠ΅Π½ ΠΊΠ°ΠΊ Β«Π·ΠΎΠ½Ρƒ хранСния», ΡΠΎΠ΄Π΅Ρ€ΠΆΠ°Ρ‰ΡƒΡŽ Β«ΡΡ‹Ρ€ΡŒΠ΅Β» для Β«Ρ„ΡƒΠ½ΠΊΡ†ΠΈΠΎΠ½Π°Π»ΡŒΠ½ΠΎΠΉ ΠΌΠ°ΡˆΠΈΠ½Ρ‹Β», Π° ассортимСнт β€” ΠΊΠ°ΠΊ Π΅Ρ‰Π΅ ΠΎΠ΄Π½Ρƒ Β«Π·ΠΎΠ½Ρƒ хранСния» для ΠΏΡ€ΠΎΠ΄ΡƒΠΊΡ‚ΠΎΠ² ΠΌΠ°ΡˆΠΈΠ½Ρ‹.

ΠœΡ‹ ΠΌΠΎΠΆΠ΅ΠΌ Π·Π°ΠΏΠΈΡΠ°Ρ‚ΡŒ Π΄ΠΎΠΌΠ΅Π½ ΠΈ Π΄ΠΈΠ°ΠΏΠ°Π·ΠΎΠ½ Π² ΠΈΠ½Ρ‚Π΅Ρ€Π²Π°Π»ΡŒΠ½ΠΎΠΉ Π½ΠΎΡ‚Π°Ρ†ΠΈΠΈ , которая ΠΈΡΠΏΠΎΠ»ΡŒΠ·ΡƒΠ΅Ρ‚ значСния Π² ΠΊΠ²Π°Π΄Ρ€Π°Ρ‚Π½Ρ‹Ρ… скобках для описания Π½Π°Π±ΠΎΡ€Π° чисСл. Π’ ΠΎΠ±ΠΎΠ·Π½Π°Ρ‡Π΅Π½ΠΈΠΈ ΠΈΠ½Ρ‚Π΅Ρ€Π²Π°Π»Π° ΠΌΡ‹ ΠΈΡΠΏΠΎΠ»ΡŒΠ·ΡƒΠ΅ΠΌ ΠΊΠ²Π°Π΄Ρ€Π°Ρ‚Π½ΡƒΡŽ скобку [ ΠΊΠΎΠ³Π΄Π° Π½Π°Π±ΠΎΡ€ Π²ΠΊΠ»ΡŽΡ‡Π°Π΅Ρ‚ ΠΊΠΎΠ½Π΅Ρ‡Π½ΡƒΡŽ Ρ‚ΠΎΡ‡ΠΊΡƒ, ΠΈ ΠΊΡ€ΡƒΠ³Π»ΡƒΡŽ скобку (, Ρ‡Ρ‚ΠΎΠ±Ρ‹ ΡƒΠΊΠ°Π·Π°Ρ‚ΡŒ, Ρ‡Ρ‚ΠΎ конСчная Ρ‚ΠΎΡ‡ΠΊΠ° Π»ΠΈΠ±ΠΎ Π½Π΅ Π²ΠΊΠ»ΡŽΡ‡Π΅Π½Π°, Π»ΠΈΠ±ΠΎ ΠΈΠ½Ρ‚Π΅Ρ€Π²Π°Π» Π½Π΅ ΠΎΠ³Ρ€Π°Π½ΠΈΡ‡Π΅Π½. НапримСр, Ссли Ρƒ Ρ‡Π΅Π»ΠΎΠ²Π΅ΠΊΠ° Π΅ΡΡ‚ΡŒ 100 Π΄ΠΎΠ»Π»Π°Ρ€ΠΎΠ², ΠΊΠΎΡ‚ΠΎΡ€Ρ‹Π΅ ΠΎΠ½ ΠΌΠΎΠΆΠ΅Ρ‚ ΠΏΠΎΡ‚Ρ€Π°Ρ‚ΠΈΡ‚ΡŒ, ΠΎΠ½ ΠΈΠ»ΠΈ ΠΎΠ½Π° Π½ΡƒΠΆΠ½ΠΎ Π²Ρ‹Ρ€Π°Π·ΠΈΡ‚ΡŒ ΠΈΠ½Ρ‚Π΅Ρ€Π²Π°Π», ΠΊΠΎΡ‚ΠΎΡ€Ρ‹ΠΉ большС 0 ΠΈ мСньшС ΠΈΠ»ΠΈ Ρ€Π°Π²Π΅Π½ 100, ΠΈ Π½Π°ΠΏΠΈΡΠ°Ρ‚ΡŒ [латСкс]\Π²Π»Π΅Π²ΠΎ(0,\тСкст{ }100\Π²ΠΏΡ€Π°Π²ΠΎ][/латСкс]. ΠžΠ±ΠΎΠ·Π½Π°Ρ‡Π΅Π½ΠΈΠ΅ ΠΈΠ½Ρ‚Π΅Ρ€Π²Π°Π»Π° ΠΌΡ‹ обсудим Π±ΠΎΠ»Π΅Π΅ ΠΏΠΎΠ΄Ρ€ΠΎΠ±Π½ΠΎ ΠΏΠΎΠ·ΠΆΠ΅.

ΠžΠ±Ρ€Π°Ρ‚ΠΈΠΌΡΡ ΠΊ поиску области опрСдСлСния Ρ„ΡƒΠ½ΠΊΡ†ΠΈΠΈ, ΡƒΡ€Π°Π²Π½Π΅Π½ΠΈΠ΅ ΠΊΠΎΡ‚ΠΎΡ€ΠΎΠΉ ΠΏΡ€ΠΈΠ²Π΅Π΄Π΅Π½ΠΎ. Часто для нахоТдСния области опрСдСлСния Ρ‚Π°ΠΊΠΈΡ… Ρ„ΡƒΠ½ΠΊΡ†ΠΈΠΉ Π½Π΅ΠΎΠ±Ρ…ΠΎΠ΄ΠΈΠΌΠΎ Π·Π°ΠΏΠΎΠΌΠ½ΠΈΡ‚ΡŒ Ρ‚Ρ€ΠΈ Ρ€Π°Π·Π½Ρ‹Π΅ Ρ„ΠΎΡ€ΠΌΡ‹. Π’ΠΎ-ΠΏΠ΅Ρ€Π²Ρ‹Ρ…, Ссли функция Π½Π΅ ΠΈΠΌΠ΅Π΅Ρ‚ знамСнатСля ΠΈΠ»ΠΈ Ρ‡Π΅Ρ‚Π½ΠΎΠ³ΠΎ корня, ΠΏΠΎΠ΄ΡƒΠΌΠ°ΠΉΡ‚Π΅, ΠΌΠΎΠ³ΡƒΡ‚ Π»ΠΈ Π΄ΠΎΠΌΠ΅Π½ΠΎΠΌ Π±Ρ‹Ρ‚ΡŒ всС Π΄Π΅ΠΉΡΡ‚Π²ΠΈΡ‚Π΅Π»ΡŒΠ½Ρ‹Π΅ числа. Π’ΠΎ-Π²Ρ‚ΠΎΡ€Ρ‹Ρ…, Ссли Π² ΡƒΡ€Π°Π²Π½Π΅Π½ΠΈΠΈ Ρ„ΡƒΠ½ΠΊΡ†ΠΈΠΈ Π΅ΡΡ‚ΡŒ Π·Π½Π°ΠΌΠ΅Π½Π°Ρ‚Π΅Π»ΡŒ, ΠΈΡΠΊΠ»ΡŽΡ‡ΠΈΡ‚Π΅ значСния Π² области Π·Π½Π°Ρ‡Π΅Π½ΠΈΠΉ, ΠΏΡ€ΠΈ ΠΊΠΎΡ‚ΠΎΡ€Ρ‹Ρ… Π·Π½Π°ΠΌΠ΅Π½Π°Ρ‚Π΅Π»ΡŒ Ρ€Π°Π²Π΅Π½ Π½ΡƒΠ»ΡŽ. Π’-Ρ‚Ρ€Π΅Ρ‚ΡŒΠΈΡ…, Ссли Π΅ΡΡ‚ΡŒ Ρ‡Π΅Ρ‚Π½Ρ‹ΠΉ ΠΊΠΎΡ€Π΅Π½ΡŒ, рассмотритС Π²ΠΎΠ·ΠΌΠΎΠΆΠ½ΠΎΡΡ‚ΡŒ ΠΈΡΠΊΠ»ΡŽΡ‡Π΅Π½ΠΈΡ Π·Π½Π°Ρ‡Π΅Π½ΠΈΠΉ, ΠΊΠΎΡ‚ΠΎΡ€Ρ‹Π΅ сдСлали Π±Ρ‹ ΠΏΠΎΠ΄ΠΊΠΎΡ€Π΅Π½Π½ΠΎΠ΅ число ΠΎΡ‚Ρ€ΠΈΡ†Π°Ρ‚Π΅Π»ΡŒΠ½Ρ‹ΠΌ.

ΠŸΡ€Π΅ΠΆΠ΄Π΅ Ρ‡Π΅ΠΌ ΠΌΡ‹ Π½Π°Ρ‡Π½Π΅ΠΌ, Π΄Π°Π²Π°ΠΉΡ‚Π΅ рассмотрим ΠΏΡ€Π°Π²ΠΈΠ»Π° записи ΠΈΠ½Ρ‚Π΅Ρ€Π²Π°Π»ΠΎΠ²:

  • ΠŸΠ΅Ρ€Π²Ρ‹ΠΌ записываСтся наимСньший Ρ‡Π»Π΅Π½ ΠΈΠ½Ρ‚Π΅Ρ€Π²Π°Π»Π°.
  • Π‘Π°ΠΌΡ‹ΠΉ большой Ρ‡Π»Π΅Π½ Π² ΠΈΠ½Ρ‚Π΅Ρ€Π²Π°Π»Π΅ ΠΏΠΈΡˆΠ΅Ρ‚ΡΡ Π²Ρ‚ΠΎΡ€Ρ‹ΠΌ послС запятой.
  • ΠšΡ€ΡƒΠ³Π»Ρ‹Π΅ скобки ( ΠΈΠ»ΠΈ ) ΠΈΡΠΏΠΎΠ»ΡŒΠ·ΡƒΡŽΡ‚ΡΡ для обозначСния Ρ‚ΠΎΠ³ΠΎ, Ρ‡Ρ‚ΠΎ конСчная Ρ‚ΠΎΡ‡ΠΊΠ° Π½Π΅ Π²ΠΊΠ»ΡŽΡ‡Π΅Π½Π°, Ρ‡Ρ‚ΠΎ называСтся ΠΈΡΠΊΠ»ΡŽΡ‡ΠΈΡ‚Π΅Π»ΡŒΠ½Ρ‹ΠΌ.
  • Π‘ΠΊΠΎΠ±ΠΊΠΈ [ ΠΈΠ»ΠΈ ] ΠΈΡΠΏΠΎΠ»ΡŒΠ·ΡƒΡŽΡ‚ΡΡ для указания Ρ‚ΠΎΠ³ΠΎ, Ρ‡Ρ‚ΠΎ конСчная Ρ‚ΠΎΡ‡ΠΊΠ° Π²ΠΊΠ»ΡŽΡ‡Π΅Π½Π°, Ρ‡Ρ‚ΠΎ называСтся Π²ΠΊΠ»ΡŽΡ‡Π΅Π½ΠΈΠ΅ΠΌ.

Β 

ΠŸΡ€ΠΈΠΌΠ΅Ρ€. НахоТдСниС области опрСдСлСния Ρ„ΡƒΠ½ΠΊΡ†ΠΈΠΈ ΠΊΠ°ΠΊ Π½Π°Π±ΠΎΡ€Π° упорядочСнных ΠΏΠ°Ρ€

НайдитС ΠΎΠ±Π»Π°ΡΡ‚ΡŒ опрСдСлСния ΡΠ»Π΅Π΄ΡƒΡŽΡ‰Π΅ΠΉ Ρ„ΡƒΠ½ΠΊΡ†ΠΈΠΈ: [латСкс]\Π²Π»Π΅Π²ΠΎ\{\Π²Π»Π΅Π²ΠΎ(2,\тСкст{}10\Π²ΠΏΡ€Π°Π²ΠΎ),\Π²Π»Π΅Π²ΠΎ(3,\тСкст{}10\Π²ΠΏΡ€Π°Π²ΠΎ),\Π²Π»Π΅Π²ΠΎ(4, \text{ }20\right),\left(5,\text{ }30\right),\left(6,\text{ }40\right)\right\}[/latex] .

ΠŸΠΎΠΊΠ°Π·Π°Ρ‚ΡŒ Ρ€Π΅ΡˆΠ΅Π½ΠΈΠ΅

ΠŸΠΎΠΏΡ€ΠΎΠ±ΡƒΠΉΡ‚Π΅

НайдитС ΠΎΠ±Π»Π°ΡΡ‚ΡŒ опрСдСлСния Ρ„ΡƒΠ½ΠΊΡ†ΠΈΠΈ:

[латСкс]\Π²Π»Π΅Π²ΠΎ\{\Π²Π»Π΅Π²ΠΎ(-5,4\Π²ΠΏΡ€Π°Π²ΠΎ),\Π²Π»Π΅Π²ΠΎ(0,0\Π²ΠΏΡ€Π°Π²ΠΎ),\Π²Π»Π΅Π²ΠΎ(5,-4 \right),\left(10,-8\right),\left(15,-12\right)\right\}[/latex]

ΠŸΠΎΠΊΠ°Π·Π°Ρ‚ΡŒ Ρ€Π΅ΡˆΠ΅Π½ΠΈΠ΅

Как: Для Π·Π°Π΄Π°Π½Π½ΠΎΠΉ Ρ„ΡƒΠ½ΠΊΡ†ΠΈΠΈ, записанной Π² Π²ΠΈΠ΄Π΅ уравнСния, Π½Π°ΠΉΡ‚ΠΈ ΠΎΠ±Π»Π°ΡΡ‚ΡŒ опрСдСлСния. 9{3}[/латСкс].

ΠŸΠΎΠΊΠ°Π·Π°Ρ‚ΡŒ Ρ€Π΅ΡˆΠ΅Π½ΠΈΠ΅

ΠŸΡ€Π°ΠΊΡ‚ΠΈΡ‡Π΅ΡΠΊΠΎΠ΅ руководство. По Π·Π°Π΄Π°Π½Π½ΠΎΠΉ Ρ„ΡƒΠ½ΠΊΡ†ΠΈΠΈ, записанной Π² Π²ΠΈΠ΄Π΅ уравнСния, Π²ΠΊΠ»ΡŽΡ‡Π°ΡŽΡ‰Π΅Π³ΠΎ Π΄Ρ€ΠΎΠ±Π½ΡƒΡŽ Ρ‡Π°ΡΡ‚ΡŒ, Π½Π°ΠΉΠ΄ΠΈΡ‚Π΅ ΠΎΠ±Π»Π°ΡΡ‚ΡŒ опрСдСлСния.

  1. ΠžΠΏΡ€Π΅Π΄Π΅Π»ΠΈΡ‚Π΅ Π²Ρ…ΠΎΠ΄Π½Ρ‹Π΅ значСния.
  2. ΠžΠΏΡ€Π΅Π΄Π΅Π»ΠΈΡ‚Π΅ Π»ΡŽΠ±Ρ‹Π΅ ограничСния Π½Π° Π²Π²ΠΎΠ΄. Если Π² Ρ„ΠΎΡ€ΠΌΡƒΠ»Π΅ Ρ„ΡƒΠ½ΠΊΡ†ΠΈΠΈ Π΅ΡΡ‚ΡŒ Π·Π½Π°ΠΌΠ΅Π½Π°Ρ‚Π΅Π»ΡŒ, установитС Π·Π½Π°ΠΌΠ΅Π½Π°Ρ‚Π΅Π»ΡŒ Ρ€Π°Π²Π½Ρ‹ΠΌ Π½ΡƒΠ»ΡŽ ΠΈ Π½Π°ΠΉΠ΄ΠΈΡ‚Π΅ [latex]x[/latex] . Π­Ρ‚ΠΎ значСния, ΠΊΠΎΡ‚ΠΎΡ€Ρ‹Π΅ Π½Π΅ ΠΌΠΎΠ³ΡƒΡ‚ Π±Ρ‹Ρ‚ΡŒ Π²Π²Π΅Π΄Π΅Π½Ρ‹ Π² Ρ„ΡƒΠ½ΠΊΡ†ΠΈΡŽ.
  3. Π—Π°ΠΏΠΈΡˆΠΈΡ‚Π΅ Π΄ΠΎΠΌΠ΅Π½ Π² Ρ„ΠΎΡ€ΠΌΠ΅ ΠΈΠ½Ρ‚Π΅Ρ€Π²Π°Π»Π°, ΠΈΡΠΊΠ»ΡŽΡ‡ΠΈΠ² ΠΈΠ· Π΄ΠΎΠΌΠ΅Π½Π° Π»ΡŽΠ±Ρ‹Π΅ ΠΎΠ³Ρ€Π°Π½ΠΈΡ‡Π΅Π½Π½Ρ‹Π΅ значСния.

ΠŸΡ€ΠΈΠΌΠ΅Ρ€. НахоТдСниС области опрСдСлСния Ρ„ΡƒΠ½ΠΊΡ†ΠΈΠΈ, содСрТащСй Π·Π½Π°ΠΌΠ΅Π½Π°Ρ‚Π΅Π»ΡŒ (Ρ€Π°Ρ†ΠΈΠΎΠ½Π°Π»ΡŒΠ½Π°Ρ функция)

НахоТдСниС области опрСдСлСния Ρ„ΡƒΠ½ΠΊΡ†ΠΈΠΈ [latex]f\left(x\right)=\dfrac{x+1}{2-x}[ /латСкс].

ΠŸΠΎΠΊΠ°Π·Π°Ρ‚ΡŒ Ρ€Π΅ΡˆΠ΅Π½ΠΈΠ΅

ΠŸΠΎΡΠΌΠΎΡ‚Ρ€ΠΈΡ‚Π΅ ΡΠ»Π΅Π΄ΡƒΡŽΡ‰Π΅Π΅ Π²ΠΈΠ΄Π΅ΠΎ, Ρ‡Ρ‚ΠΎΠ±Ρ‹ ΡƒΠ²ΠΈΠ΄Π΅Ρ‚ΡŒ большС ΠΏΡ€ΠΈΠΌΠ΅Ρ€ΠΎΠ² Ρ‚ΠΎΠ³ΠΎ, ΠΊΠ°ΠΊ Π½Π°ΠΉΡ‚ΠΈ ΠΎΠ±Π»Π°ΡΡ‚ΡŒ опрСдСлСния Ρ€Π°Ρ†ΠΈΠΎΠ½Π°Π»ΡŒΠ½ΠΎΠΉ Ρ„ΡƒΠ½ΠΊΡ†ΠΈΠΈ (с Π΄Ρ€ΠΎΠ±ΡŒΡŽ).

ΠŸΠΎΠΏΡ€ΠΎΠ±ΡƒΠΉΡ‚Π΅

НайдитС ΠΎΠ±Π»Π°ΡΡ‚ΡŒ опрСдСлСния Ρ„ΡƒΠ½ΠΊΡ†ΠΈΠΈ: [latex]f\left(x\right)=\dfrac{1+4x}{2x — 1}[/latex].

ΠŸΠΎΠΊΠ°Π·Π°Ρ‚ΡŒ Ρ€Π΅ΡˆΠ΅Π½ΠΈΠ΅

Как: Для Π·Π°Π΄Π°Π½Π½ΠΎΠΉ Ρ„ΡƒΠ½ΠΊΡ†ΠΈΠΈ, записанной Π² Π²ΠΈΠ΄Π΅ уравнСния, Π²ΠΊΠ»ΡŽΡ‡Π°Ρ Ρ‡Π΅Ρ‚Π½Ρ‹ΠΉ ΠΊΠΎΡ€Π΅Π½ΡŒ, Π½Π°ΠΉΡ‚ΠΈ ΠΎΠ±Π»Π°ΡΡ‚ΡŒ опрСдСлСния.

  1. ΠžΠΏΡ€Π΅Π΄Π΅Π»ΠΈΡ‚Π΅ Π²Ρ…ΠΎΠ΄Π½Ρ‹Π΅ значСния.
  2. ΠŸΠΎΡΠΊΠΎΠ»ΡŒΠΊΡƒ имССтся Ρ‡Π΅Ρ‚Π½Ρ‹ΠΉ ΠΊΠΎΡ€Π΅Π½ΡŒ, ΠΈΡΠΊΠ»ΡŽΡ‡ΠΈΡ‚Π΅ всС Π΄Π΅ΠΉΡΡ‚Π²ΠΈΡ‚Π΅Π»ΡŒΠ½Ρ‹Π΅ числа, ΠΊΠΎΡ‚ΠΎΡ€Ρ‹Π΅ приводят ΠΊ ΠΎΡ‚Ρ€ΠΈΡ†Π°Ρ‚Π΅Π»ΡŒΠ½ΠΎΠΌΡƒ числу Π² ΠΏΠΎΠ΄ΠΊΠΎΡ€Π΅Π½Π½ΠΎΠΌ Ρ‡Π»Π΅Π½Π΅. УстановитС ΠΏΠΎΠ΄ΠΊΠΎΡ€Π΅Π½Π½ΠΎΠ΅ число большС ΠΈΠ»ΠΈ Ρ€Π°Π²Π½ΠΎΠ΅ Π½ΡƒΠ»ΡŽ ΠΈ Π½Π°ΠΉΠ΄ΠΈΡ‚Π΅ [латСкс]Ρ…[/латСкс].
  3. РСшСниС(я) ΡΠ²Π»ΡΡŽΡ‚ΡΡ ΠΎΠ±Π»Π°ΡΡ‚ΡŒΡŽ опрСдСлСния Ρ„ΡƒΠ½ΠΊΡ†ΠΈΠΈ. Если Π²ΠΎΠ·ΠΌΠΎΠΆΠ½ΠΎ, Π·Π°ΠΏΠΈΡˆΠΈΡ‚Π΅ ΠΎΡ‚Π²Π΅Ρ‚ Π² ΠΈΠ½Ρ‚Π΅Ρ€Π²Π°Π»ΡŒΠ½ΠΎΠΉ Ρ„ΠΎΡ€ΠΌΠ΅.

ΠŸΡ€ΠΈΠΌΠ΅Ρ€. НахоТдСниС области опрСдСлСния Ρ„ΡƒΠ½ΠΊΡ†ΠΈΠΈ с Ρ‡Π΅Ρ‚Π½Ρ‹ΠΌ ΠΊΠΎΡ€Π½Π΅ΠΌ

НайдитС ΠΎΠ±Π»Π°ΡΡ‚ΡŒ опрСдСлСния Ρ„ΡƒΠ½ΠΊΡ†ΠΈΠΈ [latex]f\left(x\right)=\sqrt{7-x}[/latex].

ΠŸΠΎΠΊΠ°Π·Π°Ρ‚ΡŒ Ρ€Π΅ΡˆΠ΅Π½ΠΈΠ΅

Π’ ΡΠ»Π΅Π΄ΡƒΡŽΡ‰Π΅ΠΌ Π²ΠΈΠ΄Π΅ΠΎ ΠΏΡ€ΠΈΠ²Π΅Π΄Π΅Π½Ρ‹ Π΄ΠΎΠΏΠΎΠ»Π½ΠΈΡ‚Π΅Π»ΡŒΠ½Ρ‹Π΅ ΠΏΡ€ΠΈΠΌΠ΅Ρ€Ρ‹ опрСдСлСния области опрСдСлСния Ρ„ΡƒΠ½ΠΊΡ†ΠΈΠΈ, содСрТащСй Ρ‡Π΅Ρ‚Π½Ρ‹ΠΉ ΠΊΠΎΡ€Π΅Π½ΡŒ.

ΠŸΠΎΠΏΡ€ΠΎΠ±ΡƒΠΉΡ‚Π΅

НайдитС ΠΎΠ±Π»Π°ΡΡ‚ΡŒ опрСдСлСния Ρ„ΡƒΠ½ΠΊΡ†ΠΈΠΈ [latex]f\left(x\right)=\sqrt{5+2x}[/latex].

ΠŸΠΎΠΊΠ°Π·Π°Ρ‚ΡŒ Ρ€Π΅ΡˆΠ΅Π½ΠΈΠ΅

Вопросы ΠΈ ΠΎΡ‚Π²Π΅Ρ‚Ρ‹

ΠœΠΎΠ³ΡƒΡ‚ Π»ΠΈ Π±Ρ‹Ρ‚ΡŒ Ρ„ΡƒΠ½ΠΊΡ†ΠΈΠΈ, Π² ΠΊΠΎΡ‚ΠΎΡ€Ρ‹Ρ… Π΄ΠΎΠΌΠ΅Π½ ΠΈ Π΄ΠΈΠ°ΠΏΠ°Π·ΠΎΠ½ Π²ΠΎΠΎΠ±Ρ‰Π΅ Π½Π΅ ΠΏΠ΅Ρ€Π΅ΡΠ΅ΠΊΠ°ΡŽΡ‚ΡΡ?

Π”Π°. НапримСр, функция [latex]f\left(x\right)=-\frac{1}{\sqrt{x}}[/latex] ΠΈΠΌΠ΅Π΅Ρ‚ мноТСство всСх ΠΏΠΎΠ»ΠΎΠΆΠΈΡ‚Π΅Π»ΡŒΠ½Ρ‹Ρ… Π΄Π΅ΠΉΡΡ‚Π²ΠΈΡ‚Π΅Π»ΡŒΠ½Ρ‹Ρ… чисСл Π² качСствС области опрСдСлСния, Π½ΠΎ мноТСство всСх ΠΎΡ‚Ρ€ΠΈΡ†Π°Ρ‚Π΅Π»ΡŒΠ½Ρ‹Π΅ Π΄Π΅ΠΉΡΡ‚Π²ΠΈΡ‚Π΅Π»ΡŒΠ½Ρ‹Π΅ числа Π² качСствС Π΄ΠΈΠ°ΠΏΠ°Π·ΠΎΠ½Π°. Как Π±ΠΎΠ»Π΅Π΅ ΠΊΡ€Π°ΠΉΠ½ΠΈΠΉ ΠΏΡ€ΠΈΠΌΠ΅Ρ€, Π²Ρ…ΠΎΠ΄Ρ‹ ΠΈ Π²Ρ‹Ρ…ΠΎΠ΄Ρ‹ Ρ„ΡƒΠ½ΠΊΡ†ΠΈΠΈ ΠΌΠΎΠ³ΡƒΡ‚ Π±Ρ‹Ρ‚ΡŒ ΡΠΎΠ²Π΅Ρ€ΡˆΠ΅Π½Π½ΠΎ Ρ€Π°Π·Π½Ρ‹ΠΌΠΈ катСгориями (Π½Π°ΠΏΡ€ΠΈΠΌΠ΅Ρ€, названия Π΄Π½Π΅ΠΉ Π½Π΅Π΄Π΅Π»ΠΈ Π² качСствС Π²Ρ…ΠΎΠ΄ΠΎΠ² ΠΈ числа Π² качСствС Π²Ρ‹Ρ…ΠΎΠ΄ΠΎΠ², ΠΊΠ°ΠΊ Π½Π° Π³Ρ€Π°Ρ„ΠΈΠΊΠ΅ посСщаСмости), Π² Ρ‚Π°ΠΊΠΈΡ… случаях Π΄ΠΎΠΌΠ΅Π½ ΠΈ Π΄ΠΈΠ°ΠΏΠ°Π·ΠΎΠ½ Π½Π΅ ΠΈΠΌΠ΅ΡŽΡ‚ ΠΎΠ±Ρ‰ΠΈΡ… элСмСнтов.

ΠŸΠΎΠΏΡ€ΠΎΠ±ΡƒΠΉΡ‚Π΅

Когда Π²Ρ‹ опрСдСляСтС ΠΎΠ±Π»Π°ΡΡ‚ΡŒ опрСдСлСния Ρ„ΡƒΠ½ΠΊΡ†ΠΈΠΈ, ΠΌΠΎΠΆΠ΅Ρ‚ ΠΏΠΎΠΌΠΎΡ‡ΡŒ Π΅Π΅ графичСскоС ΠΈΠ·ΠΎΠ±Ρ€Π°ΠΆΠ΅Π½ΠΈΠ΅, особСнно Ссли Ρƒ вас Π΅ΡΡ‚ΡŒ Ρ€Π°Ρ†ΠΈΠΎΠ½Π°Π»ΡŒΠ½ΠΎΠ΅ число ΠΈΠ»ΠΈ функция с Ρ‡Π΅Ρ‚Π½Ρ‹ΠΌ ΠΊΠΎΡ€Π½Π΅ΠΌ.

Π‘Π½Π°Ρ‡Π°Π»Π° ΠΎΠΏΡ€Π΅Π΄Π΅Π»ΠΈΡ‚Π΅ ограничСния Π΄ΠΎΠΌΠ΅Π½Π° для ΡΠ»Π΅Π΄ΡƒΡŽΡ‰ΠΈΡ… Ρ„ΡƒΠ½ΠΊΡ†ΠΈΠΉ, Π° Π·Π°Ρ‚Π΅ΠΌ нарисуйтС ΠΊΠ°ΠΆΠ΄ΡƒΡŽ ΠΈΠ· Π½ΠΈΡ…, Ρ‡Ρ‚ΠΎΠ±Ρ‹ ΠΏΡ€ΠΎΠ²Π΅Ρ€ΠΈΡ‚ΡŒ, согласуСтся Π»ΠΈ ваш Π΄ΠΎΠΌΠ΅Π½ с Π³Ρ€Π°Ρ„ΠΈΠΊΠΎΠΌ.

  1. [латСкс]f(x) = \sqrt{2x-4}+5[/латСкс]
  2. [латСкс]g(x) = \dfrac{2x+4}{x-1}[/латСкс]

Π—Π°Ρ‚Π΅ΠΌ ΠΈΡΠΏΠΎΠ»ΡŒΠ·ΡƒΠΉΡ‚Π΅ ΠΎΠ½Π»Π°ΠΉΠ½-инструмСнт для построСния Π³Ρ€Π°Ρ„ΠΈΠΊΠΎΠ², Ρ‡Ρ‚ΠΎΠ±Ρ‹ ΠΎΡ†Π΅Π½ΠΈΡ‚ΡŒ свою Ρ€Π°Π±ΠΎΡ‚Ρƒ ΠΏΡ€ΠΈ ΠΎΠ±Π½Π°Ρ€ΡƒΠΆΠ΅Π½Π½ΠΎΠΌ Π²Π°ΠΌΠΈ ΠΎΠ³Ρ€Π°Π½ΠΈΡ‡Π΅Π½ΠΈΠΈ Π΄ΠΎΠΌΠ΅Π½Π°. КакоС Ρ„ΡƒΠ½ΠΊΡ†ΠΈΠΎΠ½Π°Π»ΡŒΠ½ΠΎΠ΅ Π·Π½Π°Ρ‡Π΅Π½ΠΈΠ΅ Π΄Π°Π΅Ρ‚ Π²Π°ΠΌ Desmos?

Как: ИмСя Ρ„ΠΎΡ€ΠΌΡƒΠ»Ρƒ Ρ„ΡƒΠ½ΠΊΡ†ΠΈΠΈ, ΠΎΠΏΡ€Π΅Π΄Π΅Π»ΠΈΡ‚ΡŒ ΠΎΠ±Π»Π°ΡΡ‚ΡŒ опрСдСлСния ΠΈ Π΄ΠΈΠ°ΠΏΠ°Π·ΠΎΠ½.

  1. Π˜ΡΠΊΠ»ΡŽΡ‡ΠΈΡ‚ΡŒ ΠΈΠ· Π΄ΠΎΠΌΠ΅Π½Π° Π»ΡŽΠ±Ρ‹Π΅ Π²Ρ…ΠΎΠ΄Π½Ρ‹Π΅ значСния, ΠΊΠΎΡ‚ΠΎΡ€Ρ‹Π΅ приводят ΠΊ дСлСнию Π½Π° ноль.
  2. Π˜ΡΠΊΠ»ΡŽΡ‡ΠΈΡ‚ΡŒ ΠΈΠ· Π΄ΠΎΠΌΠ΅Π½Π° Π»ΡŽΠ±Ρ‹Π΅ Π²Ρ…ΠΎΠ΄Π½Ρ‹Π΅ значСния, ΠΊΠΎΡ‚ΠΎΡ€Ρ‹Π΅ ΠΈΠΌΠ΅ΡŽΡ‚ Π½Π΅Π΄Π΅ΠΉΡΡ‚Π²ΠΈΡ‚Π΅Π»ΡŒΠ½Ρ‹Π΅ (ΠΈΠ»ΠΈ Π½Π΅ΠΎΠΏΡ€Π΅Π΄Π΅Π»Π΅Π½Π½Ρ‹Π΅) числовыС Π²Ρ‹Ρ…ΠΎΠ΄Ρ‹.
  3. Π˜ΡΠΏΠΎΠ»ΡŒΠ·ΡƒΠΉΡ‚Π΅ допустимыС Π²Ρ…ΠΎΠ΄Π½Ρ‹Π΅ значСния для опрСдСлСния Π΄ΠΈΠ°ΠΏΠ°Π·ΠΎΠ½Π° Π²Ρ‹Ρ…ΠΎΠ΄Π½Ρ‹Ρ… Π·Π½Π°Ρ‡Π΅Π½ΠΈΠΉ.
  4. ΠŸΠΎΡΠΌΠΎΡ‚Ρ€ΠΈΡ‚Π΅ Π½Π° Π³Ρ€Π°Ρ„ΠΈΠΊ Ρ„ΡƒΠ½ΠΊΡ†ΠΈΠΈ ΠΈ Ρ‚Π°Π±Π»ΠΈΡ‡Π½Ρ‹Π΅ значСния, Ρ‡Ρ‚ΠΎΠ±Ρ‹ ΠΏΠΎΠ΄Ρ‚Π²Π΅Ρ€Π΄ΠΈΡ‚ΡŒ фактичСскоС ΠΏΠΎΠ²Π΅Π΄Π΅Π½ΠΈΠ΅ Ρ„ΡƒΠ½ΠΊΡ†ΠΈΠΈ.

ΠŸΡ€ΠΈΠΌΠ΅Ρ€: поиск Π΄ΠΎΠΌΠ΅Π½Π° ΠΈ Π΄ΠΈΠ°ΠΏΠ°Π·ΠΎΠ½Π° с ΠΏΠΎΠΌΠΎΡ‰ΡŒΡŽ Ρ„ΡƒΠ½ΠΊΡ†ΠΈΠΉ Π½Π°Π±ΠΎΡ€Π° инструмСнтов 9{3}-Ρ…[/латСкс].

Π”ΠΎΠ±Π°Π²ΠΈΡ‚ΡŒ ΠΊΠΎΠΌΠΌΠ΅Π½Ρ‚Π°Ρ€ΠΈΠΉ

Π’Π°Ρˆ адрСс email Π½Π΅ Π±ΡƒΠ΄Π΅Ρ‚ ΠΎΠΏΡƒΠ±Π»ΠΈΠΊΠΎΠ²Π°Π½. ΠžΠ±ΡΠ·Π°Ρ‚Π΅Π»ΡŒΠ½Ρ‹Π΅ поля ΠΏΠΎΠΌΠ΅Ρ‡Π΅Π½Ρ‹ *